You are on page 1of 136

Departamento de Matemticas

Matemticas I
Geometra
Copyright
c 2015 - I.E.S. Llanera

I.E.S. Llanera. Principado de Asturias


Imprimido: 17 de octubre de 2016
ndice

1. Trigonometra 1
1.1. Introduccin . . . . . . . . . . . . . . . . . . . . . . . . . . . . . . . . . . . . . . . . . 1
1.1.1. Representacin de ngulos y su medida . . . . . . . . . . . . . . . . . . . . . . 1
1.1.2. Razones trigonomtricas . . . . . . . . . . . . . . . . . . . . . . . . . . . . . . 3
1.1.3. Representacin grfica de tan . . . . . . . . . . . . . . . . . . . . . . . . . . 5
1.1.4. Relaciones fundamentales . . . . . . . . . . . . . . . . . . . . . . . . . . . . . 6
1.1.5. Razones trigonomtricas de 4 , 6 y 3 . . . . . . . . . . . . . . . . . . . . . . . 7
1.1.6. Relaciones entre ngulos relacionados . . . . . . . . . . . . . . . . . . . . . . . 8
1.2. Razones trigonomtricas y calculadora . . . . . . . . . . . . . . . . . . . . . . . . . . 9
1.2.1. Las unidades . . . . . . . . . . . . . . . . . . . . . . . . . . . . . . . . . . . . 9
1.2.2. Determinacin del ngulo . . . . . . . . . . . . . . . . . . . . . . . . . . . . . 10
1.3. Resolucin de tringulos . . . . . . . . . . . . . . . . . . . . . . . . . . . . . . . . . . 11
1.3.1. Razones trigonomtricas y tringulos rectngulos . . . . . . . . . . . . . . . . 12
1.3.2. Teorema del seno . . . . . . . . . . . . . . . . . . . . . . . . . . . . . . . . . . 13
1.3.3. Teorema del coseno . . . . . . . . . . . . . . . . . . . . . . . . . . . . . . . . . 17
1.3.4. Frmula de Hern . . . . . . . . . . . . . . . . . . . . . . . . . . . . . . . . . 19
1.4. Frmulas trigonomtricas . . . . . . . . . . . . . . . . . . . . . . . . . . . . . . . . . . 20
1.4.1. Relaciones de + . . . . . . . . . . . . . . . . . . . . . . . . . . . . . . . . . 20
1.4.2. Relaciones de . . . . . . . . . . . . . . . . . . . . . . . . . . . . . . . . . 22
1.4.3. Relaciones de 2 . . . . . . . . . . . . . . . . . . . . . . . . . . . . . . . . . . 23
1.4.4. Relaciones de 2 . . . . . . . . . . . . . . . . . . . . . . . . . . . . . . . . . . . 23
1.5. Identidades y ecuaciones trigonomtricas . . . . . . . . . . . . . . . . . . . . . . . . . 24
1.5.1. Identidades . . . . . . . . . . . . . . . . . . . . . . . . . . . . . . . . . . . . . 25
1.5.2. Ecuaciones . . . . . . . . . . . . . . . . . . . . . . . . . . . . . . . . . . . . . . 26
1.5.3. El cambio tan 2 = t . . . . . . . . . . . . . . . . . . . . . . . . . . . . . . . . . 30
Ejercicios . . . . . . . . . . . . . . . . . . . . . . . . . . . . . . . . . . . . . . . . . . . . . 31

2. Geometra afn en el plano 45


2.1. Vectores en el plano . . . . . . . . . . . . . . . . . . . . . . . . . . . . . . . . . . . . . 45
2.1.1. Operaciones con vectores . . . . . . . . . . . . . . . . . . . . . . . . . . . . . . 47
2.2. Coordenadas . . . . . . . . . . . . . . . . . . . . . . . . . . . . . . . . . . . . . . . . . 50
2.2.1. Bases . . . . . . . . . . . . . . . . . . . . . . . . . . . . . . . . . . . . . . . . . 52

iii
iv ndice

2.2.2. Sistemas de referencia . . . . . . . . . . . . . . . . . . . . . . . . . . . . . . . 54


2.3. Ecuacin de la recta . . . . . . . . . . . . . . . . . . . . . . . . . . . . . . . . . . . . 59
2.4. Procedimientos de la geometra afn . . . . . . . . . . . . . . . . . . . . . . . . . . . . 65
Ejercicios . . . . . . . . . . . . . . . . . . . . . . . . . . . . . . . . . . . . . . . . . . . . . 69

3. Geometra mtrica en el plano 75


3.1. Mtrica bsica . . . . . . . . . . . . . . . . . . . . . . . . . . . . . . . . . . . . . . . . 75
3.1.1. Mdulo de un vector . . . . . . . . . . . . . . . . . . . . . . . . . . . . . . . . 76
3.1.2. Distancia entre dos puntos . . . . . . . . . . . . . . . . . . . . . . . . . . . . . 77
3.1.3. ngulo de dos vectores . . . . . . . . . . . . . . . . . . . . . . . . . . . . . . . 78
3.2. Producto escalar . . . . . . . . . . . . . . . . . . . . . . . . . . . . . . . . . . . . . . 81
3.2.1. Relaciones mtricas del producto escalar . . . . . . . . . . . . . . . . . . . . . 81
3.3. Distancias . . . . . . . . . . . . . . . . . . . . . . . . . . . . . . . . . . . . . . . . . . 83
3.3.1. Distancia punto-recta . . . . . . . . . . . . . . . . . . . . . . . . . . . . . . . . 84
3.4. ngulos . . . . . . . . . . . . . . . . . . . . . . . . . . . . . . . . . . . . . . . . . . . 86
3.5. Procedimientos de la geometra mtrica . . . . . . . . . . . . . . . . . . . . . . . . . . 88
3.5.1. rea de un tringulo . . . . . . . . . . . . . . . . . . . . . . . . . . . . . . . . 88
3.5.2. Punto simtrico con respecto a r . . . . . . . . . . . . . . . . . . . . . . . . . 88
3.5.3. Recta simtrica con respecto a otra . . . . . . . . . . . . . . . . . . . . . . . . 89
3.5.4. Mediatriz de un segmento . . . . . . . . . . . . . . . . . . . . . . . . . . . . . 90
3.5.5. Bisectriz de un ngulo . . . . . . . . . . . . . . . . . . . . . . . . . . . . . . . 91
Ejercicios . . . . . . . . . . . . . . . . . . . . . . . . . . . . . . . . . . . . . . . . . . . . . 95

4. Cnicas 103
4.1. Circunferencia . . . . . . . . . . . . . . . . . . . . . . . . . . . . . . . . . . . . . . . . 104
4.1.1. Ecuacin . . . . . . . . . . . . . . . . . . . . . . . . . . . . . . . . . . . . . . . 104
4.1.2. Interseccin recta-circunferencia . . . . . . . . . . . . . . . . . . . . . . . . . . 105
4.1.3. Interseccin de dos circunferencias . . . . . . . . . . . . . . . . . . . . . . . . . 107
4.2. Elipse . . . . . . . . . . . . . . . . . . . . . . . . . . . . . . . . . . . . . . . . . . . . 108
4.2.1. Ecuacin reducida . . . . . . . . . . . . . . . . . . . . . . . . . . . . . . . . . 110
4.2.2. Elipse trasladada . . . . . . . . . . . . . . . . . . . . . . . . . . . . . . . . . . 112
4.2.3. Excentricidad . . . . . . . . . . . . . . . . . . . . . . . . . . . . . . . . . . . . 113
4.3. Hiprbola . . . . . . . . . . . . . . . . . . . . . . . . . . . . . . . . . . . . . . . . . . 114
4.3.1. Ecuacin reducida . . . . . . . . . . . . . . . . . . . . . . . . . . . . . . . . . 115
4.3.2. Hiprbolas trasladadas . . . . . . . . . . . . . . . . . . . . . . . . . . . . . . . 117
4.3.3. Asntotas . . . . . . . . . . . . . . . . . . . . . . . . . . . . . . . . . . . . . . 117
4.3.4. Excentricidad . . . . . . . . . . . . . . . . . . . . . . . . . . . . . . . . . . . . 118
4.4. Parbola . . . . . . . . . . . . . . . . . . . . . . . . . . . . . . . . . . . . . . . . . . . 119
4.4.1. Ecuacin reducida . . . . . . . . . . . . . . . . . . . . . . . . . . . . . . . . . 120
4.4.2. Parbolas trasladadas . . . . . . . . . . . . . . . . . . . . . . . . . . . . . . . 120
Ejercicios . . . . . . . . . . . . . . . . . . . . . . . . . . . . . . . . . . . . . . . . . . . . . 123
UNIDAD 1

Trigonometra

1.1. Introduccin

1.1.1. Representacin de ngulos y su medida


Para representar un ngulo, pondremos el vrtice de este en el origen de coordenadas de unos ejes,
una de las semirrectas que definen el ngulo sobre la parte positiva del eje OX y la otra semirrecta,
si el ngulo es positivo () abarcar la medida tomando esta en el sentido contrario a la marcha de
las agujas del reloj, si el ngulo es negativo (), la orientacin ser la de avance de las agujas del
reloj. Para la medicin de los ngulos utilizaremos grados y radianes.

Figura 1.1: Representacin de ngulos.

1
Grados. Un grado corresponde al ngulo que abarca un arco de 360 de circunferencia. Es la unidad
que se utiliza de forma tradicional para la medicin de ngulos.
Radianes. Un radin es el ngulo, que en una circunferencia de radio r, abarca una longitud de arco
igual a r.
Como la longitud del arco de una circunferencia de radio r es 2r, sabemos ya que el ngulo de 360
equivale a 2 radianes, lo que nos permitir pasar de radianes a grados y viceversa.
1.1. Introduccin 1. Trigonometra

Figura 1.2: ngulo de 1 radin.

La definicin dada para un radin no depende del radio de la circunferencia, pues la longitud del arco
y el radio de la circunferencia son proporcionales.
Ejercicio 1. Representa los ngulos: 270 , 90 , 180 , 12456 y 5642 .
Ejercicio 2. Representa los ngulos dados en radianes: 2 , 20
3
y 2
3
.
Ejercicio 3. Qu ngulo es mayor el de 1 radin o el de 60 ?

Nota 1.1.1 (Grados contra radianes). Estamos acostumbrados a utilizar los grados sexagesimales
y sus unidades menores, minutos y segundos y seguiremos usndolos en numerosos contextos. La
medida de los ngulos en radianes presenta una ventaja que los hace preferibles a las unidades en
grados, la ventaja radica en que se utiliza la misma unidad de medida para los arcos que definen el
ngulo y los segmentos rectilneos que derivan del ngulo, ello nos proporcionan ciertas ventajas de
las cuales disfrutaremos cuando trabajemos con las funciones trigonomtricas.
Es conveniente familiarizarse con la conversin de uno a otro sistema.


90 80 2
110 100 70 2
2
120 60 3 3
130 50 3
2 4
140 40

150 30 6
ad

ad

160 20
1r

1r

170 10 3
180 0 0

Los ngulos en grados Los ngulos en radianes

Debido a la conversin de grados a radianes, que a x le corresponde x


180
rad es conveniente usar,
siempre que sea posible, los ngulos en radianes referenciados a .

2
1. Trigonometra 1.1. Introduccin

1.1.2. Razones trigonomtricas


Dado un ngulo , llamamos razones trigonomtricas del ngulo , a
b
cos = a sen = b tan =
a
donde los valores a y b son las coordenadas del punto de corte del segmento que define el ngulo
con la circunferencia de radio unidad, llamada circunferencia goniomtrica o trigonomtrica. A partir

P (a, b)


O 1

Figura 1.3: Circunferencia goniomtrica.

de las definiciones anteriores son necesarias algunas observaciones.

Nota 1.1.2 (Acotacin). De la definicin de las razones trigonomtricas, para cualquier R, se


verifica que:

1 cos 1 1 sen 1

Basta observar en la Figura (1.3), que el punto P , que nos define las razones trigonomtricas se mueve
en la circunferencia unidad.
Algo parecido no podemos hacer con tan , observamos que para ngulos cercanos a 2 , cos se acerca

a cero haciendo el cociente sen
cos
cada vez mayor, segn nos acercamos.

Nota 1.1.3 (Frmulas de periodicidad). Los ngulos que se diferencian en un nmero entero de
vueltas tienen las mismas razones trigonomtricas.

cos = cos( + 2k)


sen = sen( + 2k)
tan = tan( + 2k)

donde k Z.

Nota 1.1.4. Para los ngulos de la forma 2 + k donde k Z, el punto de corte de la semirrecta
que define el ngulo con la circunferencia de radio unidad es de la forma P (0, 1), el clculo de la
tangente no es posible.

3
1.1. Introduccin 1. Trigonometra

 Es un error indicar que tan( 2 ) = . Lo adecuado decir que cuando el valor del ngulo se
va acercando a 2 (con < 2 ), el valor de tan se va haciendo cada vez mayor o dicho de
otra forma tiende hacia +.
Nota 1.1.5 (Razones trigonomtricas inmediatas). De forma inmediata tenemos las siguientes razones
trigonomtricas:

3
Radianes 0 2
2
2

Grados 0 90 180 270 360
sen 0 1 0 1 0
cos 1 0 1 0 1
tan 0 No 0 No 0

Nota 1.1.6 (Signo de las razones trigonomtricas). Es conveniente tener presente los signos de
las razones trigonomtricas dependiendo del cuadrante en el que est el ngulo. Si indicamos los
cuadrantes con I, II, III y IV, tenemos:
II I

cos < 0 cos > 0


sen > 0 sen > 0
tan < 0 tan > 0

cos < 0 cos > 0


sen < 0 sen < 0
tan > 0 tan < 0
III IV

Nota 1.1.7 (Razones recprocas). Aunque no con la importancia de las razones trigonomtricas,
en ocasiones nos aparecern las razones trigonomtricas recprocas o dicho de otra forma, razones
inversas para la multiplicacin.
Dado un ngulo , se definen:

cosecante secante cotangente


csc sec cot
1 1 cos
sen cos sen
Las anteriores razones no siempre estn definidas, csc no est definida para aquellos ngulos que
hacen que sea cero el sen , esto es k con k Z. De forma parecida sec est definida siempre salvo
cuando = k 2 con k Z. Con la cotangente ocurre lo mismo que con la cosecante, slo definida si
6= k con k Z.

Ejercicio 4. Representa los ngulos: 270 , 90 , 180 , 12456 y 5642 . Estima para cada
caso el valor de cos y sen .

4
1. Trigonometra 1.1. Introduccin

1.1.3. Representacin grfica de tan


A partir de la definicin del sen y del cos , tenemos una representacin grfica de dichas razones.
El sen tiene el valor de la proyeccin del punto de corte de la semirrecta que define el ngulo con la
circunferencia unidad (P ), sobre el eje OY . El cos corresponde a la proyeccin de P sobre el eje
OX. Veamos como tener una referencia grfica para la tangente de un ngulo. Para un ngulo

1 1
P 1 1
P

O A B A O B

Q
1 1

Figura 1.4: Representacin de tan .

situado en el cuadrante I, consideramos la proyeccin de la semirrecta que define el ngulo sobre la


recta que pasa por el punto (1, 0) y es perpendicular al eje OX. De esta forma tenemos dos tringulos
OAP y OBQ que son semejantes, luego

AP BQ
tan = = = BQ
OA OB
El segmento BQ representa la tangente de .
Si el ngulo est en el segundo cuadrante, hacemos algo parecido, proyectamos el punto P sobre la
misma recta. Nos vuelven a aparecer los mismos tringulos, que vuelven a ser semejantes, verificndose
la misma relacin. La nica diferencia es que ahora tan = ab , es un valor negativo, pues a < 0 y
b > 0. Es por lo que vamos a considerar la orientacin del segmento BQ negativa como el valor de la
tangente de .
La representacin de la tangente de ngulos de otros cuadrantes es similar, si el ngulo est en III, la
tangente ser positiva y su representacin estar en la parte superior de la perpendicular en (1, 0) y
si el ngulo est en IV, la tangente estar en la parte inferior de dicha recta.
Es interesante observar que con los criterios anteriores no es posible asignar un valor a tan( 2 + k)
con k Z a partir de la representacin de la tangente, aunque si podemos observar el comportamiento
de tan en las cercanas de 2 + k.
Ejercicio 5. Representa grficamente los siguientes ngulos y estima las razones trigonomtricas
que faltan:

5
1.1. Introduccin 1. Trigonometra

a) El ngulo que est en el cuadrante II y cos = 13 .


b) El ngulo en el cuadrante III y sen = 14 .
c) El ngulo que est en el cuadrante III y tan = 2.
d) El ngulo que est en el cuadrante IV y tan = 87 .
e) El ngulo con sen > 0 y tan = 32 .

1.1.4. Relaciones fundamentales


A la vista de la definicin de las razones trigonomtricas y de la observacin de la Figura (1.3),
tenemos que a2 + b2 = 1, de otra forma
cos2 + sen2 = 1 (1.1)
igualdad que es cierta para cualquier valor del ngulo .
Como tan = ab , podemos poner, siempre que a 6= 0
sen
tan = (1.2)
cos
Estas dos igualdades se conocen como relaciones fundamentales de la trigonometra, y nos permiten
conociendo una razn trigonomtrica calcular las dems.
Nota 1.1.8. Las expresiones cos2 y sen2 utilizadas en 1.1, es una simplificacin de las expresiones
(cos )2 y (sen )2 . No hay que confundir sen2 con la expresin sen 2 , en esta ltima elevamos
primero el ngulo al cuadrado y luego determinamos el seno del valor obtenido.
I Ejemplo 1.1.1. De un ngulo del cuadrante IV se sabe que cos = 13 . Hemos de calcular las
dems razones trigonomtricas.
De cos2 + sen2 = 1 y despejando sen tenemos
sen2 = 1 cos2

sen = 1 cos2

1 8
sen = 1 =
9 3
Vemos que hay dos posibles valores
para el sen , como sabemos que el ngulo seencuentra en IV,
8 sen
podemos asegurar que sen = 3 y como tan = cos , tenemos que tan = 8. /
I Ejemplo 1.1.2. Sabemos que tan = 2 y que sen < 0. Se ha de determinar el valor de sen y

cos . De tan = sen
cos
y despejando obtenemos que sen = 2 cos que sustituyendo en la primera
de las relaciones fundamentales obtenemos
cos2 + (2 cos )2 = 1
cos2 + 4 cos2 = 1
5 cos2 = 1

1 5
cos = =
5 5

6
1. Trigonometra 1.1. Introduccin

(a) (b) (c)


B
B
B

O A O A O A C

Figura 1.5: Razones trigonomtricas de 4 ,


6
y 3 .

Como en el ejemplo anterior hay dos posibles


valores para cos , el enunciado nos indica que el
ngulo
5 2 5
se encuentra en III, con lo que cos = 5 y como sen = 2 cos , resulta que sen = 5 . /

1.1.5. Razones trigonomtricas de 4 ,


6 y
3

El clculo de las razones trigonomtricas de los ngulos de 4 , 6 y 3 , se puede deducir a partir de


hacer consideraciones sencillas sobre los tringulos que forman la representacin de dichos ngulos en
la circunferencia de radio unidad.
ngulo de 4 . Corresponde al grfico (a) de la Figura (1.5). El tringulo que se forma es issceles,
los ngulos agudos son iguales y valen 4 . Podemos asegurar que OA
= AB = x, aplicando el
2 2 2
teorema de Pitgoras, x + x = 1 y despejando tenemos que x = 2 . Con lo que:

2 2
cos = sen = tan = 1
4 2 4 2 4

ngulo de 6 . Corresponde al grfico (b) de la Figura (1.5). Consideramos el tringulo OBC formado
por dos tringulos iguales OAB y OAC simtricos con respecto al eje OX. El tringulo OBC es
equiltero, pues sus ngulos son todos iguales. El segmento OA es una altura de dicho tringulo,
divide al segmento BC en dos segmentos iguales, con lo que:
2
1 1
AB = OA2 + =1
2 2

3
despejando OA se obtiene OA = 2
, de manera que:

3 1 1 3
cos = sen = tan = =
6 2 6 2 6 3 3

ngulo de 3 . Corresponde al grfico (c) de la Figura (1.5). Consideramos el tringulo OBC, es al


menos issceles pues los lados OB y OC son igual a 1, como el ngulo O es de 60 , los ngulos
C y B que son iguales han de ser de 60 . El segmento BA es una altura del tringulo, luego

7
1.1. Introduccin 1. Trigonometra

(a) (b) (c)

Figura 1.6: Razones trigonomtricas , , y + .

divide al lado OC en dos partes iguales, cada una de ellas igual a 12 . Al igual que en el caso
anterior calculamos la longitud de AB,
2
2 1 3
AB + = 1 AB =
2 2
Con lo que:

1 3
cos = sen = tan = 3
3 2 3 2 3

1.1.6. Razones trigonomtricas de ngulos relacionados


Para cualquier ngulo siempre se puede encontrar al menos un ngulo en el primer cuadrante de
manera que las razones trigonomtricas de ambas ngulos tengan entre s relaciones sencillas.
A continuacin estudiamos algn caso. Para demostrar las igualdades que vamos a ver, basta observar
la igualdad de tringulos que se forman.
Empezamos con
ngulos y . Situacin (a) de la Figura 1.6.
cos() = cos sen() = sen tan() = tan

ngulos y . Situacin (b) de la Figura 1.6.


cos( ) = cos sen( ) = sen tan( ) = tan

ngulos y + . Situacin (c) de la Figura 1.6.


cos( + ) = cos sen( + ) = sen tan( + ) = tan

ngulos y + 2 . Situacin (a) de la Figura 1.7.





1
cos + = sen sen + = cos tan + =
2 2 2 tan

8
1. Trigonometra 1.2. Razones trigonomtricas y calculadora

(a) (b) (c)

+
2
2


Figura 1.7: Razones trigonomtricas de , + 2 ,


2
y
2
.

ngulos y 2 . Situacin (b) de la Figura 1.7.





1
cos = sen sen = cos tan =
2 2 2 tan

ngulos y 2
. Situacin (c) de la Figura 1.7.




1
cos = sen sen = cos tan =
2 2 2 tan
Ejercicio 6. Teniendo en cuenta las razones trigonomtricas de los ngulos 30 , 45 y 60 ,
completa la tabla:


Radianes 6 4 3

Grados 30 45 60 120 135 150 210 225 240 300 315 330

1 2 3
sen 2 2 2

3 2 1
cos 2 2 2

3

tan 3
1 3

1.2. Razones trigonomtricas y calculadora


Sabemos estimar las razones trigonomtricas de un ngulo y recprocamente si de un ngulo sabemos
algunas de sus razones trigonomtricas podemos estimar de que ngulo (o ngulos) se trata. Las
estimaciones no suelen ser satisfactorias a la hora de resolver problemas, hemos de recurrir a otras
herramientas que nos proporcionen mejores aproximaciones, manejaremos para ello la calculadora.

1.2.1. Las unidades


La calculadora tiene la posibilidad de trabajar con tres unidades para la medicin de ngulos:

9
1.2. Razones trigonomtricas y calculadora 1. Trigonometra

Radianes. Para seleccionar el modo de trabajo en radianes por lo general se ha de elegir la opcin
MODE y a continuacin se elige RAD.
Grados. Un grado es el ngulo que abarca el arco que se obtiene cuando se divide la circunferencia
en 360 partes iguales. Para trabajar en grados se ha de elegir la opcin DEG. Para indicar los
submltiplos de un grado, hay dos posibilidades, utilizar un sistema decimal con lo que el
1 1
ngulo 12,56 indica que el ngulo es de 12 + 5 10 + 6 100 o utilizar el sistema sexagesimal
0 00
en el que 12 33 36 , que se lee 12 grados 33 minutos y 36 segundos e indica un ngulo de
1 1
12 + 33 60 + 36 3600 .
Normalmente se utiliza el formato en el que las fracciones de grado se expresan en el sistema
decimal, pero la calculadora nos permite pasar de uno a otro formato sin dificultad.
Gradianes. Un gradin es el ngulo que abarca el arco que se obtiene cuando se divide la circunferencia
en 400 partes iguales. Se activa eligiendo GRA. Este sistema para medir ngulos no lo utilizaremos.

1.2.2. Determinacin del ngulo


El problema de determinar el ngulo que verifica cos = 0,35 ya lo hemos tratado, representbamos
las posibles soluciones y estimbamos estas, cuando se necesita una mejor aproximacin hemos de
echar mano a la calculadora.
Para sacar el ngulo la calculadora dispone de la funcin cos1 que tambin se denota poniendo
arc cos 0,35 que se lee arco coseno o ngulo cuyo coseno es 0,35, en el ejemplo en el que estamos,
cuando hacemos cos1 0.35 , la calculadora nos devuelve un nico ngulo comprendido entre 0 y 180 ,
que denotaremos con c , en este caso c 69,51 , dicho ngulo no siempre coincide con el ngulo
buscado, hemos de utilizar c para calcular el ngulo buscado .
Veremos con varios ejemplos como podemos calcular a partir del ngulo que nos devuelve la calculadora
el ngulo que nos interesa.
I Ejemplo 1.2.1. Determinaremos el ngulo en el IV cuadrante de manera que cos = 0,35
(Figura 1.9 a).
Calculamos c , c = arc cos 0,35 69,51 ngulo del primer cuadrante. Vemos que o bien el ngulo
c o bien 360 c nos resuelve el problema planteado. /
De forma parecida se resuelven los problemas de determinar un ngulo del que se conoce su seno o su
tangente, nicamente hemos de tener en cuenta que no siempre coincide el ngulo devuelto por la
calculadora (c ) con el que nos resuelve el problema que se nos plantea ().
Al igual que antes, la funcin sen1 y tan1 de la calculadora, corresponden respectivamente a arc sen
y arctan funciones que se leen, arco coseno y arco tangente.
En Figura 1.8 se indica en cada caso el intervalo de ngulos que devuelve cada una de las funciones
arc cos, arc sen y arctan.
I Ejemplo 1.2.2. Se sabe de que sen = 0,75 y que el ngulo est en el III cuadrante, de qu
ngulo se trata?
La calculadora nos devuelve c = arc sen 0,75 48,59 , ngulo del IV cuadrante. El ngulo
buscado es = 180 c 228,59 (Figura 1.9 b). /

10
1. Trigonometra 1.3. Resolucin de tringulos

arc cos x arc sen x arctan x



2 2

2 2

Figura 1.8: Rango de arc cos, arc sen y arctan.

(a) (b) (c)

c c

Figura 1.9: Ejemplos.

I Ejemplo 1.2.3. De un ngulo se sabe que tan = 2 y que cos < 0, de qu ngulo se trata?
La calculadora nos devuelve c = arctan 2 63,43 , que no verifica la segunda de las condiciones
impuestas. El ngulo buscado est en el III cuadrante = c + 180 243,43 (Figura 1.9 c). /

 Hemos de tener un cuidado especial con las notaciones sen


1
1
x y (sen x)1 . La primera de
ellas, sen x hace referencia al ngulo cuyo seno es x, es preferible para indicar lo mismo
el uso de la notacin arc sen x, que evita ambigedades. Nos referimos a sen1 o a arc sen como la
inversa de sen, esto es, deshacen lo que hace sen. Con (sen x)1 estamos indicando sen1 x o dicho de
otra forma csc x, para referirnos a esta expresin indicamos que es el recproco de sen x o inverso
multiplicativo. La misma precaucin hemos de tener con las dems razones trigonomtricas.

1.3. Resolucin de tringulos


Se conoce con el nombre de resolucin de tringulos los problemas en los que hay calcular elementos
de un tringulo, generalmente lados y ngulos, a partir de algunos elementos conocidos.
Para cualquier tringulo se verifica:
La suma de los ngulos de un tringulo es 180 .
La suma de dos de los lados de un tringulo es mayor que el otro.

11
1.3. Resolucin de tringulos 1. Trigonometra

A partir de ahora para nombrar a los ngulos de los tringulos utilizaremos letras maysculas (A, B y
C) y para los lados opuestos a dichos ngulos la misma letra pero en minscula (ver Figura 1.10).

1.3.1. Razones trigonomtricas y tringulos rectngulos


En los tringulos rectngulos (ver Figura 1.10), a parte de lo anterior, se verifica el Teorema de
Pitgoras:
b 2 = a2 + c 2
o de forma literal: La suma de los cuadrados de los catetos coincide con el cuadrado de la hipotenusa.
El Teorema de Pitgoras, nos dice ms cosas, nos asegura que si con los lados de un tringulo se
verifica una igualdad del tipo b2 = a2 + c2 podemos tener la garanta de que el tringulo es rectngulo
y que la hipotenusa es el lado b.
Ejercicio 7. De los siguientes tringulos indica cules son rectngulos:

a) 5, 4 y 3. c) 65, 72 y 97.

b) 41, 40 y 9. d) 5, 7 y 2 3.

Otra ventaja que tienen los tringulos rectngulos sobre otros, es que dos de las alturas del tringulo
coinciden con dos de los lados, los catetos. Lo anterior facilita el clculo del rea del tringulo.
1
C
Q
C
b a
A P B
A c B

Figura 1.10: Razones trigonomtricas en un tringulo rectngulo.

En lo que respecta a las razones trigonomtricas de los ngulos agudos de un tringulo rectngulo se
dan unas relaciones que nos permiten la resolucin de estos teniendo el conocimiento de nicamente
dos de los elementos del tringulo, siempre que uno de los elementos sea un lado.
Trasladando el tringulo a la circunferencia de radio unidad, tenemos dos tringulos que son semejantes
con lo que:
AB AP
= = AP = cos A
AC AQ
BC PQ
= = P Q = sen A
AC AQ
BC AP
= = tan A
AB PQ

12
1. Trigonometra 1.3. Resolucin de tringulos

utilizando el nombre de los lados


c a a
cos A = sen A = tan A =
b b c
 Las anteriores igualdades nos dan relaciones entre las razones trigonomtricas de los ngulos
agudos de un tringulo rectngulo y sus lados. nicamente son vlidas en un tringulo
rectngulo!

Se suelen redactar las anteriores igualdades sin hacer referencia a la notacin utilizada para nombrar
a los lados:
cos A = Cociente entre el cateto contiguo y la hipotenusa.
sen A = Cociente entre el cateto opuesto y la hipotenusa.
tan A = Cociente entre el cateto opuesto y el contiguo.

I Ejemplo 1.3.1. En un tringulo rectngulo con (B = 90 ), un cateto mide a = 7 y el ngulo


C = 40 . Hemos de determinar los dems elementos del tringulo.
Como conocemos el ngulo C y el cateto contiguo a dicho ngulo es el a, se da la relacin tan C = ac
que despejando c = a tan C, que nos da el valor del otro cateto. /

1.3.2. Teorema del seno


El resultado que vamos a ver se llama Teorema del seno y nos sirve para resolver cualquier tipo de
tringulo, con la condicin de que conozcamos tres de sus elementos, dos de ellos enfrentados, esto es
un lado y el ngulo opuesto.

(i) Tringulo acutngulo (ii) Tringulo obtusngulo


C C

H2
a a
b h h1 b
T1 T2 h2
D

A c B H1 A c B

Figura 1.11: Teorema del seno.

Razonemos primero sobre un tringulo acutngulo. En el tringulo (i) de la Figura 1.11 la altura que
parte de C determina dos tringulos rectngulos (T1 y T2 ) a los que se les puede aplicar lo visto en la
seccin anterior.
En T1 podemos poner h = b sen A y de h = a sen B, igualando obtenemos

b sen A = a sen B

13
1.3. Resolucin de tringulos 1. Trigonometra

que se puede expresar de cualquiera de las formas:

a b sen A sen B
= =
sen A sen B a b
razonando con las dems alturas del tringulo se deducen igualdades similares.
Resumidamente podemos expresar la totalidad de las igualdades que se dan de cualquiera de las
formas:
a b c sen A sen B sen C
= = = =
sen A sen B sen C a b c
El anterior razonamiento se puede aplicar aunque el tringulo que tengamos sea obtusngulo. Consi-
deremos el tringulo (ii) de la Figura 1.11, con un ngulo obtuso en A.
Trazamos la altura h1 desde el vrtice C que con la prolongacin del lado c se corta en el punto H1 .
Trazamos la altura h2 desde el vrtice A que corta al lado a en el punto H2 .
De considerar los tringulos ABH2 y ACH2 , obtenemos:

h2
2 = c sen B
sen B = h b c
c
h2
c sen B = b sen C = (1.3)
sen C =
b
h2 = b sen C sen B sen C

Si consideramos los tringulos CBH1 y ACH1 , obtenemos:



h1
sen B = a
h
1 = a sen B a b
h1
a sen B = b sen D = (1.4)
sen D =
b
h1 = b sen D sen D sen B

Como los ngulo A y D son suplementarios, se verifica que sen A = sen D, y de (1.3) y (1.4),
nuevamente obtenemos:
a b c
= = .
sen A sen B sen C
O lo que es lo mismo:
sen A sen B sen C
= =
a b c
Nota 1.3.1. Las anteriores igualdades nos indica que en un tringulo, el cociente entre lo que mide
un lado y el seno del ngulo opuesto al lado es una constante.
Veamos que dicho valor coincide con el dimetro de la circunferencia que circunscribe al tringulo
ABC.
a. Sabemos que el ngulo con vrtice en la circunferencia (ngulo inscrito) es la mitad del ngulo con
vrtice en el centro y abarcando el mismo arco de circunferencia que el primero (Figura 1.12).
De lo anterior se deduce que todos los ngulos con vrtice en la circunferencia y abarcando el
mismo arco de circunferencia son iguales.

14
1. Trigonometra 1.3. Resolucin de tringulos

(a) (b)
C C

A0

A A
B B

Figura 1.12: ngulo inscrito y ngulo central.

b. Si tenemos el tringulo ABC y la circunferencia circunscrita podemos considerar el tringulo


A0 BC, Figura 1.12 (b), el tringulo A0 BC tiene en comn con el ABC el ngulo A y el lado
a y adems es un tringulo rectngulo pues uno de los lados coincide con el dimetro de la
circunferencia.
En el tringulo A0 BC, tenemos que sen A0 = 2ra
con lo que 2r = sena A .
Nota 1.3.2. En un tringulo, a menor lado le corresponde menor ngulo.
De sena A = senb B = senc C , sabemos que los lados son proporcionales al seno del ngulo opuesto, nos
falta ver que en los ngulos de un tringulo a menor seno le corresponde menor ngulo. Veamos que
no se puede dar la situacin,
sen > sen >
Si es un ngulo agudo claramente > , si es un ngulo obtuso, y sen > sen , para que ocurra
que > ha de estar en el intervalo (180 , ), con lo que la suma de y sobrepasara 180 ,
con lo que:
sen > sen >
Ejercicio 8. Demuestra que todo tringulo con vrtices en una circunferencia y uno de los
lados siendo un dimetro de esta, es rectngulo.
I Ejemplo 1.3.2. De un tringulo se conoce a = 36, b = 50 y A = 30 , se han de determinar los
dems elementos.
Aplicamos la igualdad sena A = senb B despejando obtenemos sen B = 50 sen
36
30
0,694, Hemos de buscar

los ngulos que sen B 0,694, la calculadora nos devuelve B1 43,98 , pero hay un ngulo en el II
cuadrante que tiene el mismo seno B2 = 180 B1 136 .
Obtenemos pues dos soluciones posibles para el tringulo buscado C1 = 180 A B1 106 y
C2 = 180 A B2 14 (Figura 1.13).
Para cada uno de los ngulos calculados podemos calcular el lado que falta c, aplicando sena A = senc C ,
obtenemos las soluciones:
36 sen 106
c1 = 69,21
sen 30
y utilizando el otro valor del ngulo calculado
36 sen 14
c2 = 17,4
sen 30

15
1.3. Resolucin de tringulos 1. Trigonometra

B2
B1
C
A
a = 36

A b = 50 C
A C

Figura 1.13: Dos soluciones.

Obtenemos dos construcciones posibles. /

I Ejemplo 1.3.3. De un tringulo se conoce a = 20, b = 50 y A = 30 , se han de determinar los


dems elementos.

a = 20

A b = 50 C

Figura 1.14: Sin solucin.


Aplicamos la igualdad sena A = senb B despejando obtenemos sen B = 50 sen
20
30
= 1,25. Obviamente hay
un problema, pues sabemos que | sen | 1, nos encontramos con una situacin en la que no es
posible determinar el tringulo, en la Figura 1.14 se puede observar lo que ocurre. /

I Ejemplo 1.3.4. De un tringulo se conoce a = 25, b = 50 y A = 30 , se han de determinar los


dems elementos.

a = 25

A b = 50 C

Figura 1.15: Una nica solucin.

16
1. Trigonometra 1.3. Resolucin de tringulos


Al igual que en los casos anteriores tenemos sen B = 50 sen
25
30
= 1. Ahora lo que ocurre es que el ngulo

slo puede ser B = 90 , el tringulo es rectngulo. El clculo de los dems elementos es inmediato. /

Para finalizar la seccin recordamos el objetivo de esta.


Teorema 1.3.1 (Teorema del seno). En un tringulo ABC, se verifica:
a b c
= = = 2R
sen A sen B sen C
siendo a, b y c los lados del tringulo y R el radio de la circunferencia que circunscribe el tringulo
ABC.

1.3.3. Teorema del coseno


El teorema del coseno nos va a permitir la resolucin de tringulos conociendo tres elementos, ninguno
de ellos enfrentados.

b h a
T1 T2

A m B
c

Figura 1.16: Teorema del coseno.

En Figura 1.16 observando el tringulo T1 , vemos que m = b cos A. Si en el mismo tringulo aplicamos
el teorema de Pitgoras obtenemos

b2 = h2 + m2 = h2 + b2 cos2 A (1.5)

si hacemos lo mismo en el tringulo T2 , se obtiene

a2 = h2 + (c m)2 = h2 + c2 + m2 2mc (1.6)

Podemos expresar (1.6), de la forma:

a2 = h2 + c2 + b2 cos2 A 2bc cos A (1.7)

Considerando (1.5) y (1.7) podemos expresar la diferencia a2 b2 , de la forma

a2 b2 = c2 2bc cos A

17
1.3. Resolucin de tringulos 1. Trigonometra

despejando
a2 = b2 + c2 2bc cos A
que nos indica que conociendo b, c y A, podemos calcular el lado a.
Podemos razonar de igual manera con cada una de las alturas del tringulo, para obtener las relaciones

b2 = a2 + c2 2ac cos B
c2 = a2 + b2 2ab cos C

I Ejemplo 1.3.5. De un tringulo conocemos los tres lados a = 7, b = 8 y c = 12, hemos de


determinar los ngulos del tringulo.
Observamos que no disponemos de datos enfrentados, aplicamos el teorema del coseno, a2 = b2 + c2
2bc cos A, despejando cos A

a2 b 2 c 2 159
cos A = = 0,828
2bc 192
tenemos ya el ngulo A, A arc cos 0,828 34,09 .
Aunque ya disponemos de elementos enfrentados y podramos aplicar el teorema del seno, es preferible
aplicar otra vez el teorema del coseno que nos da directamente otro de los ngulos. Aplicando
2 2 c2
cos B = b a2ac
0,767, tenemos otro ngulo B 39,83 , el que falta se calcula directamente
C = 180 A B 106 . /

I Ejemplo 1.3.6. De un tringulo conocemos los tres lados a = 50, b = 25 y c = 20, hemos de
determinar los ngulos del tringulo.
2 2 c2
Determinemos el ngulo A, para lo cual cos A = a b
2bc
= 1,475, no existe tal ngulo, el tringulo
no se puede determinar.

25
20

50

Figura 1.17: Ejemplo sin solucin.

En Figura 1.17 se puede observar lo que ocurre. /

Ejercicio 9. Determina los ngulos de un tringulo, del que se conoce: a = 50, b = 30 y c = 20.
Da una explicacin de lo que ocurre.
Finalizamos la seccin enunciando el resultado visto.

18
1. Trigonometra 1.3. Resolucin de tringulos

Teorema 1.3.2 (Teorema del coseno). En un tringulo ABC, se verifican las identidades:

a2 = b2 + c2 2bc cos A
b2 = a2 + c2 2ac cos B
c2 = a2 + b2 2ab cos C

donde a, b y c son los lados del tringulo.

1.3.4. Frmula de Hern


La frmula de Hern, nos relaciona el rea de un tringulo con las longitudes de los lados.

Proposicin 1.3.1. En un tringulo de lados a, b y c, el rea S, viene expresada por:



S= s(s a)(s b)(s c) (1.8)

a+b+c
donde s es el semipermetro, esto es, s = 2

Demostracin:

b h a

A c B

Por el teorema del cosenos sabemos que:

a2 = b2 + c2 2bc cos A
b 2 + c 2 a2
cos A =
2bc

A partir de sen2 A + cos2 A = 1, despejando obtenemos:



2
b + c2 a2
2
4b2 c2 (b2 + c2 a2 )2
sen A = 1 cos2 A = 1 =
2bc 2bc

19
1.4. Frmulas trigonomtricas 1. Trigonometra

1
El rea del tringulo viene expresado por S = 2
ch, de donde:
1 1
S= ch= cb sen A =
2 2
1 4b2 c2 (b2 + c2 a2 )2
= cb =
2 2bc
1
= 4b2 c2 (b2 + c2 a2 )2 =
4
1
= (2bc (b2 + c2 a2 ))(2bc + (b2 + c2 a2 )) =
4
1
= (a2 (b c)2 )((b c)2 a2 ) =
4
1
= (a (b c))(a + (b c))(b c a)(b c + a) =
4
1
= (a b + c)(a + b c)(b c a)(b c + a)
4
Como 2s = a + b + c, podemos concluir:

S= s(s a)(s b)(s c)

1.4. Frmulas trigonomtricas


En esta seccin veremos ciertas relaciones en las que aparecen las razones trigonomtricas de los
ngulos , + , 2 y 2 relacionadas con las relaciones trigonomtricas de y , por lo general
tiles para le resolucin de ecuaciones trigonomtricas. De forma que el conocimiento de las razones
trigonomtricas de y nos permitir el clculo de las razones trigonomtricas de otros ngulos
relacionados con y .

1.4.1. Relaciones de +
Observemos la figura (1.18).
Observamos varios tringulos: OBC, OAE, OCE y CDE. Todos ellos tienen una particularidad que
nos resulta ventajosa es el hecho de ser rectngulos. Observamos que los tringulos OBC y CDE son
semejantes.
Recorramos uno por uno los tringulos expresando las razones trigonomtricas del ngulo marcado.
En el tringulo OBC, tenemos
BC
sen = BC = OC sen
OC
OB
cos = OB = OC cos
OC

20
1. Trigonometra 1.4. Frmulas trigonomtricas

E D

C
+


O A B

Figura 1.18: Relaciones con + .

En el tringulo CDE, tenemos


DE
sen = DE = CE sen
CE
CD
cos = CD = CE cos
CE
En el tringulo OCE, tenemos
CE
sen = DE = CE sen
OE
OC
cos = CD = CE cos
OE
En el tringulo OAE,
AE BD BC + CD BC CD
sen( + ) = = = = + =
OE OE OE OE OE
OC sen CE cos
= + = sen cos + cos sen (1.9)
OE OE
OA OB DE OB DE
cos( + ) = = = =
OE OE OE OE
OC cos CE sen
= = cos cos sen sen (1.10)
OE OE

sen(+)
Tangente de + . Sabemos que tan( + ) = cos(+)
, sustituyendo denominador y numerador
por los desarrollos (1.9) y (1.10), obtenemos
sen cos + cos sen
tan( + ) =
cos cos sen sen

21
1.4. Frmulas trigonomtricas 1. Trigonometra

dividiendo numerador y denominador por cos cos , se obtiene

sen cos cos sen


cos cos
+ cos cos tan + tan
tan( + ) = cos cos sen sen =
cos cos
cos cos
1 tan tan

I Ejemplo 1.4.1. Conocemos las razones trigonomtricas de los ngulos 4 y


6
(ver seccin 1.1.5),

pretendemos calcular las razones trigonomtricas de 5
12
. Observamos que 5
12
= 4
+ 6 , luego:

5

sen = sen + = sen cos + cos sen =
12 4 6 4 6 4 6
2 3 21 2( 3 + 1)
= + =
2 2 2 2 4
5
cos = cos + = cos cos sen sen =
12 4 6 4 6 4 6
2 3 21 2( 3 1)
= =
2 2 2 2 4
5
tan 4 + tan 6
tan = tan + = =
12 4 6 1 tan 4 tan 6

1 + 33 3+3
= = = 2 + 3
1 1 33 33

 Es un error comn, debido a la inercia del lgebra, utilizar falsas identidades del tipo:
sen( + ) = sen + sen
cos( + ) = cos + cos

es conveniente extremar la precaucin y tener presentes las identidades (1.9) y (1.10).

1.4.2. Relaciones de
Basta darse cuenta que = + () y aplicar las identidades vistas en la seccin anterior,
teniendo en cuenta la relacin entre las razones trigonomtricas de y , esto es cos() = cos ,
sen() = sen y tan() = tan .

22
1. Trigonometra 1.4. Frmulas trigonomtricas

sen( ) = sen( + ()) = sen cos() + cos sen() =


= sen cos cos sen
cos( ) = cos( + ()) = cos cos() sen sen() =
= cos cos + sen sen
tan + tan()
tan( ) = tan( + ()) = =
1 tan tan()
tan tan
=
1 + tan tan

Ejercicio 10. Los vrtices A y B del tringulo OAB, se encuentran sobre la circunferencia
unidad. Demuestra, usando el teorema del coseno en le tringulo OAB que cos( ) = cos cos +
sen sen .

d
B


O

1.4.3. Relaciones de 2
Basta hacer uso de 2 = + y aplicar las identidades vistas en la seccin anterior.
sen 2 = sen( + ) = sen cos + cos sen = 2 sen cos
cos 2 = cos( + ) = cos cos sen sen = cos2 sen2
tan + tan 2 tan
tan 2 = tan( + ) = =
1 tan tan 1 tan2


1.4.4. Relaciones de 2

De forma parecida a la seccin anterior, utilizaremos que = 2
+ 2 .


Coseno de 2
. Utilizaremos las identidades:

cos = cos2 sen2 (1.11)
2 2
2 2
1 = cos + sen (1.12)
2 2

23
1.5. Identidades y ecuaciones trigonomtricas 1. Trigonometra

sumando las dos identidades, obtenemos



cos + 1 = 2 cos2
2
despejando cos 2 , se obtiene
1 + cos
cos =
2 2
El signo que se toma en la frmula depende del cuadrante en el que est situado el ngulo . Por
ejemplo si est en el cuadrante III, 2 estar en II, con lo que se tomara signo negativo.


Seno de 2
. Si restamos las identidades (1.11) y (1.12), obtenemos

1 cos = 2 sen2
2
y despejando
1 cos
sen =
2 2

sen
Tangente de 2 . Es ya inmediato, teniendo en cuenta que tan 2 = cos 2 , y utilizando las dos frmulas
2
anteriores deducir
1 cos
tan =
2 1 + cos
Nota 1.4.1 (Relaciones). Recogemos las relaciones vistas hasta el momento:
tan + tan
sen( + ) = sen cos + cos sen tan( + ) =
1 tan tan
cos( + ) = cos cos sen sen
tan tan
sen( ) = sen cos cos sen tan( ) =
1 + tan tan
cos( ) = cos cos + sen sen
2 tan
sen 2 = 2 sen cos tan 2 =
1 tan2
cos 2 = cos2 sen2

1 cos 1 cos
sen = tan =
2
2 2 1 + cos
1 + cos
cos =
2 2

1.5. Identidades y ecuaciones trigonomtricas


Las identidades y ecuaciones trigonomtricas son igualdades en las que la variable (o variables) aparece
afectada por una razn trigonomtrica.

24
1. Trigonometra 1.5. Identidades y ecuaciones trigonomtricas

1.5.1. Identidades
Las identidades son igualdades que son siempre ciertas con independencia del valor que tome la
variable, por ejemplo
cos2 + sen2 = 1
es una identidad. La igualdad es siempre cierta.
Utilizaremos esta identidad y otras de las vistas con anterioridad para demostrar otras. Con unos
ejemplos veremos la forma de proceder.
I Ejemplo 1.5.1. Probar que se verifica la identidad,
1 + cos sen
=
sen 1 cos
siempre que sen 6= 0.
Vemos una igualdad de fracciones, para ver si es cierta basta ver que el producto cruzado nos da lo
mismo, es decir, (1 + cos )(1 cos ) = sen2 , operando

(1 + cos )(1 cos ) = 1 cos2 = sen2


Queda demostrada la identidad. /
I Ejemplo 1.5.2. Veamos que es cierta la identidad
1
= cos + sen tan
cos
siempre que cos 6= 0.
Pondremos el segundo miembro de la igualdad en trminos de cos y sen

sen sen2
cos + sen tan = cos + sen = cos + =
cos cos
cos2 sen2 cos2 + sen2 1
= + = =
cos cos cos cos
Partiendo de uno de los miembros llegamos al otro, la identidad est demostrada. /
I Ejemplo 1.5.3. Veamos que es cierta la identidad:

2 tan cos2 sen = tan
2
En esta igualdad nos aparece una de las razones trigonomtricas con un ngulo diferente 2 . El primer
paso a dar consistir en poner todas las razones trigonomtricas referidas al mismo ngulo , para lo
cual hacemos uso de la identidad, ya demostrada,
1 + cos
cos2 =
2 2

25
1.5. Identidades y ecuaciones trigonomtricas 1. Trigonometra

tenemos

1 + cos
2 tan cos2 sen = 2 tan sen =
2 2
(1 + cos ) sen
= sen =
cos
sen + sen cos cos sen
= = tan
cos
/

1.5.2. Ecuaciones
Antes de empezar a complicarnos, conviene reflexionar sobre algunas ecuaciones inmediatas.
sen x = a. Solamente tiene solucin si |a| 1. En esta situacin hay una solucin en el intervalo
[ 2 , 2 ] y otra en el intervalo [ 2 , 3
2
], que seran, aunque no necesariamente en el orden que se
dan, las soluciones: x1 = arc sen a y x2 = arc sen a.
A partir de las anteriores soluciones es importante tener en cuenta que cada una de ellas
genera una un conjunto infinito de soluciones dado por X1 = {arc sen a + 2k} y X2 =
{(2k + 1) arc sen a}, con k Z.
cos x = a. Slamente tiene solucin si |a| 1. Hay una solucin en [0, ] y otra en el intervalo [, 2],
que seran, los valores: x1 = arc cos a y x2 = 2 arc cos a.
Y al igual que antes, por cada uno de los valores hay una infinidad de valores solucin de
la ecuacin, que seran los valores que se consiguen cuando aadimos 2k. Tenemos pues
X1 = {arc cos a + 2k} y X2 = {2k arc cos a} con k Z.
tan x = a. Siempre tiene solucin. En esta situacin hay una solucin en el intervalo [ 2 , 2 ] y otra
en el intervalo [ 2 , 3
2
], que sern: x1 = arctan a y x2 = + arctan a.
Nuevamente tenemos el conjunto de soluciones

X1 = {arctan a + 2k} y X2 = {(2k + 1) + arctan a},

que se puede expresar de forma nica como X = {arctan a + k} con k Z.


Es importante tener en todo momento presente que aunque nos centremos en la bsqueda de soluciones
a las ecuaciones que se nos propone en el intervalo [0, 2], a partir de estas se pueden obtener otras
soluciones fuera del intervalo.
Estudiemos algunos ejemplos que se ajustan a los anteriores modelos de ecuaciones o se pueden
mediante manipulaciones algebraicas llegar a alguno de los modelos.

I Ejemplo 1.5.4. Busquemos las soluciones de la ecuacin 4 sen2 x = 2.


No se ajusta directamente a los modelos vistos, pero despejando sen x, obtenemos sen x = 12 ,

2
situacin conocida. De considerar sen x = 2
, obtenemos

26
1. Trigonometra 1.5. Identidades y ecuaciones trigonomtricas



2 x = + 2k
4
sen x = 3
2 x= 4
+ 2k.


2
De considerar sen x = 2
,


2 x = + 2k
4
sen x =
2 x = 5
4
+ 2k,

para k Z. /

No siempre nos va a resultar tan inmediato despejar una de las razones trigonomtricas como en el
caso precedente. Veremos situaciones donde vayamos complicando un poco las cosas.

I Ejemplo 1.5.5. Buscaremos las soluciones de 8 cos2 x 6 cos x + 1 = 0.


Esta situacin es muy habitual, aunque no podamos despejar cos x con la misma rapidez, plantea
una ventaja, y es que en toda la ecuacin aparece la misma razn trigonomtrica cos x.
Hacemos el cambio Y = cos x, con lo que la ecuacin original se transforma en

8Y 2 6Y + 1 = 0
1
ecuacin de segundo grado con soluciones Y1 = 2
e Y2 = 14 .
Deshacemos el cambio realizado,

1 x = + 2k
3
cos x =
2 x = 3 + 2k.

De la otra solucin,

1 x = arc cos 1 + 2k 1,13 + 2k
4
cos x =
4 x = arc cos 14 + 2k 1,13 + 2k.

En los ltimos clculos realizados, al no ser una razn trigonomtrica reconocida, hemos tenido que
dar una aproximacin con la calculadora. /

El procedimiento anterior, cambio de variable, es el que seguiremos para resolver las ecuaciones
que se nos presentan. En definitiva con este mtodo lo que hacemos es transformar las ecuaciones
trigonomtricas en otras que nos resultan ms familiares, generalmente polinmicas y racionales.
Aunque no siempre el cambio va a ser tan inmediato.

27
1.5. Identidades y ecuaciones trigonomtricas 1. Trigonometra


I Ejemplo 1.5.6. Buscaremos las soluciones de 3 sen x2 + cos x 1 = 0.
En esta ecuacin hay dos dificultades, aparecen dos razones trigonomtricas diferentes y dos ngulos
diferentes x2 y x. No es posible, de momento, hacer un cambio. Transformaremos la ecuacin en una
nueva de manera que en esta aparezca siempre el mismo ngulo y la misma razn.

Utilizamos que sen x2 = 1cos 2
x
, la ecuacin se transforma en

1 cos x

3 + cos x 1 = 0
2
Despejamos la parte radical y elevamos al cuadrado:

1 cos x

3 = 1 cos x
2
1 cos x
3 = 1 2 cos x + cos2 x
2
2 cos2 x cos x 1 = 0

 Con la anterior transformacin hay que ser cautos, pues no es una transformacin que nos
d una ecuacin equivalente. Lo que tenemos garantizado es que entre las soluciones de la
nueva ecuacin se encuentran las soluciones de la ecuacin inicial. Es por lo que al final del proceso
es obligatorio comprobar las soluciones obtenidas en la primera de las ecuaciones para desechar los
valores que no nos solucionen la ecuacin.

Ya tenemos la ecuacin en funcin de la misma razn, hacemos el cambio Y = cos x, y se transforma


la ecuacin en 2Y 2 Y 1 = 0, con soluciones Y = 1 e Y = 1
2
.
Deshaciendo el cambio, obtenemos:

Y = 1 cos x = 1 x = 360 k

1 1 x = 120 + 360 k
Y = cos x =
2 2 x = 240 + 360 k,

con k Z. /

I Ejemplo 1.5.7. Busquemos las soluciones de cos 2x = sen x + 3 .
En esta ecuacin aparecen las mismas dificultades que en la ecuacin anterior, aparecen dos razones
trigonomtricas diferentes y los ngulos que aparece con cada una de las razones trigonomtricas son
diferentes 2x y x + 3 .
Pero la estructura de la ecuacin nos
permite
una manipulacin
que nos facilita la resolucin. Basta

recordar las igualdades cos = sen 2 y sen = cos 2 .
Podemos poner la ecuacin de la forma:


sen 2x = sen x +
2 3

28
1. Trigonometra 1.5. Identidades y ecuaciones trigonomtricas

Ahora utilizaremos que si



= + 2k

sen = sen
= + 2k,

con k Z.
Tenemos pues las soluciones:

2k
2x = x + + 2k x =
2 3 18 3

2x = x + 2k x = + 2k.
2 3 6
/

La anterior estrategia se puede seguir igualmente con ecuaciones de la forma:

cos = cos

tan = tan

I Ejemplo 1.5.8. Buscaremos las soluciones de sen 2x cos x = 6 cos3 x.


Transformaremos la ecuacin de manera que obtengamos otra ecuacin equivalente en la que todo
aparezca en funcin de la misma razn y el mismo ngulo.

2 cos x sen x cos x = 6 cos3 x


2 cos2 x sen x 6 cos3 x = 0
2 cos2 x(sen x 3 cos x) = 0

cos x =0

sen x 3 cos x = 0.

De la primera de las ecuaciones sacamos las soluciones x = 2 + k.


La ecuacin sen x 3 cos x = 0, no es tan inmediata, pero es posible hacer una sencilla transformacin
que nos simplifica la ecuacin. Dividimos por cos x y nos queda
sen x cos x
3 =0
cos x cos x
o lo que es lo mismo tan x 3 = 0, con lo que otras soluciones salen de x = arctan 3 + k. /

29
1.5. Identidades y ecuaciones trigonomtricas 1. Trigonometra

1.5.3. El cambio tan 2 = t


Estudiaremos una transformacin que nos permite simplificar la mayora de las ecuaciones trigonom-
tricas transformando estas
q
en ecuaciones racionales.
Sabemos que tan 2 = 1+cos si hacemos tan 2 = t, tenemos
1cos

1 cos
t2 =
1 + cos
Despejando cos se obtiene
1 t2
cos =
1 + t2
Podemos buscar una expresin para sen en trminos de t, utilizando que sen2 + cos2 = 1 y
obtenemos
2t
sen =
1 + t2
Lo cual quiere decir una ecuacin en la que aparezca sen y cos se puede transformar en otra en la
2t 1t2
que aparezca 1+t 2 y 1+t2 , la ecuacin obtenida no siempre es de resolucin sencilla.

30
Ejercicios
Ejercicio 1. Conocemos de un ngulo que cos = 0,3. Representa y estima los valores
posibles de sen y tan .
Ejercicio 2. Si es un ngulo agudo:

a) Por qu es sen < 1? d) Cundo es sen < cos ?


b) Cundo es sen = cos ? e) Cundo es tan > 1?
c) Por qu es sen < tan ?

Ejercicio 3. Qu condiciones deben cumplir el seno y el coseno de un ngulo para que la


tangente sea positiva y mayor que 1? En qu cuadrante puede hallarse dicho ngulo?
Ejercicio 4. El ngulo est en el primer cuadrante y adems tan = n. Calcula las dems
razones trigonomtricas. Qu es lo que ocurre con las razones trigonomtricas cuando n ?
Ejercicio 5. Halla razonadamente las razones trigonomtricas del ngulo de 2655 .
Ejercicio 6. Calcula el seno, coseno y tangente de un ngulo del tercer cuadrante del que se
sabe que su cosecante es igual a 2.
Ejercicio 7. Si cos = 13 , siendo 0 < < 2 , calcula sen( ).
Ejercicio 8. En qu cuadrantes puede suceder que el seno de un ngulo sea igual a su coseno?
1 1
Ejercicio 9. Es posible encontrar un ngulo x que cumpla sen2 x > 2
y cos2 x > 2
a la vez?
Ejercicio 10. Calcula las razones trigonomtricas en cada caso, sin usar la calculadora:

a) cos x = 5/6, x III. c) tan x = 2, x III. e) tan x = 2, x IV.


b) sen x = 3/5 , x IV. d) sen x = 8/17, x II. f) sen x = 4/5, x III.

Con nmeros romanos se indica el cuadrante en el que se encuentra el ngulo x.


Ejercicio 11. Si sen() = 34 , halla cos( + 6), para < < 3
2
.
Ejercicio 12. Expresa en trminos de sen x, cos x y tan x las razones:


a) sen x + 2
b) cos x + 2
c) tan x + 2

Ejercicio 13. Halla razonadamente el valor de la expresin:

sen( 2 + x) + cos( x) + sen( x)


cos(x) + sen(x)

Ejercicio 14. Sin usar la calculadora, determina:


Ejercicios 1. Trigonometra


a) arc sen(0,5) c) arc sen( 3) e) arc cos(0,5) g) arc tg(1)

b) arc sen(1) d) arc cos(0,5) f) arc cos(2) h) arc tg( 3).

Ejercicio 15. Un faro mide 36 m sobre el nivel del mar. El ngulo de depresin de una
embarcacin es 30 . A qu distancia est el barco del faro?
Ejercicio 16. Calcula la longitud de la sombra de la torre Eiffel (altura 300 m), cuando la
inclinacin de los rayos solares es de 15 .
Ejercicio 17. Halla la altura de un edificio sabiendo que desde dos puntos alineados con la base
y distantes entre s 80 m, se ve bajo ngulos de 60 y 45 respectivamente.
Ejercicio 18. Halla la longitud de la sombra proyectada por un edificio de 200 m de altura
cuando la inclinacin de los rayos solares es de 30 .
Ejercicio 19. Determina la altura y la base de un rectngulo tal que su diagonal es 38 m y esta
forma con la base un ngulo de 30 .
Ejercicio 20. Halla la altura de una torre, sabiendo que se ve bajo un ngulo de 30 y si nos
acercamos 10 m, bajo un ngulo de 60 .
Ejercicio 21. Un topgrafo quiere medir la anchura de un ro. Coloca su teodolito en un punto
A de una de sus orillas y mide la distancia a otro punto B de la misma orilla. Esa distancia es 255
m. Desde A mide con el teodolito el ngulo que forma el punto C, que se encuentra a la otra orilla,
perpendicularmente a B, con el segmento AB, y resulta ser 45 . Cul es la anchura del ro?
Ejercicio 22. Calcula la longitud de los lados de un tringulo issceles, sabiendo que la altura
sobre el lado desigual es 10 m y el ngulo desigual es 120 .
Ejercicio 23. Las diagonales de un rombo miden 30 m y 10 m Cunto miden los lados ? Y
los ngulos?
Ejercicio 24. Cunto miden las diagonales de un rombo de 15 m de lado, si uno de sus ngulos
mide 60 ?
Ejercicio 25. En el punto ms alto de una pequea elevacin de terreno hay un poste de 3 m
de altura. Desde un punto A situado en el terreno llano se ve el pie B del poste bajo un ngulo de 30
y el extremo superior C bajo un ngulo de 45 . Halla la altura del montculo.
Ejercicio 26. En el dibujo la circunferencia tiene centro en O y radio 2 cm, los polgonos que
aparecen son regulares. Determina la distancia de O a P .

Ejercicio 27. Calcula el rea de un pentgono regular de 5 cm de lado (sen 36 0,588).

32
1. Trigonometra Ejercicios

Ejercicio 28. En una circunferencia de radio 6 cm inscribimos y circunscribimos sendos hex-


gonos regulares. Calcula el rea de la superficie comprendida entre ellos.
Ejercicio 29. En un tringulo se conocen a = 12, B = 40 y C = 75 , calcula:
a) La medida de la altura que cae sobre a.
b) La medida de la mediana que cae sobre a.
c) La medida de los lados b y c.
d) El rea del tringulo.
Ejercicio 30. Resuelve los siguientes tringulos, donde con a, b y c se representas los lados y
con A, B y C los ngulos opuestos a cada uno de los lados:

a b c A B C
1 37 24 61
2 57 100 57
3 57 100 57
4 57 100 57
5 57 57 62
6 57 57 62
7 4,7 41 59
8 321 470 123

Ejercicio 31. Resuelve el tringulo de lados a = 36 m, b = 26 m y c = 24 m. Calcula su rea.


Ejercicio 32. En un tringulo ABC se conoce a = 37, b = 42 y c = 68. Calcula A, B, C y el
rea del tringulo.
Ejercicio 33. Resuelve el tringulo en el que se conoce: A + B = 60 , a = 7 y b = 5.
Ejercicio 34. En un rombo de lado 4 el rea es 10. Cules son los ngulos del rombo?
Ejercicio 35. En un tringulo ABC, b = 42 cm, c = 25 cm y B + C = 94 . Calcula los ngulos
B y C. El lado a y el rea del tringulo.
Ejercicio 36. Los lados de un tringulo issceles miden 30, 45 y 45 cm. Calcula la medida de
los ngulos.
Ejercicio 37. En un tringulo ABC conocemos A = 94 y B = 36 , adems a + b = 30. Calcula
los lados a, b, c y el ngulo C.
Ejercicio 38. Calcula el ngulo que forman las tangentes a una circunferencia de 5 cm de radio,
trazadas desde un punto situado a 7 cm del centro.
Ejercicio 39. La resultante de dos fuerzas de 20 N y de 30 N es de 40 N. Qu ngulo forman
entre s dichas fuerzas? Qu ngulo forma cada una de ellas con la resultante?

33
Ejercicios 1. Trigonometra

Ejercicio 40. Halla los lados de un paralelogramo cuyas diagonales miden 20 cm y 15 cm


respectivamente y forman un ngulo de 42 .
Ejercicio 41. Dos de los lados de un paralelogramo miden 6 cm y 8 cm, y forman un ngulo de
32 . Cunto miden las diagonales?
Ejercicio 42. De un tringulo rectngulo se sabe que uno de sus ngulos mide 15 y que el rea
es 50. Calcula los lados del tringulo.

Ejercicio 43. En el dibujo el radio de la circunferencia es r = 2 y CD = 3, se pide:

A D

3

C O B

a) Determina sen A b) Determina la longitud AC c) Determina la longitud AD.

Ejercicio 44. En la pirmide de Keops, de base cuadrada, el lado de la base mide 230 m y el
ngulo que forma una cara con la base es de 52 . Calcula:

a) La altura de la pirmide. d) El ngulo que forma la arista con la base del


b) La altura de una cara. tringulo.
c) La longitud de una arista. e) El ngulo superior de cada cara.
f) El volumen de la pirmide.

Ejercicio 45. Dos circunferencias secantes tienen radios de 10 cm y 13 cm. Sus tangentes
comunes forman un ngulo de 30 . Calcula la distancia entre sus centros.
Ejercicio 46. Las longitudes de las diagonales de un paralelogramo son 84 y 52 cm. Si las
diagonales se cortan en un ngulo de 40 , calcula la longitud de los lados del paralelogramo.
Ejercicio 47. Las longitudes de los lados de un paralelogramo son 50 y 80 cm. Una de las
diagonales tiene 90 cm de longitud. Cul es la longitud de la otra diagonal?

Ejercicio 48. Resuelve el tringulo de datos b = 2, c = 3 + 1 y A = 45 .
Ejercicio 49. Resuelve el tringulo ABC y calcula su rea, conocidos A = 30 , B = 45 y
b = 2.
Ejercicio 50. Resuelve el tringulo ABC donde a = 5, A = 30 y b = 12.
Ejercicio 51. Los lados de un tringulo miden 13, 14 y 15 m. Calcula los ngulos y el rea del
tringulo.
Ejercicio 52. En un tringulo, el ngulo A = 45 , los lados b = 20 m y c = 18 m. Resuelve el
tringulo.

34
1. Trigonometra Ejercicios

Ejercicio 53. El cubo tiene arista igual a 4 cm. El punto P es el punto medio de la diagonal y
el Q est en la diagonal interior formando el tringulo rectngulo P QR.
P
R

Determina la distancia P Q.
Ejercicio 54. El cubo tiene arista igual a 4 cm. El punto Q est en la diagonal interior
formando el tringulo rectngulo P QR.
P

Resuelve el tringulo P QR.


Ejercicio 55. En un paralelogramo ABCD, el lado AB mide 6 m y AD mide 8 m, el ngulo
A = 30 . Halla la longitud de las diagonales.
Ejercicio 56. Las diagonales de un paralelogramo miden 10 y 12 m, y el ngulo que forman es
de 45 . Calcula la longitud de los lados.
Ejercicio 57. En la figura, los tringulos ABC y AEC son rectngulos e iguales. Siendo BC = 3,
CA = 7 y ED = EC. Calcula sin calculadora el seno del ngulo E y el rea del tringulo CED.

7
E

B 3 C D

Ejercicio 58. Dos caminantes andan a razn de 5 km/h y parten al mismo tiempo de un cruce
de dos caminos rectos que forman entre s un ngulo de 30 . Los dos van en el mismo sentido. A
qu distancia mutua se encuentran despus de haber caminado dos horas?
Ejercicio 59. Calcula los lados de un tringulo conocido su rea de 18 m2 y sus ngulos A = 30
y B = 45 .

35
Ejercicios 1. Trigonometra

Ejercicio 60. Uno de los lados de un tringulo es doble del otro y el ngulo comprendido vale

60 . Halla los otros dos ngulos.
Ejercicio 61. Al este y al sur de un globo, se observa al globo con ngulos de 45 y 60
respectivamente, si entre ambos puntos de observacin hay un kilmetro, calcula la altura a la que
est el globo.
Ejercicio 62. Desde un faro de 60 metros de altura, el ngulo de depresin con el que se ve un
barco situado al Sur es de 30 . Calcula la velocidad del barco, si despus de 15 minutos de moverse en
direccin oeste, el ngulo es de 15 .
Ejercicio 63. Dos bandas de anchura 1 cm se cruzan bajo un ngulo como se muestra en la
figura.

a) Cul es el rea de la regin comn a ambas?


b) Determina la longitud de las diagonales de la regin en funcin de .

Ejercicio 64. Un barco sale de un puerto navegando a 12 nudos. A una milla del puerto en
direccin norte se encuentra un faro. Si el barco navega direccin NO, halla la distancia del barco al
faro al cabo de dos horas y media. (1 nudo =1 milla/hora).
Ejercicio 65. Dos observadores distantes 5 km entre s, divisan un avin, dirigiendo sus pris-
mticos con ngulos de elevacin respectivos de 80 y 65 , a qu altura se encuentra el avin, si
suponemos que est situado sobre la lnea que une a los dos observadores?
Ejercicio 66. La distancia entre dos puntos A y B no se puede medir directamente, pues entre
ellos hay obstculos. Se recurre a un punto C y se miden las distancias de A a C y de B a C que son
de 48 y 68 metros respectivamente. El ngulo ACB es de 80 . Con estos datos calcula la distancia
entre A y B por redondeo y tres cifras significativas.
Ejercicio 67. Un jugador de golf lanza una pelota desde la posicin de salida de un hoyo distante
350 metros y alcanza una distancia de 180 metros. Pero el golpe ha sido defectuoso y la direccin de
la pelota forma un ngulo de 20 respecto de la direccin hacia el hoyo. A qu distancia del hoyo ha
quedado la pelota?

36
1. Trigonometra Ejercicios

Ejercicio 68. Dos lados de una finca triangular miden 20 y 15 metros respectivamente. El
ngulo comprendido entre esos dos lados es de 70 . Si deseamos vallar la finca, cuntos metros de
valla necesitaremos?
Ejercicio 69. Dos barcos salen de un puerto a la misma hora con rumbos distintos, formando
un ngulo de 110 . Al cabo de 2 horas, el primer barco est a 34 km del punto inicial y el segundo
barco, a 52 km de dicho punto. En ese mismo instante, a qu distancia se encuentra un barco del
otro?
Ejercicio 70. Un vehculo sale en direccin oeste con una velocidad de 120 km/h. Al cabo de 15
minutos gira 82 hacia el sur con respecto a la lnea del oeste. A qu distancia se encontrar del
punto de partida al cabo de otros 15 minutos?
Ejercicio 71. Tenemos un esquema de la posicin de cuatro poblaciones en un mapa, en el
que figuran parte de los datos.

B 15 C

150
10 12
60
A D

Determina la distancia entre las poblaciones A y D.


Ejercicio 72. En el grfico = 47 , h = 3 cm y r = 1 cm, determina el valor de x.
x
r

Ejercicio 73. De un tringulo ABC se sabe que a = 18, A = 25 , b = 30 y su permetro es


menor que 70. Resuelve el tringulo.
Ejercicio 74. En el cuadriltero:
D

4
3,5
6
A C
2
5,5

37
Ejercicios 1. Trigonometra

a) Determina su rea.
b) Determina la longitud de la diagonal que falta.
Ejercicio 75. En el rectngulo ABCD de lados 8 y 12 cm, se trazan desde B y D perpendiculares
a la diagonal AC, cortando a esta en los puntos M y N . Determina la longitud del segmento M N .
Ejercicio 76. Expresa cos( 6 + x) en trminos de sen x y cos x.
Ejercicio 77. Expresa tan( 4 + x) en trminos de tan x.
Ejercicio 78. Utiliza el desarrollo de cos(x + y) para hallar el valor exacto de cos 105 .
Ejercicio 79. Halla las razones trigonomtricas de los ngulos de 105 , 15 , 75 , 315 y 157 300 ,
buscando relaciones con ngulos conocidos.
Ejercicio 80. La tangente de un ngulo del segundo cuadrante es 45 . Halla las razones
trigonomtricas de los ngulos 2 y 2 .
Ejercicio 81. Expresa mediante una fraccin simple las razones trigonomtricas:

a) sen 315 b) tan 225 c) tan 120 .

Ejercicio 82. Deduce una frmula que permita expresar la tan(x + y + z) en funcin de tan x,
tan y y tan z.
A partir de la frmula anterior demuestra que si x, y, z son los ngulos de un tringulo cualquiera,
entonces se cumple que tan x + tan y + tan z = tan x tan y tan z.
Ejercicio 83. Demuestra que si x, y y z son los ngulos de un tringulo, entonces tan(x + y) +
tan z = 0.
Ejercicio 84. Expresa cos(3a) en funcin de cos a.
Ejercicio 85. Obtn una frmula para cos 4a en trminos de cos a.
Ejercicio 86. Si tan a = 1,6 . Calcula tan(2a) y tan(3a).
Ejercicio 87. Si cos = 35 , siendo 0 < < 2 , calcula cos(2) y sen(2).
Ejercicio 88. Si tan = 15 , siendo 0 < < 2 , calcula tan(2) y tan(4).
Ejercicio 89. Si tan = 13 , siendo 0 < < 2 , calcula tan(2) y sen(2).
Ejercicio 90. Demuestra las identidades:

a) sen( + ) sen( ) = sen2 sen2 e) cos4 sen4 = 2 cos2 1


b) sen2 cos2 = sen2 cos2 f) sen4 cos4 = sen2 cos2
c) (cos + sen )2 = sen(2) + 1 1 cos(2)
g) sen2 =
d) (cos + sen )2 + (sen cos )2 = 2 2

cos(x+y)+cos(xy)
Ejercicio 91. Demuestra la frmula cos x cos y = 2
.
Ejercicio 92. Calcula sen(a + b + c) en funcin de las razones trigonomtricas de a, b y c.

38
1. Trigonometra Ejercicios

Ejercicio 93. Demuestra las identidades:


a) sen( + ) sen( ) = sen2 sen2
b) sen( + ) sen( ) = cos2 cos2
Ejercicio 94. Demuestra la identidad: 1 + sen 2 = (sen + cos )2 .
Ejercicio 95. Demuestra que cos4 sen4 = cos(2).
Ejercicio 96. Simplifica las expresiones:

sen(2a) 1 + cos a
sen(a + ) tan a +
a) c) 2
1 cos2 a cos a cot( a)
sen(2a) sen a 2
b) sen a2 cos a2 (1 + sen a)
1 cos2 a cos a d)
sen(2a)

Ejercicio 97. Demuestra las identidades:

a) sen(a + b) sen(a b) = cos2 b cos2 a cos(a b) cos(a + b)


d) = tan b
1 2 sen(a + b) + sen(a b)
b) tan x + =
tan x sen(2x) 2 sen x sen2 x
x e) = cos x
c) 1 + cos x = 2 cos2 tan(2x) cos x
2


f) tan + a tan a = 2 tan(2a)
4 4

Ejercicio 98. Halla todos los nmeros reales, x, tales que cos x = cos 20 .
Ejercicio 99. Resuelve las ecuaciones en el intervalo que se indica.

a) cos(2) = sen en 0 360 . e) cos(2) 2 sen2 = 1 en 0 360 .


b) 3 tan = tan(2) en 0 360 . f) tan1 tan = 0 en 0 360 .
c) 3 cos(2) + sen = 1 en 0 360 . g) cos3 x + sen3 x = 0 en 0 x 2.
d) 4 sen2 3 = 0 en 0 2.

Ejercicio 100. Resuelve las ecuaciones trigonomtricas:


1
a) cos(15x) = sen(5x) c) 8 sen x 7 cos x = 0 e) sen x + cos x = 2
b) sen(2x) = cos(x 3 ) d) sen2 (3x) 5 sen(3x) + 4 = 0

Ejercicio 101. Resuelve las siguientes ecuaciones trigonomtricas:

a) sen4 x 2 cos4 x + 1 = 0 d) tan(2x) = tan x


b) 4 sen x2 + 2 cos x = 3 e) sen x + cos x = cos x(sen x + cos x)
2
f) (cos2 x sen2 x) = sen(2x)


c) 4 sen x 6
cos x 6
= 3

39
Ejercicios 1. Trigonometra

Ejercicio 102. Demuestra las identidades:




+

+

a) sen + sen = 2 sen cos c) cos + cos = 2 cos cos
2 2 2 2


+

+

b) sen sen = 2 cos sen d) cos cos = 2 sen sen
2 2 2 2

Ejercicio 103. Simplifica las expresiones:


sen(2a) + sen(4a) sen x + sen(3x) + sen(5x) + sen(7x)
a) c)
cos(2a) cos(4a) cos x + cos(3x) + cos(5x) + cos(7x)
sen a + sen b cos a cos b
b)
sen a sen b cos a + cos b

Ejercicio 104. Resuelve la ecuacin:

cos(2x) cos(6x) = sen(5x) + sen(3x)

Ejercicio 105. Resuelve, paso a paso, razonadamente y sin calculadora:




1 3
sen x = sen x +
3 2 2 2
Expresa todas las soluciones de la ecuacin.
Ejercicio 106. Calcula cos(3x) en funcin de cos x.
Ejercicio 107. Cunto vale cos x2 , si sen x = 1, 25?
2
Ejercicio 108. Puede haber un ngulo tal que el seno sea 3
y el coseno 35 ?
Ejercicio 109. Si , y son los tres ngulos de un tringulo, es cierto qu sen(+) = sen ?
Ejercicio 110. Sean A, B y C los ngulos de un tringulo rectngulo, donde A es el ngulo
recto. Demuestra que:

a) sen2 B + sen2 C = 1 b) tan B tan C = 1

Ejercicio 111. Demuestra que sen cos( ) + cos sen( ) = sen .


Ejercicio 112. Entre qu valores oscila el seno y el coseno de un ngulo?
Ejercicio 113. Los ngulos A, B y C, de un tringulo, cumplen la relacin:

sen B + sen C = cos B + cos C.

Demuestra que el tringulo es rectngulo.


Ejercicio 114. Siendo A, B y C los ngulos de un tringulo, demuestra que:

40
1. Trigonometra Ejercicios

a) sen A = sen(B + C) b) cos A = cos(B + C)

Ejercicio 115. Simplifica:


sen( + ) sen( ) sen(2) sen(2) sen(2)
a) b) c)
cos + cos 1 + cos(2) 1 cos2 cos

sen(3) + sen
Ejercicio 116. Simplifica la expresin:
sen cos(3)
Ejercicio 117. La forma de empaquetar cuatro tringulos equilteros, de manera que el envol-
torio sea el cuadrado mnimo, es la que se presenta en la figura.
Suponiendo que en los tringulos equilateros el lado mide 1. Expresa la medida del rea del cuadrado
envoltorio y del cuadrado central.

Ejercicio 118. En el dibujo los polgonos son regulares el lado del cuadrado mide 1 dm. Deter-
minar el rea y el permetro de la figura sombreada.

Ejercicio 119. Del tringulo ABC sabemos: b = 52 cm, c = 49 cm y B C = 12 . Calcular A,


B, C, a, el rea y el permetro.
Ejercicio 120. Se quiere disear un adorno bordado para una camiseta siguiendo el esquema y
las condiciones siguientes:
1. Las puntadas se realizarn en zigzag entre dos rectas que forman un ngulo .
2. La primera puntada empezar en el punto O, comn a las dos rectas, y acabar en una de las
rectas (que llamaremos horizontal).
3. Todas las dems puntadas debern tener la misma longitud y se trazarn sin superponerse ni
volver hacia atrs.
4. La ltima puntada debe ser perpendicular a la lnea horizontal.
5. Queremos dar exactamente 20 puntadas.

41
Ejercicios 1. Trigonometra

O A

a) Cul debe ser el ngulo para que se cumplan esas condiciones?


b) Si la distancia entre O y el punto de la horizontal por donde pasa la ltima puntada fuera de 25
cm. Cul sera la longitud de cada puntada?
c) Qu ocurrira si quisiramos hacer 21 puntadas en vez de 20 con las mismas condiciones, esto es,
que la nmero 21 fuera perpendicular a la horizontal?
Ejercicio 121. Determina todas las medidas del ngulo (0 360 ), para los que se
verifique:
log2 (3 sen ) = 2 log2 (cos ) + 1
Ejercicio 122. Los centros de las circunferencias estn alineados. Las circunferencias tienen
radio r. La recta OC es tangente a la circunferencia de centro C3 y corta a la circunferencia del
medio en los puntos A y B. Determina la longitud de AB en trminos de r.
C
B
A

O
C1 C2 C3

Ejercicio 123. Tenemos una hoja rectangular de dimensiones a b, siendo a > b. Doblamos el
rectngulo por su diagonal. Determina el rea del tringulo ABC.
B

Ejercicio 124. En el dibujo el tringulo ABC es 30 ; 60 ; 90 , AB = 20, D es el punto medio


del segmento AC, ED es perpendicular a AC, EC es paralelo a AB, F est en la extensin de AB y
EF es perpendicular a ED. Determina:
a) Longitud de ED b) Longitud de DF

E C

F A B

42
1. Trigonometra Ejercicios

Ejercicio 125. ABC es un tringulo equiltero; BCDE es un cuadrado de lado 2 cm construido


exteriormente al tringulo. Los vrtices A, D y E pertenecen a la misma circunferencia. Halla el
valor del radio de la circunferencia.
E

A B

Ejercicio 126. En el dibujo el sector y el rectngulo tienen el mismo rea. Si la altura del
rectngulo es R, cul es la base del rectngulo?

1 rad
R

Ejercicio 127. Resuelve la ecuacin:


 2

x
4 16sen = 26 sen x
para 0 x 2.
Ejercicio 128. Los siete tringulos coloreados son issceles. Tambin lo es el tringulo formado
con todos ellos. Determina el rea del tringulo.

2 cm

Ejercicio 129. El lado del cuadrado mide 4 cm. El punto E es el punto medio del lado CD.

A B

C D
E

43
Ejercicios 1. Trigonometra

Determina el rea del tringulo sombreado.


Ejercicio 130. Siendo y nmeros reales del intervalo (0, 2 ) tal que

cos2 ( ) = sen 2 sen 2

Hallar + .
Ejercicio 131. En el cuadriltero dibujado sabemos que A = 52 y C = 78 . Determina el rea
del cuadriltero:

5cm

9cm C

7cm

A D

Ejercicio 132. Un tringulo equiltero de papel se dobla como muestra el grfico, el vrtice C
se posiciona en el lado opuesto en un punto C 0 de manera que AC 0 = 1 y BC 0 = 2. Determina la
longitud de P Q.

P
Q

A C0 B

Ejercicio 133. Un tringulo ABC tiene un ngulo recto en B, si P es un punto del interior
del tringulo de manera que P A = 10, P B = 6, y AP B = BP C = CP A, cul es la longitud
P C?

44
UNIDAD 2

Geometra afn en el plano

Trabajaremos sobre un conjunto que se llama plano y cuyos elementos se llaman puntos. Los puntos
son los elementos bsicos sobre los que construimos nuestra geometra. Los puntos se suelen denotar
con letras maysculas A, B, C. . . Al conjunto de puntos en el plano los designaremos por 42 .
Tenemos algo de experiencia en cuanto al trabajo en dicho conjunto, por ejemplo reconocemos unos
subconjuntos (o partes del plano) que llamamos segmentos y otros que llamamos rectas. Sabemos de
ciertas relaciones entre los puntos y las rectas:
En una recta hay puntos que pertenecen a ella y otros que no.
Si tenemos dos puntos distintos hay una nica recta que contiene a esos puntos.
Hay rectas que no tienen ningn punto en comn y otras que comparten un punto.
Un punto en una recta divide a esta en dos semirrectas. Una recta divide al plano en dos
semiplanos.
Dos semirrectas con origen comn determinan un ngulo. Todo ngulo tiene una medida.
Todos los segmentos tienen dos extremos y una longitud mayor que 0. Cuando hablamos de
longitud indirectamente estamos tomando un patrn de medida, unidad de longitud.

2.1. Vectores en el plano


Algunas magnitudes fsicas como la masa, el volumen, la temperatura, etctera, pueden ser cuantificadas
mediante un nmero, pero hay otras magnitudes en las que es necesario algo ms. Magnitudes fsicas,
por ejemplo la velocidad, la aceleracin, la fuerza, el campo elctrico, etc., a parte de una intensidad
tienen asociada una direccin, necesitamos algo ms que un nmero para poder caracterizarlas. Son
lo que se llaman magnitudes vectoriales. Utilizaremos un objeto matemtico que llamaremos vector,
que nos servir para representar dichas magnitudes.
Definicin 2.1.1 (Vector fijo). Llamaremos vector fijo a un segmento orientado, es decir con un
#
punto origen A y un punto final B. Lo denotaremos por AB.
#
Cuando tengamos un vector fijo en el que coincida el origen con el final, AA, diremos que es el vector
2.1. Vectores en el plano 2. Geometra afn en el plano

fijo nulo.
Diremos que dos vectores fijos son equivalentes, si se da alguna de las siguientes situaciones:

C D A B C D
C
D
A
B
A B E F
1 2 3

1. Son nulos.
# #
2. Los vectores AB y CD son tal que ACDB es un paralelogramo.
#
3. Existe otro vector EF de forma que AEF B y CEF D son paralelogramos.

 Es significativo el orden en que se toman los puntos, para comprobar si se forman un para-
# # # #
lelogramo: Origen de AB Origen de CD Final de CD Final de CD.

D C

A B

Los vectores no nulos equivalentes tiene caractersticas comunes por el hecho de ser lados de paralelo-
gramos.
Son paralelos, a esta caracterstica la llamaremos direccin.
#
Miden lo mismo, diremos que tienen el mismo mdulo. La medida del vector AB, se indica
#
utilizando la notacin |AB|.
Teniendo vectores con la misma direccin y mdulo hay una caracterstica que les puede
diferenciar, es el orden en que se tome su origen y final, a ello lo llamaremos sentido. Observamos
que dada una direccin (dos puntos) hay dos posibles sentidos.
Definicin 2.1.2 (Vector libre). Se llama vector libre o simplemente vector, a cada conjunto de
vectores equivalentes.

D
#u
B F
C #u #u

A E

46
2. Geometra afn en el plano 2.1. Vectores en el plano

# # #
Aunque como vectores fijos CD, AB y EF son diferentes, representan el mismo vector libre, al que
hemos denotado por # u.
Los vectores fijos nulos forman un conjunto que llamaremos vector nulo.
Un vector libre est caracterizado por su mdulo, direccin y sentido y no por su posicin en el plano.
El vector nulo es una excepcin, pues no podemos decir que defina una direccin en el plano, con lo
cual sobra hablar de sentido del vector nulo, aunque que tiene lgica hablar de mdulo del vector
#
nulo que indicaremos con | 0 | = 0.

 En lo que sigue, y aunque no lo comentemos, siempre que hablemos de vectores hemos de tener
en cuenta la singularidad que supone el vector nulo.
#
Para representar los vectores libres utilizaremos letras minsculas #
a , b , #
c , . . . ; para representar el
#
vector nulo usaremos 0 .
Al conjunto de los vectores libres en el plano lo representaremos por V2 .
Nota 2.1.1. Algunas consideraciones:
# #
1. Si de un vector sabemos que AK = 0 podemos asegurar que K = A.
# #
2. Si de dos vectores sabemos que AL = AM podemos asegurar que L = M .
#
3. Para todo #
u y todo punto A, hay un nico punto B de manera que AB = #
u.

2.1.1. Operaciones con vectores


Por lo general los objetos matemticos nos descubren sus posibilidades cuando empezamos a operar
con ellos. El comportamiento de algunas de las magnitudes vectoriales, como por ejemplo las fuerzas,
nos da pistas de como hemos de definir las operaciones con vectores para que lo que definamos se
ajuste a lo que realmente ocurre.
Definicin 2.1.3 (Suma). Dados dos vectores # u , #
v V se define el vector #
2 u + #
v , como el vector
que se obtiene aplicando lo que se conoce con el nombre de la regla del paralelogramo.

D
C
#v
#u +
#
v #
v #
u

# B
u
A

En la definicin anterior es importante darse cuenta de la seleccin hecha de los representantes de #


u
y #
v , compartiendo el mismo origen, otra seleccin de diferentes representantes de #
u y #
v producen el
mismo resultados o lo que es lo mismo un vector con las mismas caractersticas que el obtenido.
Nota 2.1.2 (Configuracin de Chasles). Podemos obtener el mismo resultado poniendo los vectores
uno a continuacin del otro y considerar el vector que parte del origen del primero de los vectores

47
2.1. Vectores en el plano 2. Geometra afn en el plano

con el final del segundo de los vectores, tal forma de sumar vectores es equivalente a la regla del
paralelogramo.

#v #
#u + v

# B
u
A
Configuracin de Chasles. #
u + #
v.

Esta forma de sumar vectores es particularmente cmoda cuando tenemos que sumar ms de dos
vectores.
Nota 2.1.3. A partir de la definicin que hemos dado para la suma de vectores, podemos realizar
algunas sumas sin dificultad:
# # #
1. AB + BC = AC
# # # #
2. De forma ms general AB + BC + + Y Z = AZ
# # # # # #
3. AB + BA = AA = 0 . Al vector BA se le llama opuesto del vector AB.
# # #
4. AB + BB = AB.
Proposicin 2.1.1. La suma de vectores verifica las propiedades:
1. #
u + #
v = #v + #
u . Propiedad conmutativa.
2. u + ( v + w) = ( #
# # # u + # # Propiedad asociativa.
v ) + w.
# # #
3. El vector 0 , que verifica #
u + 0 = 0 + #
u = #
u . Existencia de elemento neutro para la suma.
#
4. Para cada vector u existe un vector, que representaremos por #
# u , verificando #
u + ( #
u) = 0.
Existencia de vector opuesto.

Demostracin: Para demostrar 1, 2 y 3 basta slo tener en cuenta la definicin de suma.


En 4 basta ver que si tenemos un vector #
u y se considera el vector con la misma direccin y mdulo,
pero sentido opuesto, la suma nos da el vector nulo (observar el punto (3) de la Nota 2.1.3).

Definicin 2.1.4 (Resta de dos vectores). Dados dos vectores #


u y #
v denotaremos con #
u #
v la
# #
operacin u + ( v ).

u# #
v
#
u
#
# u
#
v
v

Proposicin 2.1.2. Dados dos vectores #


u y #
v , se verifica:

48
2. Geometra afn en el plano 2.1. Vectores en el plano

# # #
1. u u = 0.
2. #
u #
v y #
v #
u son vectores opuestos.

Demostracin: Las dos propiedades son consecuencia de la definicin de resta de dos vectores:
#
1. #
u #
u = #
u + ( #
u) = 0.
#
2. Basta sumar los dos vectores para ver que #
u + ( #
v ) + #
v + ( #
u) = 0.

#
Definicin 2.1.5 (Producto por un escalar). Dado el vector #u V2 , siendo #
u 6= 0 y k R se define
el vector k #
u como el vector con la misma direccin que #
u , cuyo mdulo es igual al mdulo de # u
#
multiplicado por |k|, con el mismo sentido que u si k > 0 y sentido opuesto, si k < 0.
#
Si k = 0, definimos 0 #
u = 0.
# #
Para el vector nulo definimos k 0 = 0 .

k<0
#
k>0 #
#u u k u
k

Producto de escalar por vector. k #


u

Proposicin 2.1.3. Tenemos , R y # u , #


v V2 . El producto de un nmero por un vector verifica
las propiedades :
1. () #u = ( #
u ). Propiedad asociativa.
# #
2. 1 u = u . Existencia del elemento neutro para el producto por un escalar.
3. ( + ) #u = #
u + #
u . Propiedad distributiva.
4. ( u + v ) = u + #
# # # v . Propiedad distributiva.

 Es importante fijarse que en las anteriores propiedades aparecen diferentes operaciones, por
una parte la suma, en dos versiones, de nmeros reales + y de vectores # u + #
v , que an
siendo operaciones diferentes las indicaremos con el mismo smbolo. De forma parecida con el producto,
que aparece entre nmeros reales, y el producto entre un nmero y un vector, #
u . Se indica las
operaciones de la misma manera, y al igual que en la suma no hay ambigedad para saber cuando se
trata de una o de otra.

Nota 2.1.4. De la definicin de producto de un vector por un nmero y del concepto de opuesto de
un vector podemos, deducir la igualdad

1 #
u = #
u

Definicin 2.1.6 (Plano afn). El conjunto V2 y el conjunto de puntos en el plano 42 se denomina


plano afn.

49
2.2. Coordenadas 2. Geometra afn en el plano

2.2. Coordenadas
Hasta ahora hemos descrito lo que son los vectores y como podemos operar con ellos, necesitamos dar
un paso para poder traducir la geometra que hemos visto en trminos de lgebra y nmeros.
A partir de un vector #
u podemos considerar otro vector de la forma k # u donde k es un nmero,
cuando tengamos dos vectores que se relacionen de esa forma expresaremos tal relacin de diferentes
formas:
En semejanza con la divisibilidad de los enteros diremos que k #
u es mltiplo de #
u.
# #
Haciendo consideraciones geomtricas, si consideramos los vectores k u y u compartiendo el
origen, observamos que el origen y los extremos de los vectores estn alineados, diremos que los
vectores son colineales.

Definicin 2.2.1 (Dependencia lineal). Diremos que dos vectores # u y #


v son linealmente dependientes
si uno de ellos se puede poner como mltiplo del otro. O dicho de otra forma existe k de manera que
o bien #
v = k #
u o #
u = k #
v.
Si por el contrario no ocurre lo anterior diremos que los vectores son linealmente independientes.

Nota 2.2.1 (Significado geomtrico). El concepto definido tiene una traduccin geomtrica inmediata,
los vectores linealmente dependientes tienen la misma direccin, basta recordar el efecto que produce
sobre un vector el multiplicar este por un nmero y por contra los linealmente independientes no
comparten direccin.
# #
Nota 2.2.2. Los vectores 0 y # u son linealmente dependientes, basta ver que 0 = 0 # u.

Definicin 2.2.2 (Combinacin lineal). Dados dos vectores #


u , #
v y dos nmeros a y b, diremos que
a #
u + b #
v es una combinacin lineal de #
u y #
v.

#v
b
#u +
a
#
v

#
u

Nota 2.2.3. En el grfico de ejemplo vemos que los valores que se usan son a = 3 y b = 2, con el fin
de ver con claridad lo que se hace, pero podemos considerar cualquier combinacin lineal eligiendo
cualquier tipo de nmeros.

Ejercicio 1. Indica que vectores se obtienen cuando consideramos las siguientes combinaciones
lineales de vectores:

a) 1 #
u +0 #
v c) 1 #
u + 1 #
v
#
b) 0 u + 1 #
v d) (1) u + 0 #
# v

50
2. Geometra afn en el plano 2.2. Coordenadas

e) 0 #
u + (1) #
v g) 12 #
u +0 #
v
f) (1) u + (1) #
# v #
h) 1 u + 12 #
v

Nota 2.2.4. Seguiremos los convenios habituales para la simplificacin de cierta notaciones, as por
ejemplo (1) #
u + (1) #
v se denotar de la forma #u #
v.
Proposicin 2.2.1. Dados dos vectores #u , # # siempre se
v linealmente independientes y un vector w,
# como combinacin lineal de #
puede poner w u y #
v , esto es, existen x, y R de manera que
# = x #
w u + y #
v

Demostracin: Tenemos dos vectores, #


u y # # siempre
v linealmente independientes, y otro vector w,
podemos realizar la secuencia:

y~v
~v
y
w~ w~ ~u +
x
~v

~v

~v

~u ~u ~u x~u

1. Consideramos representantes de los vectores compartiendo el mismo origen.


# hace de diagonal y los lados del paralelogramo
2. Construimos el paralelogramo, en el que el vector w
siguen la direcciones marcadas por los vectores # u y #
v.
3. Los lados del paralelogramo nos define los vectores x #u e y #
v , que verifican
# = x #
w u + y #
v.

Nota 2.2.5. Los valores x e y determinados antes son nicos, pues si hubiera otros dos valores y
verificando lo mismo, tendramos que
# = x #
w u + y #
v
w = u + #
# # v
Restando las dos igualdades tendramos:
# w
w # = x #
u + y #
v ( #
u + #
v)
#
0 = (x ) #
u + (y ) #
v
Despejando obtenemos (x ) #u = (y ) #
v . Si alguno de los valores x y y fuera distinto
de cero, podramos despejar y poner un vector como mltiplo del otro, cuestin que no se puede hacer
pues son linealmente independientes. Luego se ha de verificar que x = 0 y y = 0, con lo que
x = e y = .

51
2.2. Coordenadas 2. Geometra afn en el plano

2.2.1. Bases
Estamos en disposicin de definir un concepto importante, el de base.
Definicin 2.2.3 (Base). Llamaremos base de los vectores del plano a un conjunto de vectores lineal-
mente independientes y de manera que cualquier vector del plano se pueda poner como combinacin
lineal de ellos.
Nota 2.2.6. La anterior definicin no es muy concreta, tiene la ventaja que nos ser til cuando
estudiemos los vectores en el espacio. Con lo visto hasta el momento podemos concretar algo ms y
podemos decir que dos vectores del plano, B = { #
u , #
v }, con distinta direccin verifican las condiciones
que se imponen en la definicin (ver proposicin 2.2.1).
Definicin 2.2.4 (Coordenadas de un vector). Dada una base B = { #
u , # # se
v }, cualquier vector w
puede poner como combinacin lineal de la base.
# 7 x #
w u + y #
v 7 (x, y)
# con respecto a la base B.
Llamaremos a (x, y) coordenadas del vector w
Nota 2.2.7. Conviene observar los siguientes puntos:
Las coordenadas de un vector son nicas (Nota 2.2.5).
#
El vector nulo, 0 tiene coordenadas (0, 0) con respecto a cualquier base.
En la base B = { # u , #
v } las coordenadas de #
u y #
v son respectivamente: (1, 0) y (0, 1). Basta
observar que #u = 1 # u + 0 #
v y que #
v = 0 #
u + 1 #
v.
El orden en el que se dan los vectores de la base influye en el orden de las coordenadas de un
# = (x, y) con respecto a la base B = { #
vector. As si w u , #
v }, con respecto a la base B = { #
v , #
u}
#
tendr coordenadas w = (y, x).

Ejercicio 2. Consideramos la base B = { #


v , #
u },
~v

~u

Con respecto a B tenemos unos vectores con coordenadas:

a) (1, 0) d) (1, 1) g) (1/2, 2) j) (1, 2)


b) (0, 1) e) (2, 1) h) (3, 4) k) (1, 2)
c) (1, 1) f) (1, 2) i) (1, 3/2) l) (3, 0)

Representa los vectores anteriores.


Ejercicio 3. Consideramos las bases:

52
2. Geometra afn en el plano 2.2. Coordenadas

~t
~u ~s

~r ~r ~r
#
B = # B = { #
r , # B = { #
r , #

0 00
r, t u} s}

Representa respecto a cada una de las bases el vector de coordenadas (2, 1). Qu podemos afirmar?
Ejercicio 4. Sobre un exgono regular consideramos la base B = { # u , #
v}

#
w #
w
4 6

#w
7
3
w#

#
w 8
1
w#

2
w#

~v ~v 5
#w

~u ~u

# ,..., w
Determina las coordenadas de los vectores w # , con respecto a la base B.
1 8

2.2.1.1. Operaciones y coordenadas


#
Tenemos la base B = { #
u , #
v } y dos vectores #
a = (a1 , a2 ) y b = (b1 , b2 ).
#
Nos preguntamos si habr una buena relacin entre las coordenadas de los vectores #
a + b y #
a, y
#
las coordenadas de los vectores #
a y b.
#
Proposicin 2.2.2. Las coordenadas de #
a + b son
# #
a + b = (a1 + b1 , a2 + b2 ) (2.1)

Demostracin: A partir de las propiedades de las operaciones con vectores podemos ver
# #
a + b = a1 #
u 1 + a2 #u 2 + b1 #
u 1 + b2 #
u2 =
# #
= (a1 + b1 ) u 1 + (a2 + b2 ) u 2 =
= (a1 + b1 , a2 + b2 )

Proposicin 2.2.3. Sea R las coordenadas del vector #


a son

#
a = (a1 , a2 ) (2.2)

53
2.2. Coordenadas 2. Geometra afn en el plano

Demostracin: Basta observar

#
a = (a1 #u 1 + a2 #
u 2) =
= a1 u 1 + a2 #
# u2 =
= (a1 , a2 )

Nota 2.2.8. De lo anterior dos inmediatas conclusiones::


1. Como no poda ser de otra manera: #
a = (a1 , a2 )
2. Combinando las dos proposiciones anteriores podemos generalizar:
#
#
a + b = (a1 + b1 , a2 + b2 ).

En esta seccin hemos empezado, una vez fijada una base, asignando a los vectores del plano V2 , un
par de nmeros, sus coordenadas. Cuando hacemos este tipo de equivalencias estamos identificando
dos objetos matemticos diferentes, vectores y pares de nmeros. Lo que hemos hecho va ms all de
la identificacin de los objetos, pues vemos que se establece una equivalencia entre las operaciones
que podemos realizar con los vectores y con los pares de nmeros.

Nota 2.2.9 (Vectores con la misma direccin). Dos vectores no nulos # u y #


v tienen la misma direccin
# #
( u k v ), si uno de ellos es mltiplo del otro, o dicho de otra forma, si existe un nmero de manera
que #
u = #v.
Veamos como podemos traducir lo anterior considerando las coordenadas de los vectores. En trminos
de coordenadas, si # u = (u1 , u2 ) y #
v = (v1 , v2 ), ha de ocurrir que u1 = v1 y u2 = v2 para cierto
R. Despejando , obtenemos la condicin:
u1 u2
= (2.3)
v1 v2

I Ejemplo 2.2.1. Determinaremos x, para que los vectores #


u = (2, x) y #
v = (5, 8) tengan la misma
direccin. Repetimos el argumento anterior, sabemos que para ello
#
v = #
u (2, x) = (5, 8) (2, x) = (5, 8)
2 x
= =
5 8
16
Ya solo necesitamos despejar x, para obtener que x = 5
. /

2.2.2. Sistemas de referencia


A partir de la consideracin de una base hemos asignado a los vectores del plano con pares de nmeros.
Sin mucho esfuerzo vamos a hacer algo parecido con los puntos del plano. Para ellos necesitamos lo
que llamaremos sistema de referencia

54
2. Geometra afn en el plano 2.2. Coordenadas

Definicin 2.2.5. Llamaremos sistema de referencia del plano afn a una terna R = {O; #
u 1 , #
u 2}
donde:

~u2
O ~u1

1. O es un punto del plano y


2. { #
u 1 , #
u 2 } una base del conjunto de vectores.

2.2.2.1. Coordenadas de un punto

Fijada una referencia en el plano y dado un punto P ,

P
2
b~u
~u2

O ~u1 a~u1

# #
este nos define el vector OP que llamaremos vector de posicin del punto P , el vector OP tendr en
#
la base { #
u 1 , #
u 2 }, las coordenadas OP = (a, b).
Diremos que el punto P tiene coordenadas P (a, b).

Punto Vector de posicin Coordenadas


#
P OP = a #
u 1 + b #
u2 (a, b)

 Llegado a este punto tenemos dos objetos matemticos diferentes, pero que sin embargo tienen
algo en comn, las coordenadas. Para que no se produzcan equvocos se suele utilizar, como
#
criterio ms extendido, la notacin A(a, b) para los puntos (sin el =) y OP = (a, b) para los vectores
(con el =).

 Hay
#
#
que extremar la precaucin tener presente las diferencias entre P y OP . Mientras que
OP es un representante de un vector libre, con lo cual a efectos prcticos podramos considerar
cualquier otro representante en otra posicin del plano, P es un objeto inamovible.

Ejercicio 5. En el grfico, R = {O; #


u , #
v } es el sistema de referencia .

55
2.2. Coordenadas 2. Geometra afn en el plano

~v

O ~u

Determina las coordenadas de los puntos marcados.

2.2.2.2. Vector determinado por dos puntos

En esta y subsiguientes secciones, salvo que se indique lo contrario, se supone que tenemos un sistema
de referencia previamente fijado, con lo que nos referiremos a puntos y vectores directamente por sus
coordenadas.
#
Si tenemos dos puntos A(a1 , a2 ) y B(b1 , b2 ) determinan en el plano un vector AB, nos planteamos
determinar las coordenadas de dicho vector. Fijmonos en los elementos que tenemos:

# B
AB
A
~v

O ~u

# # # #
Sabemos que se verifica la igualdad vectorial OA + AB = OB con lo que despejando AB obtenemos
# # # # #
la igualdad vectorial AB = OB OA, donde OA y OB son los vectores de posicin de los puntos A
#
y B, podemos expresar AB en trminos de coordenadas:
# # #
AB = OB OA = (b1 a1 , b2 a2 ) (2.4)

 Con los vectores hemos definido operaciones, no as con los puntos. No tiene ningn sentido
para nosotros expresiones de tipo B A, aunque si tiene sentido expresiones de la forma
# #
OB OA.

Ejercicio 6. Un representante del vector libre w # = (1, 4) tiene su origen en el punto A(3, 2),
determina las coordenadas del final del vector.
# tiene su final en el punto B(5, 7), determina las coordenadas
Otro representante del mismo vector w
del origen del vector.

56
2. Geometra afn en el plano 2.2. Coordenadas

2.2.2.3. Punto medio de un segmento

Dados dos puntos A y B, con coordenadas A(a1 , a2 ) y B(b1 , b2 ) el punto medio (M ) del segmento
AB tiene coordenadas:

a1 + b1 a2 + b2

M , (2.5)
2 2

B
M
A

# #
Basta observar la igualdad vectorial 2AM = AB. Como no conocemos las coordenadas de M ,
supondremos que estas son M (x, y), con lo que
# #
2AM = AB
2(x a1 , y a2 ) = (b1 a1 , b2 a2 )

a1 + b 1 a2 + b 2

(x, y) = ,
2 2

El anterior resultado se puede generalizar para obtener las coordenadas de un punto T en un segmento
# #
AB de manera que k AT = AB, siendo k un nmero real cualquiera.
En este caso y de forma similar al caso anterior, si consideramos que A(a1 , a2 ), B(b1 , b2 ) y T (x, y),
obtenemos que
# #
k AT = AB
k(x a1 , y a2 ) = (b1 a1 , b2 a2 )

k(x a a1 (k1)+b1
= b 1 a1
1)
x = k
k(y a ) = b a
a2 (k1)+b2
2 2 2 y= k

De lo anterior ponemos expresar que:

a1 (k 1) + b1 a2 (k 1) + b2

T ,
k k

Ejercicio 7. Dados los puntos A(1, 2) y B(5, 7), determina las coordenadas de dos puntos T1 y
T2 , de forma que dividan al segmento AB en tres segmentos de igual tamao.
Trata de generalizar para el caso en que A(a1 , a2 ) y B(b1 , b2 ).
Ejercicio 8. Determina el punto simtrico del punto A(1, 2) con respecto al punto B(1, 6).
Ejercicio 9. Dado un cuadriltero ABCD demuestra que el cuadriltero formado uniendo los
puntos medios del ABCD es un paralelogramo.

57
2.2. Coordenadas 2. Geometra afn en el plano

B Q
C
P
R

A D
S

2.2.2.4. Puntos alineados


# #
Tres puntos A(a1 , a2 ), B(b1 , b2 ) y C(c1 , c2 ), estn alineados, cuando los vectores AB y AC tienen la
# #
misma direccin, o de otra forma, cuando los vectores AB y AC son linealmente dependientes (nota
2.2.1) .
En trminos de coordenadas (repasar nota 2.2.9):

(b1 a1 , b2 a2 ) = (c1 a1 , c2 a2 )

b 1 a1 b 2 a2
=
c 1 a1 c 2 a2

2.2.2.5. El baricentro

Sea ABC un tringulo, tres puntos en el plano no alineados. Consideremos el punto medio del
segmento BC, punto que denotaremos con MA . Al segmento AMA se le llama mediana. Hay en el
tringulo tres medianas una por cada vrtice.

MA

A B

# #
Consideramos el punto G, en la mediana AMA , verificando adems que AG = 2GM A . Veamos algunas
propiedades de G:
Determinemos en primer lugar sus coordenadas G(x, y), para ello supondremos que A(a1 , a2 ),
B(b1 , b2 ) y C(c1 , c2 ).
Como MA es el punto medio de B y C, sus coordenadas son MA ( b1 +c 2
1 b2 +c2
, 2 ). Empecemos a

58
2. Geometra afn en el plano 2.3. Ecuacin de la recta

operar:
# #
AG = 2GM A

b 1 + c1 b2 + c 2

(x a1 , y a2 ) = 2 x, y
2 2
(x a1 , y a2 ) = (b1 + c1 2x, b2 + c2 2y)

x a a1 +b1 +c1
1 = b1 + c1 2x =
x
3
y a = b + c 2y
a2 +b2 +c2
2 2 2 y= 3

El punto G tiene coordenadas ( a1 +b31 +c1 , a2 +b32 +c2 ).


El clculo anterior tiene consecuencias, si hiciramos lo mismo con los otros vrtices y determi-
nsemos las coordenadas del correspondiente G, obtendramos las mismas coordenadas para G,
ello quiere decir que las medianas se cortan en el mismo punto, el baricentro del tringulo. El
punto G divide a cada mediana en la razn 2 : 1.
C

MB MA

A MC B
# # # #
El punto G verifica GA + GB + GC = 0 .
Tiene propiedades fsicas importantes, si tuviramos un modelo del tringulo ABC, a efectos
de equilibrio podemos considerar su masa concentrada en G. En G se encuentra el centro de
masas del tringulo.
C

G
A
B

Ejercicio 10. Dado un tringulo ABC, siendo G su baricentro demuestra que


# # # #
GA + GB + GC = 0 .

2.3. Ecuacin de la recta


El concepto de recta, al igual que el de punto, lo hemos abordado a lo largo de los aos desde diferentes
perspectivas, aunque siempre con el mismo significado. Nos acercamos ahora al significado de recta
desde un punto vectorial. Lo primero que observamos, es que de forma independiente, una direccin
#
v y un punto P no nos determinan una nica recta:

59
2.3. Ecuacin de la recta 2. Geometra afn en el plano

~v
P

Lo cosa cambia cuando consideramos un punto P del plano y un vector no nulo #


v , ambos elementos
nos determinan una recta.

P ~v P ~v

O X

El vector #
v se le denomina vector de direccin de la recta.
#
Para que un punto X est en nuestra recta, vectorialmente ha de ocurrir, que los vectores P X y # v
tengan la misma direccin, o dicho de otra forma, que sean linealmente dependientes. Concretando
#
ms, X est en la recta, si existe R de manera que P X = #
v.
Supondremos que los elementos que nos define las rectas tienen coordenadas P (a1 , a2 ) y #
v = (v1 , v2 ),
y que el punto genrico de la recta es X(x, y).
#
P X = #
v
# #
OX = OP + # v (2.6)
(x, y) = (a1 , a2 ) + (v1 , v2 )
(x, y) = (a1 + v1 , a2 + v2 )

Identificando las componentes de los vectores obtenemos



x = a1 + v1
(2.7)
y = a2 + v2

La ecuacin (2.6), la ms sencilla, se denomina ecuacin vectorial de la recta. Las ecuaciones (2.7) se
denomina ecuaciones paramtricas de la recta, el parmetro es y el significado es bastante grfico,
para diferentes valores de obtenemos diferentes puntos de nuestra recta.

P ~v

2
=
=

=

1
1,

,5
0

=
25

=
1
2,
25

El valor del parmetro = 0 corresponde al punto P .


En la ecuacin (2.7) podemos despejar , siempre que v1 y v2 sean distintos de cero, e igualar las
expresiones obtenidas, con lo que resulta
x a1 y a2
= (2.8)
v1 v2

60
2. Geometra afn en el plano 2.3. Ecuacin de la recta

que llamaremos ecuacin continua de la recta.


Quitando denominadores y agrupando trminos en la ecuacin (2.8), obtenemos la ecuacin

v2 x v1 y + a2 v1 a1 v2 = 0 (2.9)

que llamaremos ecuacin general o implcita de la recta. Si renombramos los coeficientes,






A = v2

B = v1 Ax + By + C = 0, (2.10)


C = a2 v1 a1 v2

La ecuacin (2.10), ecuacin general, nos dice que cualquier par de nmeros (x, y) que verifican la
identidad, corresponden a las coordenadas de un punto de la recta y recprocamente, si P (a, b) es un
punto de la recta ha de verificar la ecuacin (2.10).
Siempre que podamos despejar y, podemos expresar (2.9) de la forma y = mx + n que llamaremos
ecuacin explcita de la recta.
En la ecuacin explcita el valor m, coeficiente de x, siempre y cuando exista se le llama pendiente de
la recta, siendo
v2 A
m= =
v1 B
donde A es el coeficiente de x y B el coeficiente de y.

Nota 2.3.1 (Ecuacin punto-pendiente). A partir de la ecuacin (2.8) podemos expresar, siempre
que sea #
v 1 6= 0:

y a2 = m(x a1 ) (2.11)

Esta forma de expresar la recta se llama punto-pendiente, en la expresin (2.11) queda en evidencia
la pendiente de la recta y un punto de esta.

I Ejemplo 2.3.1. Vamos a determinar, paso a paso, las ecuaciones de la recta que pasa por P (2, 1)
y tiene vector de direccin #
v = (1, 3).
#
El planteamiento inicial era buscar los puntos X que verifican que P X y #
v tienen la misma direccin:
# # #
P X = #
v OX = OP + # v
(x, y) = (2, 1) + (1, 3) Ecuacin vectorial
(x, y) = (2, 1) + (, 3)

x =2
y = 1 + 3
Ecuaciones paramtricas

61
2.3. Ecuacin de la recta 2. Geometra afn en el plano

x2 y1
Como = 1
y= 3
, igualando obtenemos:

x2 y1
= Ecuacin continua
1 3
3(x 2) = 1(y 1)
3x 6 + y 1 = 0
3x + y 7 = 0 Ecuacin general
y = 3x + 7 Ecuacin explcita

La recta tiene pendiente 3 y ordenada en el origen 7. La ecuacin punto-pendiente tiene la forma:

y 1 = 3(x 2)

/
Nota 2.3.2. En el ejemplo anterior si se determinase la ecuacin de la recta a partir del mismo punto
# = k #
P (2, 1) pero con un vector de direccin de la forma w v , como es de esperar obtendramos una
ecuacin equivalente a la obtenida en el ejemplo

 La anterior observacin hace que ocasiones incurramos en el error de considerar que las rectas

P (6, 3) Q(2, 1)
r : # s : #
v = (9, 3) v = (9, 3)

son coincidentes. El efecto de dividir por un nmero las coordenadas de un vector, no modifica su
direccin, que es lo que nos interesa cuando determinamos la ecuacin de la recta. La divisin de las
coordenadas de un punto por un nmero, hace que tengamos un punto diferente del que tenamos. Sin
embargo, como ya comentamos, equivalente a la recta r es:

P (6, 3)
t : #
w = (3, 1)

I Ejemplo 2.3.2. Vamos a hacer lo mismo, en una situacin un poco conflictiva, ahora consideramos
el vector de direccin #
v = ( 12 , 0) y el punto P ( 12 , 32 ).

P ~v
~u2

O ~u1

Antes de determinar las ecuaciones de la recta, observamos que esta tiene la misma direccin que
el primero de los vectores de la referencia, es paralela a la direccin del vector #
u 1 . Ya sospechamos
como pueden ser las coordenadas de los puntos X de la recta, la primera de las coordenadas puede

62
2. Geometra afn en el plano 2.3. Ecuacin de la recta

ser cualquier valor, pero la segunda coincide siempre con la segunda coordenada de P ( 12 , 32 ), luego
X(x, 32 ), o de otra forma y = 32 .
Determinemos, al igual que en el ejemplo anterior, las ecuaciones:


1 3
1
(x, y) = , + ,0 Ecuacin vectorial
2 2 2
1 1 3
(x, y) = + ,
2 2 2
x = + 1
1
2 2
y = 3
Ecuaciones paramtricas
2

Las ecuaciones paramtricas nos caracterizan los puntos de la recta como ya sospechbamos, la y = 32
y la x puede tomar cualquier valor.
Como v2 = 0, no podemos dar el paso de despejar , pues uno de los coeficientes de es cero, y por
cero no nos est permitido dividir.
Como y = 32 , la pendiente es 0, coeficiente que en este caso acompaa a la x. /
I Ejemplo 2.3.3. De forma similar vamos a determinar las ecuaciones de la recta que pasando por
P ( 45 , 1) tiene vector de direccin #
v = (0, 1).
~v

P
~u2

O ~u1

Observamos que la recta tiene la misma direccin que el segundo de los vectores de la referencia. Las
coordenadas de los puntos X de la recta, tienen la segunda de las coordenadas igual que la segunda
coordenada de P ( 54 , 1), luego X( 54 , y), o de otra forma x = 54 .
Determinemos, al igual que en el ejemplo anterior, las ecuaciones:


5
(x, y) = , 1 + (0, 1) Ecuacin vectorial
4
5
(x, y) = ,1 +
4
x = 5
4
y = 1 +
Ecuaciones paramtricas

Ya vemos como han de ser los puntos de la recta, siempre ha de ocurrir que x = 54 .
No podemos dar el paso de despejar , pues uno de los coeficientes de es cero, y por cero no nos
est permitido dividir.

63
2.3. Ecuacin de la recta 2. Geometra afn en el plano

En esta situacin no podemos despejar la y, con lo cual no disponemos de la ecuacin explcita, en


esta situacin diremos que la pendiente de la recta no existe. /
Nota 2.3.3 (A partir de la ecuacin...). Por como hemos ido construyendo las ecuaciones, vemos
que si tenemos la ecuacin de una recta, en cualquiera de sus formas, podemos extraer informacin
sobre las caractersticas de la recta,

x= a + c
P (a, b) y #
v = (c, d)
y= b + d
xa yb
= P (a, b) y #
v = (c, d)
c d
Ax + By + C = 0 #
v = (B, A)

En los casos anteriores P es un punto de la recta y #v un vector de direccin de esta.


En la situacin de la ecuacin general, las coordenadas de un punto no se pueden extraer directamente
de la expresin de la ecuacin, pero la determinacin de puntos de la recta no es nada dificultoso,
basta dar una valor a una de las variables, por ejemplo x = 0 y despejar la y (siempre que se pueda)
C C
para obtener y = B , con lo que P (1, B ).
Nota 2.3.4 (Rectas paralelas). Hemos visto que si una recta r tiene ecuacin general Ax+By +C = 0,
un vector de direccin r es el vector #
v = (B, A), #
v es vector de direccin de cualquier recta paralela
a r. Vemos que la informacin sobre la direccin de r se almacena en los coeficientes de x e y, cualquier
recta s paralela a la recta r ha de tener en esos coeficientes la misma informacin, luego su ecuacin
general de cualquier recta paralela a r ha de ser de la forma Ax + By + k = 0.

 A partir de la ecuacin de una recta podemos extraer informacin sobre la direccin de la


recta. Si tenemos una recta expresada por su ecuacin en forma continua
xa yb
=
v1 v2
un vector de direccin es #
v = (v1 , v2 ), esto es, los denominadores de las fracciones, siempre que los
numeradores sean de la forma x a y y b. Por ejemplo, un vector de direccin de
1x y3
=
2 3
es el vector (2, 3) y no el (2, 3).

Ejercicio 11. Dada la recta de ecuacin 2x11


= y3
2
. Se piden las coordenadas de un punto de
la recta y las coordenadas de un vector de direccin de la recta.
Ejercicio 12. Determina un vector de direccin de la recta de ecuacin
1 2x 3y 3
=
2 3

64
2. Geometra afn en el plano 2.4. Procedimientos de la geometra afn

Ejercicio 13. En un sistema de referencia, R = {O; # u 1 , #


u 2 }, se llaman ejes a las rectas que
pasan por O y tienen por vector de direccin alguno de los vectores de la base.
Determina las ecuaciones de los ejes.

Ejercicio 14. Demuestra que si la ecuacin general de una recta r es Ax + By + C = 0 con


A, B, C 6= 0, siempre es posible obtener una ecuacin equivalente de la forma

x y
+ =1 (2.12)

y que adems r pasa por los puntos de coordenadas (, 0) y (0, ).


La ecuacin de la forma (2.12) se denomina ecuacin segmentaria de la recta r.

2.4. Procedimientos de la geometra afn

Antes de ponernos a trabajar con todo lo visto, conviene hacer alguna reflexin. Hemos visto que a la
hora de determinar la ecuacin de una recta as como resolver algunos de los problemas bsicos que
han ido apareciendo, la eleccin de una determinada referencia es irrelevante. Por ejemplo en cada
uno de los siguientes casos se toma diferente referencia:

P v# P v#
P v# Q
Q Q
~v ~v
~v

O ~u O ~u O ~u

Los elementos que definen la recta son el vector de direccin # v con coordenadas ( 12 , 41 ) en la
correspondiente base y el punto P con coordenadas ( 12 , 1). La ecuacin de la recta es la misma en
cualquiera de las referencias y el punto Q que est en la recta tiene en cada caso las mismas coordenadas
Q( 32 , 12 ). Hemos visto que el que un punto est en una recta es una caracterstica independiente del
sistema de referencia considerado, es una propiedad afn.
Hay otras propiedades con la misma caracterstica anterior:
1. El que tres puntos estn o no alineados.
2. El paralelismo de rectas o vectores.
3. El punto medio de un segmento.
4. La razn entre segmentos paralelos.

65
2.4. Procedimientos de la geometra afn 2. Geometra afn en el plano

B B

M M

#
v
A #
v
A

O #
u O #
u

Nota 2.4.1 (Recta que pasa por dos puntos). Sabemos determinar las ecuaciones de una recta a
partir del conocimiento de un punto de esta y un vector de direccin. Si sabemos que A y B son
puntos de una recta rAB , podemos determinar las ecuaciones de la recta, pues basta considerar el
# #
vector de direccin AB, si A(a1 , a2 ) y B(b1 , b2 ), tenemos que AB = (b1 a1 , b2 a2 ), y podemos
poner directamente la ecuacin (2.8),
x a1 y a2 x b1 y b2
= =
b 1 a1 b 2 a2 b 1 a1 b 2 a2
Siempre que los denominadores no sean cero.

I Ejemplo 2.4.1. Vamos a determinar la ecuacin de la recta que pasa por los puntos A(1, 2) y
B(3, 4).
#
Determinamos las coordenadas de AB = (3 1, 4 2) = (4, 2), la ecuacin continua es:
x1 y2
=
4 2
#
Tiene igual validez la ecuacin que se obtiene de considerar el punto B y el mismo vector AB. En
este ejemplo podemos considerar el vector de direccin (2, 1), pues tiene la misma direccin que el
#
AB, la ecuacin que se obtiene es equivalente a la obtenida. /

Nota 2.4.2 (Est P en r?). Un problema que se nos puede presentar es el determinar si un
determinado punto P (a1 , a2 ) est en en una recta r, la situacin ms favorable es cuando r est
expresada por su ecuacin general Ax + By + C = 0, pues slo basta sustituir las coordenadas de
P (a1 , a2 ) en la ecuacin y comprobar si se verifica la igualdad.
Si la recta viene expresada por su forma paramtrica,

x = 1 + v 1
y = + v
2 2

hemos de ver si existe un parmetro que nos d el punto, o lo que es lo mismo, comprobar que
podemos determinar ese :
a1 1 a2 2
= =
v1 v2
El cociente nos da el valor del parmetro que nos permite obtener P .

66
2. Geometra afn en el plano 2.4. Procedimientos de la geometra afn

I Ejemplo 2.4.2. Comprobemos si los puntos A(1, 1) y B(12, 3) estn en la recta r : x + 6y 5 = 0.


1 + 6 1 5 = 1 + 6 5 = 0 A r
12 + 6 3 5 = 12 + 18 5 = 1 6= 0 B 6 r
/
Nota 2.4.3 (Los puntos A, B y C, estn alineados?). Podemos resolver esta cuestin de dos formas:
# #
1. Comprobando que los vectores AC y AB tienen la misma direccin (ver 2.2.2.4), para lo cual
# #
hemos de ver si existe de manera que AC = AB. Los vectores elegidos pueden ser otros
cualesquiera.
2. Determinando la recta rAB , la que pasa por A y B, y comprobamos si C est en rAB .
I Ejemplo 2.4.3. Comprobemos si los puntos A(1, 1), B(4, 23 ) y C(13, 3) estn alineados.
# #
Determinamos los vectores AC y AB:
#
AC = (13 1, 3 1) = (12, 2)
#
AB = (4 1, 3/2 1) = (3, 1/2)

# #
Veamos si existe de manera que AC = AB,

12 =3 12 2
(12, 2) = (3, 1/2) = = =4
2 = 21 3 1/2

Podemos concluir que los puntos estn sobre la misma recta.


La segunda forma de enfocar el problema tiene la ventaja, en el caso que estn alineados los puntos,
la determinacin de la recta que contiene a estos. /
Nota 2.4.4 (Haz de rectas). Si tenemos un punto P (a, b), llamamos haz de rectas de P , al conjunto
de todas las rectas que pasan por P .

Considerando la ecuacin punto-pendiente no resulta complicado expresar como han de ser las
ecuaciones de las rectas del haz:
y b = m(x a)
Las anteriores rectas pasan todas ellas por el punto P (a, b), pero hay una recta que pasa por P que
no se considera en la anterior familia, es la recta de ecuacin x a = 0, la que no tiene pendiente.
Las rectas que pasan por P :
y b = m(x a) xa=0

67
2.4. Procedimientos de la geometra afn 2. Geometra afn en el plano

Nota 2.4.5 (Recta paralela a otra). Dada una recta r : Ax + By + C = 0 y un punto P (a1 , a2 ) vamos
a determinar la recta paralela a r que pasa por P .
Como hemos visto en la nota (2.3.4), cualquier recta s paralela a r ha de ser de la forma s :
Ax + By + k = 0, nos falta determinar el valor de k, como P s, se ha de verificar que

Aa1 + Ba2 + k = 0 k = Aa1 Ba2

I Ejemplo 2.4.4. Tenemos la recta r : 2x + 3y 6 = 0, veamos cual es la paralela a r que pasa por
P (5, 6).
Las paralelas a r son de la forma 2x + 3y + k = 0, como queremos que P est contenido en la nueva
recta, y como x = 5 e y = 6,

2 5 + 3 6 + k = 0 10 + 18 + k = 0 k = 28

La recta buscada es r = 2x + 3y 28 = 0. /

Nota 2.4.6 (Punto de corte). Dadas dos rectas r y s, al resolver el sistema formado por las ecuaciones
generales de las dos rectas:
1. Que el sistema de ecuaciones tenga una solucin. Geomtricamente significa que las dos rectas
se cortan en un punto.
2. Que el sistema no tenga solucin. Ello indica que no hay ningn punto que compartan las dos
rectas, solo pueden ser paralelas.
3. Que el sistema tenga infinitas soluciones, o que una de las ecuaciones sea equivalente a la otra.
Las dos rectas son una, o dicho de otra forma son coincidentes.

I Ejemplo 2.4.5. Vamos a determinar el punto de corte de las rectas:

r : 2x y + 5 = 0 s : 3x + 2y 4 = 0.

Resolvemos el sistema

2x y +5=0 5 25
x = ,y =
3x + 2y 4 = 0 7 7

El punto de corte de r y s es ( 57 , 25
7
). /

68
Ejercicios
Ejercicio 1. Tres vrtices consecutivos de un paralelogramo son los puntos A(2, 1), B(4, 7) y
C(3, 2). Halla las coordenadas del cuarto vrtice.
Ejercicio 2. Se sabe que tres vrtices de un paralelogramo son los puntos A(4, 0), B(1, 2) y
C(3, 2). Halla las coordenadas del cuarto vrtice. Encuentra todas las soluciones.
Ejercicio 3. Dados los puntos, A(3, 1) y B(2, 1), determina:
a) Las coordenadas de un punto M simtrico de A respecto a B.
b) Las coordenadas de un punto N simtrico de B respecto a A.
#
Ejercicio 4. Los vectores #
a = (3, 5) y b = (4, x) tienen la misma direccin, cunto vale x?
# # #
Ejercicio 5. Sea OABC un cuadrado. Determinar OA + OB + OC.
Ejercicio 6. Sea ABCD un cuadrado. Sean los vectores # u = AB y #v = AD. Representa los
vectores:

a) #
u+ #
v d) 32 #
u g) 3 #
u + 2 #v u + 23 #
j) # v
b) #
u #
v e) 3 # u #
h) 2 u v #
c) 2 #
u f) 2 u + #
# v i) 32 #
u 52 #
v

No es necesario dibujar todos en el mismo dibujo.


Ejercicio 7. Sea ABCDEF un hexgono regular, representa grficamente los vectores:
# # # # # # # #
a) AB + AF c) AC + EF e) BE F A g) 2AE + 21 AC
# # # # # #
b) AB + BC d) AE AF f) AD AC

Ejercicio 8. Si B1 = { #
u , #
v } es una base de V2 , B2 = { #
u , #
v } es base de V2 ?
#
Si las coordenadas de w en la base B1 son (a, b), qu coordenadas tendr w # en la base B ?
2
# #
Ejercicio 9. Consideramos la base B = { u , v },

#
v
#
u

Busca las coordenadas de los vectores que se indican:


Ejercicios 2. Geometra afn en el plano

#
w1
#
w #
w
2 4

#
w3

Ejercicio 10. Consideramos la base B = { #


u , #
v },

#
v
#
u

Busca las coordenadas de los vectores que se indican:

#
w1
#
w #
w
2 4

#
w3

Ejercicio 11. Tenemos en el plano los puntos con coordenadas A(3, 7), B(8, 26/3) y C(24, 25),
estn alineados?
Ejercicio 12. Tenemos en el plano los puntos con coordenadas A(1, 2), B(8, 12), C(5, 4) y
D(2, 6), son vrtices de un paralelogramo ABCD?
Ejercicio 13. Tenemos el segmento de vrtices A(1, 5) y B(8, 2). Determina el punto M
(mitad del segmento). Determina los puntos T1 y T2 que dividen al segmento en tres segmentos iguales.
Ejercicio 14. Si ABCD es un paralelogramo, siendo A(1, 1), B(2, 1) y C(3, 2). Determina
las coordenadas del vrtice D.
# #
Ejercicio 15. Los vectores AB y CD son equivalentes. Si A(1, 3), B(5, 7) y C(8, 4) determina
las coordenadas del punto D.

Ejercicio 16. A(2, 3), B(3, 0) y C(5, 0) son vrtices consecutivos de un hexgono regular.
Halla las coordenadas de los dems vrtices.
# #
Ejercicio 17. Considera un tringulo equiltero ABC y una base de V2 formada por AB y AC.
Usa una trama triangular y dibuja los vectores: (2, 0), (0, 3), (1, 1), (1, 1) y ( 21 , 12 ).
Ejercicio 18. A partir de la figura:

70
2. Geometra afn en el plano Ejercicios

#u v#

w#

#
z

a) Son linealmente dependientes # y #


w v? # combinacin lineal de #
d) Es w u y #
v?
b) Son linealmente dependientes # #
u y v? # # #
e) Es w combinacin lineal de u y z ?
c) Son linealmente dependientes #
u y #
z? f) Es #
z combinacin lineal de w# y #
v?
#
Ejercicio 19. Sean las bases B1 = { #
u , #
v } y B2 = #
n o
a , b de la figura.

#
#
v
b

#
u #
a

a) Halla las coordenadas de los vectores de B2 respecto de B1 .


b) Dibuja el vector #
r = (2, 2) en B1 .
c) #
Dibuja el vector s = (1, 1) en B2 .
d) Qu coordenadas tiene #r en B2 ?
e) #
Qu coordenadas tiene s en B1 ?
#
Ejercicio 20. Sean los puntos A(3, 2), B(1, 4) y C(2, 5). Calcula D sabiendo que AB =
#
3CD.
Ejercicio 21. Sean los puntos A(1, 2), B(2, 2) y C(1, 1).
# #
a) Calcula las coordenadas de D sabiendo que BD = CA.
b) Comprueba que los puntos medios de los segmentos AB y CD coinciden.
c) Divide el segmento AB en cuatro partes iguales.
# #
Ejercicio 22. Sea un tringulo equiltero OBC y M el punto medio de BC. Ponemos b = OB
# # # # #
y #
c = OC. Construye los vectores m# = OM y p = OM + 2M B y ponlos como combinacin lineal
# #
de los vectores b y c .

71
Ejercicios 2. Geometra afn en el plano

#
Sea el sistema de referencia {O; b , #
c }, construye el punto A(1, 1) y determina las coordenadas de los
puntos que dividen al segmento AM en tres partes iguales.
Ejercicio 23. Halla las coordenadas del simtrico de A(3, 5) respecto del punto C(4, 20).
Ejercicio 24. Halla el valor de k para que los puntos A(3, 1), B(2, 4) y C(k, 7) estn alineados.
Ejercicio 25. El punto M (3, 6) es el punto medio de un segmento en el que uno de los vrtices
tiene coordenadas A(5, 8). Determina las coordenadas del otro vrtice.
Ejercicio 26. Calcula las coordenadas del simtrico del punto P (1, 12 ) con respecto al punto
Q(5, 6).
Ejercicio 27. Halla las coordenadas de los puntos que dividen al segmento de extremos A(1, 2)
y B(10, 20) en cuatro segmentos iguales.
Ejercicio 28. Utilizando una trama cuadriculada, considera la base B = { # u , #
v }, como en la
figura y dibuja los vectores de componentes: (1, 1), (2, 3), (2, 1), (3, 2), (3, 2), (5, 0), ( 32 , 52 )
y (2, 13 ).

#
v
#
u

Ejercicio 29. Sean los puntos M (3, 2), N (1, 1) y P (4, 2) los puntos medios de los lados
de un tringulo. Halla las coordenadas de sus vrtices y las de su baricentro.
Ejercicio 30. Halla un punto situado sobre el segmento AB de modo que equidiste de A los 3/8
de la longitud del segmento AB, siendo A(7, 4) y B(17, 8).
Ejercicio 31. Sea ABCD un paralelogramo, A(2, 3), B(4, 3) y C(5, 5). Calcula las coorde-
nadas de D y el centro del paralelogramo.
Ejercicio 32. Sea ABCD un paralelogramo de centro O(0, 0). Sabemos que el lado CD lo
determinan los puntos C(3, 2) y D(5, 8). Calcula las coordenadas de A y B.
Ejercicio 33. En el grfico, R = {O; #
u , #
v } es el sistema de referencia .

#
v

O #
u

72
2. Geometra afn en el plano Ejercicios

Determina las coordenadas de los puntos marcados.


Ejercicio 34. Sea ABCD un paralelogramo, A(1, 1), B(2, 1) y C(3, 2). Determina las coor-
denadas de D y comprueba que los baricentros de los tringulos ABD y BCD dividen a la diagonal
AC en tres partes iguales.
Ejercicio 35. Sea la estrella adjunta y el sistema de referencia {O; #
u , #
v }, donde O es el centro
de la estrella. Halla las coordenadas de los vrtices.

#
v
O #
u

Ejercicio 36. Sean los puntos A(1, 3) y B(3, 1) dos puntos del plano. Halla el simtrico de A
respecto del punto P (2, 0), llmalo C. Halla el simtrico de B respecto del punto P y llmalo D.
Comprueba que ABCD es un paralelogramo y calcula su centro.
Ejercicio 37. Encuentra la ecuacin vectorial, paramtrica y continua de la recta que pasa por
los puntos A = (3, 2) y B = (1, 1).
Ejercicio 38. Cul es la ecuacin paramtrica de la recta que pasa por los puntos P = (2, 1) y
Q = (1, 2). Para qu valores del parmetro se obtienen los puntos P y Q y el punto medio de P y
Q?.
Ejercicio 39. Escribe la ecuacin explcita de la recta que pasa por:
a) El punto (4, 5) y tiene por vector director (2, 6).
b) Los puntos (3, 1) y (4, 2).
c) El punto (4, 3) y es paralela a la recta 3x + 4y = 5.
Ejercicio 40. Hallar la ecuacin de la recta que pasa por el punto (2, 1) y corta a la parte
positiva del eje de abscisas en el punto A, al de ordenadas en B de forma que la longitud de OA es el
triple que la de OB.
Ejercicio 41. Tres vrtices consecutivos de un paralelogramo son los puntos A(2, 1), B(5, 4) y
C(2, 8), halla en forma general las ecuaciones de las rectas que contienen a los cuatro lados del
paralelogramo.
Ejercicio 42. Prueba que si una recta pasa por los puntos (a, 0) y (0, b) su ecuacin es
x y
+ =1
a b

Ejercicio 43. Un paralelogramo tiene un vrtice en A(3, 3) y dos de sus lados sobre las rectas
r1 x 2y 3 = 0, r2 x + y 6 = 0. Hllese las coordenadas de sus otros vrtices y las ecuaciones
de las rectas determinadas por sus otros lados.

73
Ejercicios 2. Geometra afn en el plano

Ejercicio 44. El centro de un paralelogramo es el punto (3, 3) y dos de sus lados se hallan
sobre las rectas y = 3x y x + y = 4. Halla las coordenadas de los vrtices del paralelogramo.
Ejercicio 45. Comprueba si los puntos A(2, 3), B(2, 3) y C(2, 5) pertenecen o no a la recta
que pasa por P (2, 6) y tiene como vector director #
v = (1, 3). Calcula otros dos puntos de esta
recta.
Ejercicio 46. Estudia si las rectas r y s son paralelas, en caso contrario calcular el punto de
corte:
a) r : x 3y = 8 y s : 3x y = 2
b) r : 2x 3y = 4 y s : 4x + 6y = 4
Ejercicio 47. Calcula el valor del parmetro a para que las rectas (a + 1)x + ay + 4 = 0 y
2ax (2a + 1)y 3 = 0 sean paralelas.
Ejercicio 48. La recta r pasa por los puntos A(3, 1) y B(1, 3). Halla un vector director de r,
su pendiente, su ecuacin en forma continua y su ecuacin en forma segmentaria.
Ejercicio 49. Determina a de forma que las rectas r : ax + (a 1)y 2 = 0 y s = (a + 1)x +
(a 2)y 3 = 0 sean paralelas.
Ejercicio 50. Halla a y b de forma que las rectas r : ax + by 1 = 0 y s : 2x 3y + 4 = 0 sean
paralelas y r pase por el punto A(1, 1).
Ejercicio 51. Los vrtices consecutivos de un paralelogramo son A(1, 2), B(5, 0), C y D. Se
sabe que los lados AD y BC son paralelos a la recta (x, y) = (7, 2) + (1, 1), y que el punto P (6, 4)
pertenece a la recta que pasa por C y D. Halla estos vrtices.
Ejercicio 52. Si la recta r corta a y = 2x corta a las rectas 2y + m = 4x?
Ejercicio 53. Calcula el valor de m para que las rectas r : mx + 2y + 6 = 0, s : 2x + y 1 = 0
y t : x + y = 5 pasen, las tres, por un mismo punto.
Ejercicio 54. Determina m y n sabiendo que la recta 2x + ny = 0 pasa por el punto (1, 2) y es
paralela a la recta mx 2y + 3 = 0.
Ejercicio 55. En el tringulo de vrtices A(2, 2), B(2, 0) y C(2, 4), halla la ecuacin de las
medianas.

74
UNIDAD 3

Geometra mtrica en el plano

En este tema trabajaremos con aspectos mtricos en el plano, esto es, aquellos en los que aparece
de alguna forma la idea de medida. Determinaremos longitudes de segmentos, medidas de ngulos,
distancias entre puntos, distancias entre un punto y una recta y reas de figuras planas. Resolveremos
problemas en los que no haremos ninguno de los anteriores procedimientos, pero en los que el concepto
de medida aparece de alguna manera, aunque de forma no evidente.
Al contrario que con los problemas afines (incidencia y paralelismo) en los cuales la consideracin
del sistema de referencia elegido nos resultaba intrascendente; en los problemas mtricos la eleccin
de un sistema de referencia facilita mucho los clculos que hemos de hacer. Usaremos un sistema de
referencia ortonormal, esto es B = {O; # u 1 , #
u 2 } de manera que

| #
u 1 | = | #
u 2| = 1
# #
u 1 u 2

~u2

O ~u1

La eleccin de un sistema ortonormal, frente a otras referencias, nos da la ventaja de poder usar el
Teorema de Pitgoras.

3.1. Mtrica bsica


En esta seccin extenderemos conceptos bsicos, como distancia y ngulo, a los elementos principales
de la geometra analtica, los puntos y los vectores, para lo cual haremos uso de mtodos elementales.
3.1. Mtrica bsica 3. Geometra mtrica en el plano

3.1.1. Mdulo de un vector


Si tenemos un vector de # v de coordenadas #
v = (a, b), nos planteamos el clculo de su mdulo,
o la comparacin de # v con cualquiera de los vectores del sistema de referencia u#i , esto es con
|u#i | = 1. Siempre podemos tomar un representante de #
v con origen en O y construir el rectngulo de
#
dimensiones a b, del cual v es su diagonal.

b~u2
~v
~u2

O ~u1 a~u1

La longitud de la diagonal es de fcil clculo, sin ms que aplicar el Teorema de Pitgoras, obteniendo:

a2 + b 2 (3.1)

El nmero real a2 + b2 es lo que representamos con la notacin | #
v |.
# # #
Nota 3.1.1 (Mdulo de 0 ). El vector nulo 0 tiene | 0 | = 0. Vemos que la forma de calcular el
#
mdulo para un vector no nulo, frmula (3.1), es vlida en el caso del vector nulo, | 0 | = 02 + 02 = 0.

Ejercicio 1. Representa en una referencia ortonormal los vectores y determina su mdulo.

a) #
v = (3, 4) c) #
v = (0, 4) e) #
v = (2, 0)
# # #
b) v = (1, 2) d) v = ( 2, 2) f) v = (1, 1)
3 4

 Hemos de tener cuidado con la notacin, a partir de ahora nos puede aparecer || y | # v |,
usamos el mismo smbolo para indicar el valor absoluto de un nmero, || y para el mdulo de
un vector, | #
v |.
#
Proposicin 3.1.1 (Propiedades). Sean los vectores # a = (a1 , a2 ) y b = (b1 , b2 ) en V2 , se verifica:
a) | #
a| > 0
#
b) | a | = 0 #
# a = 0
c) | #
a | = || | #
a | con R
# #
d) | a + b | 6 | #
# a | + | b |. Desigualdad triangular

Demostracin:
a) Basta observar que | #

a | = a21 + a22 es siempre un valor no negativo.
b) El valor a2 + a2 , solamente es nulo, si y solo si, #

1 2 a = (0, 0).

76
3. Geometra mtrica en el plano 3.1. Mtrica bsica

c) Como #
a = (a1 , a2 ) entonces

| #

a| = 2 a21 + 2 a22 = 2 (a21 + a22 ) =

a21 + a22 = || | #

= 2 a21 + a22 = || a|

d) Esta desigualdad es consecuencia de la relacin que han de verificar los lados de cualquier tringulo:
#
b

#
a#
b
#a +

# #
Basta ver la relacin entre los vectores #
a , b y #
a + b , para concluir que:
# #
| #
a + b | 6 | #
a| + | b |

#
Nota 3.1.2. Dado un vector # v 6= 0 , siempre podemos encontrar un vector con la misma direccin y
sentido pero con mdulo igual a 1, a dicho vector se le denomina vector unitario, asociado a # v . El
1 # #
vector | #v | v es el vector unitario asociado a v , basta observar

1 1
# = # | #

# v

| v |

v| = 1
|v|

Ejercicio 2. Dado un vector #v = (a, b), demuestra que el vector de coordenadas ( a2a+b2 , a2b+b2 ),
tiene la misma direccin que el vector #
v y mdulo igual a 1.
Dado el vector v = (3, 4), determina un vector con la misma direccin que #
# v y con mdulo igual a 1.

3.1.2. Distancia entre dos puntos


Dados dos puntos A(a1 , a2 ) y B(b1 , b2 ), expresaremos mediante d(A, B), la longitud del segmento
#
AB, que geomtricamente coincide con |AB|.

B
, B)
d(A b2 a 2

A b1 a1

#
Como AB = (b1 a1 , b2 a2 ), podemos expresar
#
d(A, B) = |AB| = (b1 a1 )2 + (b2 a2 )2 (3.2)

Proposicin 3.1.2 (Propiedades). La distancia entre dos puntos verifica las propiedades

77
3.1. Mtrica bsica 3. Geometra mtrica en el plano

a) d(A, B) = d(B, A)
b) d(A, B) > 0
c) d(A, B) = 0 A = B
d) d(A, B) 6 d(A, C) + d(C, B).

Demostracin: Aunque las propiedades son en su mayora geomtricamente evidentes, podemos


demostrarlas a partir de la identidad (3.2).
a) La igualdad d(A, B) = d(B, A) es consecuencia de (bi ai )2 = (ai bi )2 .

b) Consecuencia del valor no negativo de x.

c) Si d(A, B) = 0 (b1 a1 )2 + (b2 a2 )2 = 0 (b1 a1 )2 = 0 y (b2 a2 )2 = 0. De las dos
ltimas igualdades concluimos que ai = bi , con lo que los puntos tiene las mismas coordenadas y
A = B.
d) d(A, B) 6 d(A, C) + d(C, B). Al igual que la desigualdad triangular para el mdulo de vectores,
es consecuencia de la relacin entre los lados de un tringulo, la suma de dos cualesquiera de los
lados de un tringulo es mayor que el otro, a + b > c.

B
b

a
C
c A

Solo se da la igualdad cuando los puntos A, B y C estn alineados.

3.1.3. ngulo de dos vectores


#
Definicin 3.1.1 (ngulo de dos vectores). Si tenemos dos vectores #
a y b V2 no nulos, recordemos
que nos determinan dos ngulos, y 2 ,

#b #b


# #
a
a

#
al menor de ellos es al que llamamos ngulo de los vectores y lo indicaremos con ( #
a , b ).
#
El ngulo = ( #
a , b ) est comprendido entre 0 y .
#
Proposicin 3.1.3 (Determinacin). Si tenemos dos vectores #
a = (a1 , a2 ) y b = (b1 , b2 ) no nulos y
llamamos al ngulo que forman, este se puede determinar mediante:
a1 b 1 + a2 b 2
cos = (3.3)
a21 + a22 b21 + b22

78
3. Geometra mtrica en el plano 3.1. Mtrica bsica

Demostracin: Empezamos con un dibujo

#b #b # #
b a

#
a #
a

#
los dos vectores determinan un tringulo en el que el tercer de los lados, es el vector b #
a (podramos
# #
haber considerado el vector a b ). Apliquemos el teorema del coseno al tringulo que tenemos para
determinar :
# # #
| b #
a |2 = | #
a |2 + | b |2 2| #
a || b | cos

Despejamos cos ,
# #
| #
a |2 + | b |2 | b #
a |2
cos = #
2| #
a || b |
a21 + a22 + b21 + b22 (a1 b1 )2 (a2 b2 )2
=
2 a21 + a22 b21 + b22
a1 b 1 + a2 b 2
=
a21 + a22 b21 + b22

I Ejemplo 3.1.1. Vamos a determinar el ngulo que forman los vectores de coordenadas #
a = (1, 0)
# 1 3
y b = ( 2 , 2 ). Aplicamos directamente (3.3),

1 3
1 2
+ 0 2
cos =  2 =
1 2

3
12 + 02 2
+ 2
1 1
1
= 2 = 2
=
1
1 4
+ 43 1 2

1
El ngulo que buscamos , es tal que cos = 2
, luego = arc cos 1
2
= 120 . /

Ejercicio 3. Determina el ngulo que forma el vector #a = (2, 3) con el primero de los vectores
de la base (ngulo con el semieje positivo OX).
#
Determina el ngulo que forma el vector b = (3, 2) con el segundo de los vectores de la base (ngulo
con el semieje positivo OY ).
#
Proposicin 3.1.4 (Condicin de perpendicularidad). Dos vectores #
a y b V2 no nulos son
perpendiculares, si y solo si, se verifica que a1 b1 + a2 b2 = 0.

79
3.1. Mtrica bsica 3. Geometra mtrica en el plano

#
Demostracin: ) Si #
a y b son perpendiculares

a1 b1 + a2 b2
cos =0 = 0 a1 b 1 + a2 b 2 = 0
2 a21 + a22 b21 + b22

) Recprocamente, si a1 b1 + a2 b2 = 0 cos = 0 y necesariamente los vectores han de forma un


ngulo de 2 .

#
Ejercicio 4. Tenemos dos vectores # a = (2, 3) y b = (2, x). Determina x para que sean
perpendiculares.
Ejercicio 5. Demuestra que dado un vector #
u = (a, b) con a b 6= 0, los vectores #
v = (b, a) y
# #
w = (b, a) son perpendiculares al vector u .
Demostrar que | #
u | = | # #
v | = | w|.
Ejercicio 6. Demuestra que si #u = (a, b) y #
v = (c, d) son perpendiculares, entonces #
u = (a, b)
#
y w = (kc, kd) siendo (k 6= 0) son perpendiculares.

Nota 3.1.3 (Vector caracterstico de una recta). Hemos visto que dada una recta r de ecuacin
Ax + By + C = 0, el vector # v = (B, A) es un vector de direccin de r. El vector # n = (A, B),
se llama vector caracterstico o normal de la recta y tiene la particularidad de indicar la direccin
perpendicular a la que indica el vector #
v . Basta observar que:
#
v #
n = (B, A) (A, B) = AB + AB = 0
A
Llambamos pendiente de r, siempre que B 6= 0, al nmero mr = B

Nota 3.1.4 (Perpendicularidad y pendientes). Tenemos ahora dos rectas, r y s, siempre que podamos
considerar las pendientes, tenemos:

Ecuacin Vector de direccin Pendiente


r Ar x + Br y + Cr = 0 #
v = (B , A ) Ar
mr = B
r r r r

s As x + Bs y + Cs = 0 #
v s = (Bs , As ) As
ms = B s

Nos preguntando por la relacin entre las pendientes de las rectas para que estas sean perpendiculares.
Sabemos que para que las rectas sean perpendiculares se ha de verificar que # v r #
v s = 0, lo que se
traduce:
Ar As
(Br , Ar ) (Bs , As ) = 0 Br Bs + Ar As = 0 1 + = 0 mr ms = 1
Br Bs

Podemos concluir, si una recta tiene pendiente m 6= 0, la pendiente de la recta perpendicular es m1 .

Nota 3.1.5. El ngulo que forma un vector #


v = (a, b) con el vector #
u 1 de la base, es fcil de
determinar observando:

80
3. Geometra mtrica en el plano 3.2. Producto escalar

a
#v
b
#
u2
#
u1

Observamos que tan = ab .


Si tenemos una recta r : Ar x + Br y + Cr = 0, el vector de direccin #
v r = (Br , Ar ) y la pendiente
Ar
mr = Br , el valor mr coincide con tan , donde es el ngulo que forma la recta r con el primer
eje de la base.

3.2. Producto escalar


En nuestra geometra la caracterstica de perpendicularidad tiene una importancia determinante,
pues est relacionada con el clculo de distancias mnimas y otros problemas de inters. El hecho de
#
que dados dos vectores #
a = (a1 , a2 ) y b = (b1 , b2 ) de V2 , la expresin:

a1 b 1 + a2 b 2 ,

sea la que nos indique si son o no perpendiculares, nos obliga a fijarnos en ella y estudiar sus
propiedades.
#
Definicin 3.2.1 (Producto escalar). Llamaremos producto escalar de #
a = (a1 , a2 ) por b = (b1 , b2 ),
#
que denotaremos con #
a b , a la expresin
# #
a b = a1 b 1 + a2 b 2 (3.4)

Nota 3.2.1 (Vector nulo). La anterior definicin es vlida an en el caso de considerar el vector nulo,
#
verificndose que #
u 0 = 0.

 Nuevamente utilizamos
#
el mismo smbolo para indicar diferentes operaciones, producto
# #
de nmeros, v producto de un nmero por un vector y u v producto de escalar entre
vectores. No hacemos distincin alguna pues pensamos que el contexto indica con claridad el campo
en el que nos estamos moviendo. En ocasiones indicaremos cualquiera de los tres productos omitiendo
el smbolo .

3.2.1. Relaciones mtricas del producto escalar


Con anterioridad hemos mencionado, proposicin (3.1.4), la relacin mtrica ms importante del
producto escalar; su uso para identificar la perpendicularidad de vectores.

81
3.2. Producto escalar 3. Geometra mtrica en el plano

Nota 3.2.2. A partir de la igualdad (3.3) y de la definicin de producto escalar:


# #
a1 b 1 + a2 b 2 a b
cos = = # #
a21 + a22 b21 + b22 |a| | b |

podemos expresar este de la forma


# # #
a b = | #
a || b | cos (3.5)
#
siendo el ngulo que forman los vectores #
a y b.
Una situacin particular de la anterior igualdad se da cuando consideramos el mismo vector, en ese
caso, se da la identidad:

| #
a |2 = #
a #
a

Nota 3.2.3 (Interpretacin geomtrica). Podemos dar una interpretacin geomtrica del producto
#
escalar de dos vectores #
a y b , para ellos basta fijarse en la figura,

# #
b b



# #
a # #
a
p #
a b p #
a b

para ver que


# #  # #
a b = | #
a | | b | cos = | #

a |p #a b (3.6)
# # #
siendo p #a b un nmero, que corresponde a la proyeccin del vector b sobre el vector #
a , donde a p #a b
se le asigna un signo teniendo en cuenta la coincidencia de la proyeccin con el sentido del vector # a.
#
Proposicin 3.2.1 (Propiedades). En lo que sigue, # a , b y #
c V2 y R.
# # # #
a) a b = b a
# #
b) ( #
a b ) = ( #a) b
#
c) #
a #
a > 0 adems # a #
a = 0 #
a = 0
# #
d) #
a ( b + # c ) = #
a b + #a #
c
#
Demostracin: En lo que sigue suponemos que # a = (a1 , a2 ), b = (b1 , b2 ) y #
c = (c1 , c2 ).
# # # #
a) a b = a1 b1 + a2 b2 = b a
# #
b) ( #
a b ) = (a1 b1 + a2 b2 ) = a1 b1 + a2 b2 = ( #
a) b

82
3. Geometra mtrica en el plano 3.3. Distancias

c) #
a #
a = a21 + a22 > 0. Adems la anterior expresin solamente es cero, si y solo si, es cero cada uno
de los sumandos ai , pero a2i = 0 ai = 0.
#
d) Partimos de #a ( b + #
c)

# #
a ( b + #
c ) = (a1 , a2 )(b1 + c1 , b2 + c2 ) = (a1 (b1 + c1 ) + a2 (b2 + c2 )) =
#
= (a b + a c + a b + a c ) = #
1 1 1 1 2 2 2 2 a b + #a #
c

#
Ejercicio 7. Tenemos los vectores # a = (3, 1) y b = (1, 4) referidos a una base ortonormal.
#
a) Dibuja los vectores # a y b.
#
b) Dibuja p #a b .
# #
c) Calcula #
a b , | #
a | y | b |.
#
d) Calcula el ngulo que forman los vectores #
a y b.
Ejercicio 8. Demuestra:
# # #
a) ( a b )2 = | #
# a |2 + | b |2 2 #a b
# # #
b) ( #
a + b )( #
a b ) = | #a |2 | b |2
#
Ejercicio 9. Dados dos vectores #
a = (a1 , a2 ) y b = (b1 , b2 ) demuestra que se verifica la
igualdad:
# #
| #
a |2 | b |2 = ( #
a b )2 + (a1 b2 a2 b1 )2 (3.7)

Ejercicio 10. Determina la recta perpendicular a la recta 2x 3y + 5 = 0 que pase por el punto
P (7, 6).

3.3. Distancias
Sabemos que dado un vector #
v = (a, b), en una base ortonormal, el mdulo del vector viene expresado
por

| #
v | = a2 + b 2

Hemos visto tambin en (3.2) como determinar la distancia entre dos puntos A y B

d(A, B) = (b1 a1 )2 + (b2 a2 )2

A partir de las anteriores herramientas bsicas podemos determinar la distancia entre un punto y una
recta en el plano.

83
3.3. Distancias 3. Geometra mtrica en el plano

3.3.1. Distancia punto-recta


Antes de empezar hemos de aclarar lo que entendemos como la distancia de un punto a una recta.
Dada una recta r y un punto P , la distancia entre r y P , que indicaremos con d(r, P ), es la distancia
entre el punto P y el punto de la recta Q, de forma que d(P, Q) sea el menor valor posible.

r
Q

El punto Q ha de ser el punto de corte de la recta r con la perpendicular a r que pasa por P . Vemos
que cualquier otro punto sobre r determina la hipotenusa de un tringulo rectngulo, por supuesto
mayor que d(P, Q). El punto Q se denomina proyeccin de P sobre r.
Lo anterior nos da unas pautas claras para determinar para calcular el punto Q y el valor d(P, Q).
Supongamos que r : Ax + By + C = 0 y P (a, b). Vamos a determinar Q(, ),

r
#
v
Q

#
De Q sabemos que est en r y que QP ha de ser perpendicular a la direccin de la recta r. Como un
vector de direccin r es #
v = (B, A), podemos concretar una de las condiciones que ha de verificar
Q,
#
QP #
v = 0 (a , b ) (B, A) = 0 B A + Ab Ba = 0 (3.8)
Por otra parte Q est en r, con lo que
A + A + C = 0 (3.9)
Tenemos dos ecuaciones con incgnitas y , la resolucin del sistema

A + B +C =0
B A
(3.10)
+ Ab Ba = 0

Nos proporciona el punto Q(, ) buscado.


A C a B2 + b A B B C + a A B b A2
= =
B 2 + A2 B 2 + A2
Solo tendramos que determinar d(P, Q), que como sabemos coincide con el valor buscado, d(P, r).

84
3. Geometra mtrica en el plano 3.3. Distancias

I Ejemplo 3.3.1. Tenemos la recta r : 2x + 3y 5 = 0 y el punto P (1, 4), vamos a determinar la


proyeccin de P sobre r, que llamaremos Q y la distancia de P a r, que sabemos que coincide con
d(P, Q).
#
Suponemos que Q(, ), con lo que 2 + 3 5 = 0. Aparte, como QP = (1 , 4 ) y un vector
de direccin de r es #
v = (3, 2), obtenemos la condicin:
(1 , 4 ) (3, 2) = 0 3 + 3 + 8 2 = 0 3 2 + 5 = 0
Tenemos el sistema:

5
2 + 3 5=0 = 13
5 25

Q ,
3 2 + 5 = 25
13
13 13

Nos falta determinar d(P, Q),


s

10 2 15 2 89
d(P, Q) = 2 + 3 = 2,62
13 13 13
/

Podemos determinar el punto Q, esto es, la proyeccin de P sobre r, de otra forma:


1. Determinando la recta perpendicular a la recta r que pasa por P .
2. Calculando el punto de corte de la recta anterior con la recta r
Habiendo calculado el punto Q, es inmediato el clculo de la distancia. Veamos otra forma de calcular
la distancia d(P, r) con alguna ventaja sobre las anteriores.
Proposicin 3.3.1 (Distancia punto-recta). Dado un punto P (a, b) y una recta r de ecuacin
Ax + By + C = 0, la distancia entre P y r viene determinada por:
|Aa + Bb + C|
d(P, r) = (3.11)
A2 + B 2
Demostracin: Vayamos paso a paso:
P P P

d #
n
r r r
Q Q #
n

1. Hemos de determinar la distancia d = d(P, r).


2. Sea Q(, ) la proyeccin de P sobre la recta r, y # n = (A, B) el vector caracterstico de r.
#
Vamos a determinar |P Q|, con lo que tendramos resuelto el problema. Nos ayudamos del
producto escalar:
# # #
n P Q = | #
n | |P Q|

85
3.4. ngulos 3. Geometra mtrica en el plano

El ngulo que forman los dos vectores o es 0 o 180 , por eso ponemos la doble opcin .
Despejando:
# #
# n PQ
|P Q| = # (3.12)
|n|

3. Es momento de poner a hacer clculos, para ello como el punto Q est en r verifica,

A + B + C = 0 A + B = C (3.13)

Desarrollamos (3.12), obtenemos:


# (A, B) ( a, b) Aa Bb + A + B
|P Q| = = (3.14)
A2 + B 2 A2 + B 2
Teniendo en cuenta (3.13) podemos poner la expresin (3.14) de la forma:
# Aa Bb C Aa + Bb + C
|P Q| = 2 2
= 2 (3.15)
A +B A + B2
#
La ambigedad que hemos venido arrastrando, consecuencia del ngulo de #
n y P Q, no es tal,
#
pues sabemos que |P Q|, es un valor no negativo, podemos poner
|Aa + Bb + C|
d(P, r) =
A2 + B 2

Nota 3.3.1. La ltima forma que hemos visto de determinacin de la distancia entre un punto y
una recta, tiene ciertas ventajas, la ms apreciable, es poder disponer de una expresin que nos d la
distancia sin tener que recurrir a la realizacin de pasos intermedios. Como desventaja, no calculamos
el punto Q, punto de mnima distancia o proyeccin.

Ejercicio 11. Determina la distancia de P (1, 2) a la recta 3x + 4y 5 = 0. Utiliza los tres


mtodos comentados.

Nota 3.3.2 (Distancia entre paralelas). si tenemos dos rectas r y s paralelas, para determinar la
distancia entre ellas basta considerar un punto P de una y calcular la distancia de ese punto a la otra.

3.4. ngulos
Hemos definido el concepto de ngulo de dos vectores, teniendo como referencia el concepto de ngulo
visto en la geometra bsica. Definamos que entendemos el ngulo que forman dos rectas, obviamente
sin desprendernos del concepto de ngulo.
Dos rectas r y s, siempre que se corten, determinan cuatro ngulos iguales dos a dos, al menor de
ellos le llamaremos ngulo de las dos rectas, que denotaremos por (r, s).

86
3. Geometra mtrica en el plano 3.4. ngulos

Si las rectas son paralelas diremos que forman un ngulo de 0 . El ngulo que forman dos rectas
siempre est comprendido entre 0 y /2 radianes.
Si r tiene vector caracterstico #
n r y #
n s es el de s, pueden darse dos situaciones:
#
ns #
ns
#
nr s s

r r

1 # 2
nr

1. La situacin ms favorable, el ngulo de los vectores caractersticos de las rectas s y r coincidan


con el ngulo que forman las rectas. En esta situacin
cos(r, s) = cos( #
n r , #
n s) (3.16)

2. La menos favorable, el ngulo que forman los vectores caractersticos de las rectas no coincide con
el que forman las rectas, es el suplementario. En este caso se verifica que (r, s) = ( #
n r , #
n s ),
cos(r, s) = cos( ( #
n , #
n )) = cos( #
r s n , #
n ) r s (3.17)

En las dos situaciones anteriores, vemos que siempre hay que calcular cos( #
n r , #
n s ) y en el peor de
los casos cuan ese valor sea negativo (3.17), considerar el valor cambiado de signo que nos dara el
ngulo buscado, resumidamente podramos poner una expresin que englobara las dos situaciones:
cos(r, s) = | cos( #
n , #
n )|r s (3.18)

Luego
| #
n r #
n s|
cos(r, s) = #
| n r | | #
n s|
I Ejemplo 3.4.1. Vamos a determinar el ngulo que forman las rectas r : 2x 3y + 2 = 0 y
s : 3x + 4y 1 = 0. Consideramos los vectores normales, # n r = (2, 3) y #
n s = (3, 4), tenemos
# #
| n r n s| |(2, 3) (3, 4)|
cos(r, s) = # # = =
| n r | | n s| 2 + (3)2 32 + 42
2

|2 3 + (3) 4| | 6| 6
= = = 0,333
13 25 5 13 5 13
Luego = arc cos(0,333) 70,56 . /

87
3.5. Procedimientos de la geometra mtrica 3. Geometra mtrica en el plano

3.5. Procedimientos de la geometra mtrica


Hemos visto hasta el momento las herramientas bsicas para movernos en la parcela mtrica de la
geometra en el plano. Veremos a continuacin unos procedimientos habituales que nos servirn para
enfrentarnos con problemas no inmediatos.
Antes de empezar conviene decir que como en casi todas las disciplinas las vas para enfrentarse con
una situacin suelen ser diversas, intentaremos comentar las diferentes formas de abordar un mismo
problema y comparar estas.

3.5.1. rea de un tringulo


El problema de calcular el rea de un tringulo no es nuevo, se puede abordar desde mltiples
perspectivas. Supongamos que tenemos las coordenadas de los vrtices de un tringulo, A(a1 , a2 ),
B(b1 , b2 ) y C(c1 , c2 ).
Podemos adoptar un estrategia trigonomtrica, calculamos lo que miden los lados:

a = |BC| b = |AC| c = |AB|

y a partir del conocimiento de los lados, se podra calcular el rea S, directamente haciendo:

S= s(s a)(s b)(s c) (Frmula de Hern)

donde s = a+b+c
2
.
Podemos enfrentarnos al problema desde una perspectiva mtrica, la situacin es la siguiente:

d
rAB
B

Determinaramos la ecuacin de la recta rAB , a continuacin la d = d(C, rAB ) que es la altura que
#
parte de C y por ltimo la medida de nuestra base, que corresponde a |AB|, con lo que nos quedara:
1 #
S = d |AB|
2

3.5.2. Punto simtrico con respecto a r


Hemos determinado el punto simtrico de un punto A con respecto a un punto P . Ahora pretendemos
dada una recta r determinar el simtrico de un punto P con respecto a la recta en r.

88
3. Geometra mtrica en el plano 3.5. Procedimientos de la geometra mtrica

P
r

El simtrico de P es el que se obtiene mediante una reflexin en la que el eje de simetra es la recta r.
El grfico nos da pistas claras de como podemos actuar.
1. Determinamos la recta perpendicular a r que pasa por el punto P .
2. Determinamos el punto de corte de la recta determinada, con la recta r. Ya tenemos el punto
M.
3. El punto M es el punto medio del P y el punto que queremos determinar, el Q. Vectorialmente
# #
podemos plantear 2P M = P Q, lo que nos permite calcular las coordenadas de Q.
I Ejemplo 3.5.1. Tenemos la recta r : 2x 3y + 6 = 0 y el punto P (3, 6), veamos cual es el simtrico
de P con respecto a r.
Conviene tomar una pequea precaucin, comprobar que el punto no est sobre la recta. El el caso
en que P r, el simtrico es el propio punto P . En este caso 2 3 5 6 + 6 6= 0, luego P 6 r.
1. Un vector de direccin de la recta perpendicular a la recta r es # n = (2, 3). La recta
r
perpendicular es
x3 y6
= 3x + 9 = 2y 12 3x + 2y 21 = 0
2 3
2. Determinamos el punto de corte de las dos rectas.

2x 3y +6=0 51 60
x= ,y =
3x + 2y 21 = 0 13 13
Tenemos el punto que medio del segmento P Q, el M .
# #
3. Suponiendo que Q(x, y), sabemos que se verifica 2P M = P Q, que en coordenadas:

63

51 60
24 36
= x
13
2 3, 6 = (x 3, y 6) , = (x 3, y 6) 42
13 13 13 13 y= 13

63 42
Hemos determinado las coordenadas de Q( 13 , 13 ). /

3.5.3. Recta simtrica con respecto a otra


Este problema tiene un planteamiento parecido al anterior, tenemos una recta r que nos determina
una simetra, r es el eje de simetra, y nos planteamos determinar, la transformada de una recta s
mediante la simetra de eje r. Grficamente

89
3.5. Procedimientos de la geometra mtrica 3. Geometra mtrica en el plano

0
B0 s

A
s
B

Si las recta s corta al eje de simetra r, ya podemos decir que el punto de corte A pertenece a la recta
resultado de la simetra, recta s0 . Tomamos un punto cualquiera de la recta s, en nuestro dibujo el
punto B, y determinamos el simtrico de B con respecto a r. El simtrico de B, el punto B 0 pertenece
a la recta que andamos buscando. Tenemos ya dos puntos de s0 , podemos expresar ya la ecuacin de s.

3.5.4. Mediatriz de un segmento


La mediatriz de un segmento AB es la recta perpendicular al segmento AB, que pasa por el punto
medio de AB.
B B

M M

A A

La mediatriz de un segmento AB tiene una propiedad importante, si tomamos cualquier punto de la


mediatriz P , se verifica que d(A, P ) = d(B, P ). En particular d(A, M ) = d(B, M ), cuestin que ya
sabemos, pues M es el punto medio del segmento AB. Utilizaremos esta propiedad para determinar
la ecuacin de la mediatriz del segmento AB.
Supongamos que A(a1 , a2 ) y B(b1 , b2 ). Sea X(x, y) un punto genrico de la mediatriz, se ha de verificar:

d(A, X) = d(B, X) (x a1 )2 + (y a2 )2 = (x b1 )2 + (y b2 )2
(x a1 )2 + (y a2 )2 = (x b1 )2 + (y b2 )2
x(2b1 2a1 ) + y(2b2 2a2 ) + a21 + a22 b21 b22 = 0
I Ejemplo 3.5.2. Vamos a determinar la mediatriz del segmento de extremos A(3, 2) y B(5, 4).
Repitamos el razonamiento, cualquier punto X(x, y) de la mediatriz ha de verificar:

d(A, X) = d(B, X) (x 3)2 + (y 2)2 = (x + 5)2 + (y 4)2
(x 3)2 + (y 2)2 = (x + 5)2 + (y 4)2
x2 6x + 9 + y 2 4y + 4 = x2 + 10x + 25 + y 2 8y + 16
16x + 4y 28 = 0 4x y + 7 = 0
La ecuacin de la mediatriz es 4x y + 7 = 0. /

90
3. Geometra mtrica en el plano 3.5. Procedimientos de la geometra mtrica

Ejercicio 12. Determina las coordenadas del circuncentro del tringulo ABC, donde A(1, 1),
B(5, 0) y C(3, 5).

3.5.5. Bisectriz de un ngulo


La bisectriz de un ngulo, es la semirrecta que divide a este en dos ngulos iguales,

b b
P

el hecho de dividir al ngulo en dos ngulos iguales implica necesariamente, que los puntos de la
bisectriz b, verifican que han de equidistar de las semirrectas que definen el ngulo.
Cuando tenemos dos rectas que se cortan, nos determinan cuatro ngulos, iguales dos a dos. Para
determinar la bisectriz de los ngulos que se forman, buscaremos los puntos P que equidistan , en
este caso, de las rectas que nos definen los ngulos.
r

b2
b1

s
P

Si r : A1 x + B1 y + C1 = 0 y r : A2 x + B2 y + C2 = 0, P (, ) est en la bisectriz si:


d(P, r) = d(P, s)
|A1 + B1 + C1 | |A2 + B2 + C2 |
=
A21 + B12 A22 + B22
A1 + B1 + C1 A2 + B2 + C2
= (3.19)
2 2
A1 + B1 A22 + B22
La expresin (3.19), nos indica la condicin que ha de verificar P (, ), para que est en la bisectriz de
r y s. Observamos que obtenemos dos rectas que verifican tal condicin. Por supuesto la denominacin
que hemos dado a las coordenadas de P es intrascendente, pudiendo expresar las ecuaciones (3.19),
de la forma habitual:
A1 x + B1 y + C1 A2 x + B2 y + C2
= (3.20)
2 2
A1 + B1 A22 + B22

91
3.5. Procedimientos de la geometra mtrica 3. Geometra mtrica en el plano

I Ejemplo 3.5.3. Vamos a buscar las bisectrices de las rectas r : 3x+4y 1 = 0 y s : 5x+12y +1 = 0.
Si P (, ) es un punto de la bisectriz ha de verificar:

d(P, r) = d(P, s)
|3 + 4 1| |5 + 12 + 1|
= 2
2
3 +4 2 5 + 122
3 + 4 1 5 + 12 + 1
=
5 13
13(3 + 4 1) = 5(5 + 12 + 1)
39 + 52 13 = (25 + 60 + 5)
b1 : 14 8 18 = 0
b2 : 64 + 112 8 = 0

Obtenemos las ecuaciones b1 : 7 4 9 = 0 y b2 : 16 + 28 2 = 0.


En este ejemplo observamos algo sospechado, las bisectrices son perpendiculares. Basta extraer los
vectores normales de b1 y b2 ,

#

n = (7, 4)
#
n 1 #
1
#
n 2 = (7, 4) (16, 28) = 0
n 2 = (16, 28)

Nota 3.5.1. Veamos que lo anterior siempre


ocurre. Nos fijamos
en las ecuaciones de las bisectrices
(3.20), renombramos por comodidad m = A1 + B1 y n = A2 + B22 , las ecuaciones de las bisectrices
2 2 2

nos quedan:
A1 x + B1 y + C1 A2 x + B2 y + C2
=
m n
b1 : (nA1 mA2 )x + (nB1 mB2 )y + (nC1 mC2 ) = 0
b2 : (nA1 + mA2 )x + (nB1 + mB2 )y + (nC1 + mC2 ) = 0

Los vectores normales de las bisectrices son #


n 1 = (nA1 mA2 , nB1 mB2 ) y #
n 2 = (nA1 + mA2 , nB1 +
mB2 ), calculemos el producto escalar:
#
n 1 #
n 2 = (nA1 mA2 , nB1 mB2 ) (nA1 + mA2 , nB1 + mB2 ) =
= n2 A21 m2 A22 + n2 B12 m2 B22 = n2 (A21 + B12 ) m2 (A22 + B22 ) =
= (A22 + B22 )(A21 + B12 ) (A22 + B22 )(A22 + B22 ) = 0

Nota 3.5.2. Otra forma de determinar la bisectriz definida por dos rectas, se basa en el hecho de
que la diagonal de un paralelogramo con los lados iguales, divide el ngulo del paralelogramo en dos
partes iguales.
Si tenemos dos rectas con vectores de direccin #
u y #
v y compartiendo un punto P , el vector suma
no marca la direccin de la bisectriz.

92
3. Geometra mtrica en el plano 3.5. Procedimientos de la geometra mtrica

#
v
v#1

P #
u P u#1

Solucionamos el problema si consideramos los vectores unitarios correspondientes, u#1 y v#1 , el vector
suma de estos u#1 + v#1 marca la direccin de la bisectriz.

Ejercicio 13. Determina las coordenadas del incentro del tringulo de vrtices A(1, 1), B(0, 2)
y C(2, 0).
Determina el radio de la circunferencia inscrita.

93
Ejercicios
Ejercicio 1. Determina el valor de a para que las rectas:

r1 : ax + (a 1)y 2(a + 2) = 0
r2 : 3ax (3a + 1)y (5a + 4) = 0

sean paralelas. Determina el valor de a para que sean perpendiculares.


Ejercicio 2. Dados los puntos A(0, 1) y B(1, 2), halla las coordenadas de todos los puntos P
situados sobre la recta x + y = 2 tales que las rectas P A y P B sean perpendiculares.
Ejercicio 3. Sea la base B = { #u , #
v } siendo | #
u | = 2, | #
v | = 6 y el ngulo que forman de 120 .
Calcula el mdulo del vector (2, 5) en dicha base.
Ejercicio 4. Siendo #u = (5, b) y #v = (a, 2) dos vectores en una base ortonormal, halla a y b
sabiendo que son ortogonales y | #
v | = 12.
Ejercicio 5. Calcula h para que el vector (3, h) sea perpendicular al vector (1, 4).
Ejercicio 6. Calcula m para que los vectores (1, m) y (4, m) sean perpendiculares.
Ejercicio 7. Halla el vector #
v = (x, y) para que el ngulo que forman #
v y #
u = (0, 2) sea 45 ,
# #
y el ngulo entre v y w = (1, 1) sea 90 .
Ejercicio 8. Calcula h para que el mdulo del vector (h, 3) sea 5.
Ejercicio 9. Calcula el vector #
v de manera que #
v #
u , | #
v | = 3 y #
u = (1, 2).
Ejercicio 10. Determina b para que los vectores (b, 2) y (1, 2):

a) Sean paralelos. c) Formen un ngulo de 45 .


b) Sean ortogonales. d) Formen un ngulo de 30 .

Ejercicio 11. Considera el tringulo ABC, en donde la recta AB tiene por ecuacin x12y+6 =
0 y el vrtice C tiene por coordenadas (1, 1). Calcula la ecuacin de la recta altura correspondiente al
lado AB y la longitud del segmento altura correspondiente al vrtice C.
Ejercicio 12. Halla el rea del tringulo cuyos vrtices son A(6, 0), B(3, 0) y C(6, 3) y averigua
de qu clase es segn sus lados.
Ejercicio 13. Determina el valor de m para que las rectas mx + y = 12 y 4x 3y = m + 1 sean
paralelas. Halla su distancia.
Ejercicio 14. Averigua si el tringulo de vrtices A(2, 2), B(4, 7), C(9, 4) es issceles.
Ejercicio 15. En los tres tringulos siguientes, averigua si son acutngulos, rectngulos u
obtusngulos:
a) A(2, 0), B(1, 5) y C(3, 3)

b) A(2, 0), B(6, 2 3) y C(2 + 3, 2)
Ejercicios 3. Geometra mtrica en el plano

c) A(3, 1), B(3, 3) y C(0, 6)


Ejercicio 16. Calcula la distancia de los puntos A(2, 5), B(1, 2) y C( 13 , 25 ) a la recta de
ecuaciones paramtricas:

x = 1 + 4t
r:
y = 2 + 3t

Ejercicio 17. Halla los puntos de la recta 7x y 28 = 0 que distan 5 unidades de longitud de
la recta 3x 4y 12 = 0.
Ejercicio 18. Un cuadrado tiene un vrtice en el punto (0, 7) y una de sus diagonales sobre la
recta de ecuacin 3x 2y 6 = 0. Halla el rea.
Ejercicio 19. Un cuadrado tiene un vrtice en el origen y un lado sobre la recta de ecuacin
x 2y + 10 = 0. Halla el rea del cuadrado y la longitud de la diagonal.
Ejercicio 20. Halla las ecuaciones de las rectas paralelas a la recta 3x + 4y + 2 = 0 que distan
una unidad de ella.
Ejercicio 21. Halla el valor de a para que la distancia de O(0, 0) a la recta de ecuacin
y = 1 + a(x 2) sea 2.
Ejercicio 22. Calcula los ngulos que forma la recta r : 2x + 7y 6 = 0 con los ejes coordenados.
Ejercicio 23. Las rectas de ecuaciones 3x + 4y 5 = 0 y px + 7y + 2 = 0 forman un ngulo
cuyo seno vale 3/5. Halla p.
Ejercicio 24. Dados los puntos A(1, 2), B(7, 4) y C(5, 6). Calcula el rea del tringulo de
vrtices ABC. Halla la ecuacin de la recta mediatriz del lado AB.
Ejercicio 25. Halla la ecuacin de la recta que pasa por el punto P (1, 0) y forma un ngulo de

30 con la recta r : x + 2y 3 = 0.
Ejercicio 26. Halla las coordenadas del simtrico del punto P (0, 6) respecto de la recta y =
2x 3.
Ejercicio 27. Halla:
a) Las coordenadas del punto P 0 , simtrico del P (2, 3) respecto del M (2, 5).
b) Las coordenadas del punto A0 , simtrico de A(2, 3) respecto de t : x + 2y + 3 = 0.
c) La ecuacin de la recta r0 , simtrica de r : x 2y + 1 = 0 respecto de la recta s : x y = 0.
Ejercicio 28. Halla un punto de la recta r : x + y 2 = 0 que equidista de los puntos A(1, 3)
y B(2, 1).
Ejercicio 29. Halla las rectas que son incidentes con el punto A(2, 1) y distan una unidad del
origen de coordenadas.
Ejercicio 30. Los puntos A(1, 2) y C(3, 4) son vrtices opuestos de un rombo. El vrtice D est
situado sobre la recta de ecuacin r : x y + 5 = 0. Halla las coordenadas de los otros dos vrtices
del rombo.

96
3. Geometra mtrica en el plano Ejercicios

Ejercicio 31. Los puntos A(1, 2) y B(2, 4) son vrtices consecutivos de un rectngulo. Sabiendo
que el vrtice D, opuesto de B, est sobre la recta y = 3x + 6, halla las coordenadas de los vrtices C
y D, as como las ecuaciones de los lados.
Ejercicio 32. El lado AB de un cuadrado ABCD est sobre la recta de ecuacin r : 2x+3y+7 =
0. Si el centro del cuadrado es el punto M (1, 1), halla las ecuaciones de los restantes lados.
Ejercicio 33. En un tringulo ABC de vrtices A(2, 0), B(4, 1) y C(3, 5) se pide:
a) Ecuacin de la mediatriz del lado AB.
b) Ecuacin de la altura del vrtice C.
c) Longitud de los lados. Qu tipo de tringulo es?
d) rea del tringulo.
Ejercicio 34. Dado el tringulo cuyos vrtices son A(1, 0), B(4, 6) y C(1, 4).
a) Es equiltero, issceles o escaleno?
b) Es rectngulo?
c) Calcula el rea del tringulo.
Ejercicio 35. Halla las ecuaciones de las bisectrices b1 y b2 de los ngulos que forma las rectas
r : 4x 3y + 2 = 0 y s : 24x 7y + 12 = 0.
Ejercicio 36. Halla el simtrico del punto A(1, 4) respecto de la recta x + y + 2 = 0.
Ejercicio 37. Un rombo tiene un vrtice en el punto (6, 1), una diagonal sobre la recta 2x+y3 =
0 y sabemos que su rea vale 20. Halla los restantes vrtices del rombo y la longitud de sus lados.
Ejercicio 38. Dadas las rectas r1 : 4x + 3y + 1 = 0, r2 : 2x y + 3 = 0, halla:
a) El coseno del ngulo que forman.
b) Las coordenadas de un punto de la recta r2 que est a una distancia 6 de la recta r1 (hay dos
soluciones).

Ejercicio 39. Un segmento de recta mide 2 5 y est sobre la recta x 2y + 4 = 0, halla las
coordenadas de los puntos extremos del segmento de la recta si su punto medio es M (4, 4).
Ejercicio 40. El vrtice del ngulo recto de un tringulo est en A(3, 4); la hipotenusa est
sobre la recta 3x + 2y 1 = 0 y uno de los extremos de la hipotenusa est en B(1, 2). Determina las
coordenadas del otro extremo C.
Ejercicio 41. Halla la ecuacin de la mediatriz del segmento determinado por los puntos A(1, 2)
y B(3, 0). Halla, tambin, el ngulo que forma esta mediatriz con el eje de abscisas.
Ejercicio 42. La recta 4x 3y = 12 es la mediatriz del segmento AB. Halla las coordenadas
del punto B, sabiendo que A(1, 0).
Ejercicio 43. Los puntos B(1, 3) y C(3, 3) son los vrtices de un tringulo issceles que
tiene el tercer vrtice A en la recta x + 2y = 15, siendo AB y AC los lados iguales. Calcula las
coordenadas de A y las ecuaciones y las longitudes de las tres alturas del tringulo.
Ejercicio 44. Por el punto A(2, 6) se trazan dos rectas perpendiculares a las bisectrices del

97
Ejercicios 3. Geometra mtrica en el plano

primer cuadrante y del segundo cuadrante. Halla las ecuaciones de dichas rectas y las coordenadas de
los vrtices del tringulo formado por esas dos rectas y la recta de ecuacin 3x 13y = 8.
Ejercicio 45. Halla las ecuaciones de todas las rectas que pasen por el punto P (2, 3) y formen
un ngulo de 45 con la recta 3x 4y + 7 = 0
Ejercicio 46. Halla las ecuaciones de las bisectrices de los ngulos que forma la recta 5x +
12y 60 = 0 con el eje de ordenadas.
Ejercicio 47. Dados los puntos A(4, 2) y B(10, 0), halla el punto de la bisectriz del II y IV
cuadrantes que equidista de ambos puntos.
Ejercicio 48. Dados los puntos A(2, 1), B(3, 5) y C(4, m), calcula el valor de m para que el
tringulo ABC tenga de rea 6.
Ejercicio 49. Un rayo de luz r pasa por el punto de coordenadas (1, 2) e incide sobre el eje de
abscisas formando con este, un ngulo de 135 . Suponiendo que sobre el eje de abscisas se encuentra
un espejo, halla la ecuacin del rayo r y del rayo reflejado en el espejo.
Ejercicio 50. Determina las coordenadas del circuncentro del tringulo ABC, sabiendo que
A(2, 7), B(5, 9) y C(2, 7).
Ejercicio 51. Los puntos A(3, 2) y C(7, 4) son vrtices opuestos de un rectngulo ABCD, el
cual tiene un lado paralelo a la recta 6x y + 2 = 0. Halla las coordenadas de los otros dos vrtices
del rectngulo y las ecuaciones de sus lados.
Ejercicio 52. Los puntos A(2, 1) y C(3, 6) son vrtices opuestos de un rectngulo ABCD.
Sabiendo que B est en la recta de ecuacin x + 4y = 0, halla las coordenadas de los vrtices B y D.
Ejercicio 53. De un rombo ABCD se conocen A(1, 3), B(4, 6), C(4, y). Halla la longitud de
sus diagonales y la medida de los ngulos del rombo.
Ejercicio 54. Un vrtice de un tringulo equiltero es el punto (0, 0); una de sus medianas est
sobre la recta y = 2x + 5. Halla el rea del tringulo y las coordenadas de los otros dos vrtices.
Ejercicio 55. Halla la ecuacin de una recta que forma un ngulo de 120 con el semieje de
abscisas positivo y que dista 2 unidades del origen.
Ejercicio 56. Halla las coordenadas de un punto P situado sobre la recta de ecuacin x+y15 =
0 que equidiste de las rectas y 2 = 0, 3y = 4x 6.
Ejercicio 57. Se considera un trapecio rectngulo ABCD cuyo lado oblicuo es CD. Se sabe que
A(1, 2), B(1, 7) y la ecuacin de la recta CD es x + y 1 = 0. Calcula los vrtices C y D y el rea
del trapecio.
Ejercicio 58. Determina las longitudes de los lados y los ngulos del tringulo cuyos lados se
ecuentran sobre las rectas 2x + y = 2, 5x + 2y = 10 y el eje de ordenadas.
Ejercicio 59. Un hexgonoregular tiene su centro en el origen de coordenadas y uno de sus
lados sobre la recta de ecuacin 2x + y 3 = 0. Halla su rea.
Ejercicio 60. Halla el rea y los ngulos del cuadriltero de vrtices A(0, 3), B(3, 8), C(8, 6) y
D(8, 2).

98
3. Geometra mtrica en el plano Ejercicios


Ejercicio 61. Las rectas mx + y = 0 y 3x y = 1 son medianas de un tringulo equiltero de
lado 2. Halla las coordenadas de sus vrtices.
Ejercicio 62. Halla las coordenadas del punto C de la recta r : x + y + 3 = 0 que es vrtice de
un tringulo issceles, siendo A(1, 3) y B(2, 1) los otros dos vrtices que forman el lado desigual.
Ejercicio 63. Un punto es equidistante de A(6, 10) y B(4, 8). Su distancia al eje OX es doble
que al eje OY . Halla las coordenadas del punto.
Ejercicio 64. Los puntos A(1, 2), B(3, 4) y C(5, 8) son vrtices de un tringulo. Prueba que el
ortocentro, circuncentro y baricentro estn alineados.
Ejercicio 65. Halla las coordenadas del vrtice C de un tringulo ABC, sabiendo que A(2, 1),
B(5, 3) y el ortocentro M (4, 0).
Ejercicio 66. El punto A(2, 5) es vrtice de un tringulo ABC, Las ecuaciones de las rectas
que contienen a las alturas hB y hC son x 2y = 0 y 2x + 5y 13 = 0 respectivamente (hB es la
altura de B y hC la altura del vrtice C). Halla la ecuacin de la recta que contiene al lado a, siendo
a el lado opuesto al vrtice A.
Ejercicio 67. Sea A(4, 3) y la recta q : x + y + 11 = 0.
a) Halla el simtrico A0 de A respecto de q.
b) Sea B(5, 6). Calcula las coordenadas de M que dista de A un tercio del segmento AB, siendo M
del segmento.
c) Halla una recta s paralela a la bisectriz del primer cuadrante y que pase por C(4, 3).
d) Halla el punto P , corte de las rectas s y q.
e) Halla el rea del tringulo A0 M P .
Ejercicio 68. Sean los puntos B(1, 3) y C(4, 0).
a) Halla el punto de corte A de la rectas x 3y = 0 y la que pasa por los puntos B y C.
b) Desde A traza una perpendicular a BC y halla el punto de corte, D, con el eje de ordenadas.
c) Calcula el rea del tringulo BCD.
Ejercicio 69. Halla las coordenadas del punto simtrico del origen de coordenadas respecto de
la recta r : x2
3
= y16
4
.
Ejercicio 70. Sea el punto A(5, 7) y la recta q : x 2y + 4 = 0.
a) Halla el simtrico A0 de A respecto de la recta q.
b) Sea B(5, 1), halla un punto M que diste de B un tercio del segmento AB, siendo M del segmento.
c) Halla una recta s paralela a la bisectriz del primer cuadrante, y que pase por el punto C(5, 0).
d) Halla el punto P , corte de las rectas s y q .
e) Halla el rea del tringulo A0 M P .
Ejercicio 71. Sea ABC un tringulo issceles cuyo lado desigual es AB, con A(2, 1) y B(4, 3).
Halla las coordenadas de C sabiendo que pertenece a la recta r : 2x y + 2 = 0. Calcula su rea.
Ejercicio 72. Los puntos B(1, 3) y C(3, 3) determinan el lado desigual de un tringulo

99
Ejercicios 3. Geometra mtrica en el plano

issceles ABC. Determina las coordenadas del punto A sabiendo que est en la recta x + 2y 15 = 0.
Ejercicio 73. Halla el permetro del tringulo cuyos vrtices son el origen de coordenadas, el
punto A(5, 4) y el simtrico de A respecto de la recta r : 4x + 3y = 7.
Ejercicio 74. Los puntos A(3, 3) y B(5, 3) son vrtices de un paralelogramo ABCD. Sabiendo
que el centro del paralelogramo es el punto M (5, 1), calcula:
a) Las coordenadas de C y D.
b) El ngulo que determinan las diagonales del paralelogramo. Es un rectngulo?
c) Los puntos que dividen a la diagonal AC en tres partes iguales.
Ejercicio 75. Los puntos A(2, 3), B(5, 2) y C(4, 4) son vrtices de un paralelogramo ABCD.
a) Calcula el vrtice D.
b) El paralelogramo, es un rectngulo?
c) Comprueba que los puntos medios de sus diagonales coinciden. Qu punto es ese?
Ejercicio 76. Calcula el rea de un cuadrado que tiene un vrtice en el punto A(3, 5) y uno de
sus lados est en la ecuacin r : 4x + 3y 12 = 0.
Ejercicio 77. Los puntos A(1, 2) y B(2, 4) son vrtices consecutivos de un rectngulo. Sabiendo
que el vrtice D, opuesto de B, est sobre la recta y = 2x + 1, halla las coordenadas de los vrtices
C y D. Calcula tambin su permetro y su rea.
Ejercicio 78. Los puntos A(1, 2) y C(3, 4) son vrtices opuestos de un rombo. El vrtice D est
situado sobre la recta r : x y + 5 = 0. Halla las coordenadas de los otros dos vrtices del rombo.
Ejercicio 79. De un rombo ABCD se conocen dos vrtices opuestos A(2, 3) y C(4, 1), y el
lado AB es paralelo a la bisectriz del primer cuadrante. Halla los otros dos vrtices y el rea del
rombo.
Ejercicio 80. La recta x + y 2 = 0 y una recta paralela a ella que pasa por el punto P (0, 5)
determina junto con los ejes de coordenadas un trapecio issceles. Calcula su rea.
Ejercicio 81. De un cuadrado ABCD centrado en el origen, se sabe que A(3, 2). Halla los otros
vrtices y el rea del cuadrado.
Ejercicio 82. Responde a las siguientes preguntas, razonando las respuestas:
a) Si el producto escalar de dos vectores no nulos es cero,cmo son los vectores?
# # #
b) Si | #
a | = | b |, es cierto qu #
a b #a + b?
c) La pendiente de la recta que pasa por los puntos A(1, 2) y B(2, 3) es igual a 2?
d) Si el vector director de una recta es # v = (3, 5) entonces, su pendiente es 53 ?; y la pendiente
de una recta perpendicular a ella es 53 ?
e) Los vectores # n = (3, 1) y #
n 0 = (6, 2) son vectores normales (perpendiculares) a la recta de
ecuacin y = 3x 2?
f) Los puntos A(1, 0), B(4, 6) y C(1, 4), forman un tringulo?
g) Los puntos A(1, 3), B(2, 5) y C(3, 7), forman un tringulo?

100
3. Geometra mtrica en el plano Ejercicios

Ejercicio 83. Sea ABCD un paralelogramo de centro P (2, 2). Sabemos que el lado CD lo
determinan los puntos C(3, 2) y D(5, 6).
a) Calcula las coordenadas de A y B.
b) Calcula el ngulo qu forma el vrtice A. Es un rectngulo?
c) El rea del tringulo ABC.
d) Es un rectngulo? Es un rombo?
Ejercicio 84. Sea ABCD un paralelogramo de centro O(0, 0). Sabemos que el lado CD lo
determinan los puntos C(3, 2) y D(5, 8). Calcula las coordenadas de A y B.
Ejercicio 85. Halla las ecuaciones de las rectas que pasando por el punto A(1, 2) distan 2
unidades del punto B(3, 1).
Ejercicio 86. Consideramos la grfica de la funcin y = x2 y dos rectas r1 y r2 paralelas al eje
de abscisas; la distancia entre ellas es 1, y r1 est ms cerca del eje de abscisas que r2 . El punto A es
uno de los puntos de interseccin de la parbola con r1 ; el punto B es el punto de interseccin de r2
con el eje de ordenadas, y O es el origen de coordenadas. Calcula el valor del ngulo OAB.

101
UNIDAD 4

Cnicas

Se llama lugar geomtrico a un conjunto de puntos en el plano que verifican una propiedad. Por
ejemplo, la mediatriz de un segmento, que son los puntos del plano que equidistan de dos puntos dados
es un lugar geomtrico. De la misma forma la bisectriz, que son los puntos del plano que equidistan
de dos rectas dadas, es otro de los lugares geomtricos estudiados.
Entre los lugares geomtricos destacan las cnicas, que son las figuras que se obtienen cuando se
toman diferentes secciones en un cono. La situacin ms reconocible es cuando en un cono recto
seccionamos con un plano paralelo a la base de este:

obtenemos un circunferencia. Cuando inclinamos el plano la situacin cambia, y dependiendo de la


inclinacin del plano en relacin con la generatriz, obtenemos diferentes curvas:

Elipse Parbola Hiprbola


4.1. Circunferencia 4. Cnicas

Las cnicas tienen unas propiedades mtricas que las caracterizan como lugares geomtricos en el
plano y que permiten su identificacin algebraica. Veremos que sus ecuaciones son polinmicas y de
grado 2, esto es, de la forma

Ax2 + Bxy + Cy 2 + Dx + Ey + F = 0

4.1. Circunferencia
La circunferencia es un lugar geomtrico que se engloba dentro de las cnicas, pues se obtiene como
seccin de un cono. Pero debido a su importancia se le da un tratamiento individualizado.
Definicin 4.1.1. Una circunferencia es el lugar geomtrico de los puntos del plano que estn a una
distancia r (radio) de un punto C(a, b) que llamaremos centro.

P
r

C r

4.1.1. Ecuacin
Si X(x, y) es un punto de la circunferencia se ha de verificar:

d(C, X) = r (x a)2 + (y b)2 = r
(x a)2 + (y b)2 = r2
x2 + y 2 2xa 2yb + a2 + b2 r2 = 0 (4.1)

La ecuacin (4.1) nos indica, al igual que otras ecuaciones estudiadas, que un punto X est en la
circunferencia si verifica la ecuacin y al revs, si tenemos un par de nmeros (x, y) que satisface la
ecuacin (4.1), podemos tener la garanta que ese par de nmeros corresponden a las coordenadas de
un punto sobre la circunferencia. De forma recproca si tenemos una ecuacin cuadrtica

x2 + y 2 + Ax + By + D = 0 (4.2)

y podemos transformarla en una ecuacin del tipo (4.1), la ecuacin (4.2) corresponde a una circunfe-
rencia.

A = 2a a = A/2
B = 2b b = B/2
s
A2 B 2 A2 B 2
D = a2 + b 2 r 2 D = + r2 r = + D
4 4 4 4

104
4. Cnicas 4.1. Circunferencia

Para que una ecuacin de segundo grado en x e y corresponda a la ecuacin de una circunferencia
hemos de poder poner los coeficientes de x2 e y 2 igual a 1 y adems se ha de verificar que
A2 B 2
+ D >0
4 4
De esta forma obtenemos los elementos que caracterizan a una circunferencia, el centro y el radio:
C(A/2, B/2)
s
A2 B 2
r= + D
4 4
I Ejemplo
4.1.1. Vamos a determinar la ecuacin de la circunferencia de centro C(1, 1) y radio
r = 5.
Sabemos que cualquier punto X(x, y) de la circunferencia que buscamos ha de verificar que:

d(C, X) = 5 (x 1)2 + (y 1)2 = 5
(x 1)2 + (y 1)2 = 5
x2 2x + 1 + y 2 2y + 1 = 5
x2 + y 2 2x 2y 3 = 0
Nos preguntamos si el punto P (0, 1), es un punto de la circunferencia anterior. La respuesta la
obtenemos tras una inmediata comprobacin:
02 + (1)2 2 0 2 (1) 3 = 0
Podemos asegurar que el punto (0, 1) est en la circunferencia. /
Ejercicio 1. En cada caso demuestra:
a) La ecuacin de la circunferencia de centro C(a, 0) y radio r es: (x a)2 + y 2 = r2 .
b) La ecuacin de la circunferencia de centro C(0, b) y radio r es: x + (y b)2 = r2 .
c) La ecuacin de la circunferencia de centro C(0, 0) y radio r es: x2 + y 2 = r2 .
Ejercicio 2. Demuestra que la ecuacin de una circunferencia que pasa por (0, 0) es de la
forma x2 + y 2 + ax + by = 0.
Ejercicio 3. Es x2 +y 2 4x4y +3 = 0 la ecuacin de una circunferencia? En caso afirmativo,
determina el centro y el radio de esta.

4.1.2. Interseccin recta-circunferencia


Dadas una recta r : x + y + = 0 y una circunferencia x2 + y 2 + Ax + By + D = 0, los puntos de
interseccin de ambas son las soluciones (x, y) del sistema

x + y + =0
x2 + y 2 + Ax + By + D = 0

En el anterior sistema caben tres posibilidades:

105
4.1. Circunferencia 4. Cnicas

1. Sin soluciones. La recta es exterior a la circunferencia.


2. Una solucin. La recta es tangente a la circunferencia.
3. Dos soluciones. La recta es secante a la circunferencia.

Nota 4.1.1. La tangente a la circunferencia en un punto y el radio correspondiente al punto de


contacto son perpendiculares.

I Ejemplo 4.1.2 (Determinacin de la tangente). Como la recta tangente a una circunferencia en


una punto P es la recta que solo tiene un punto de contacto con ella, nos da un argumento para
determinar la ecuacin de esa recta.
Tenemos la circunferencia x2 + y 2 2x 4y + 3 = 0 y el punto P (2, 1), vamos a determinar la
ecuacin de la recta tangente a la circunferencia que pasa por P . Las rectas que pasan por P son de
la forma y 1 = m(x 2), determinamos la expresin de los puntos de contacto de esa rectas con la
circunferencia:

x2+ y 2 2x 4y + 3 = 0
y 1 = m(x 2)

x2 + (m(x 2) + 1)2 2x 4(m(x 2) + 1) + 3 = 0


(m2 + 1)x2 + (4m2 2m 2)x + 4m2 + 4m = 0

Si queremos que solo haya un punto de contacto entre la recta y la circunferencia, la ltima ecuacin
solo ha de tener una solucin, para lo cual su discriminante ha de ser igual a cero,

= (4m2 2m 2)2 4(m2 + 1)(4m2 + 4m) = 0


4m2 8m + 4 = 0 m = 1

Luego la ecuacin de la recta tangente tiene ecuacin y 1 = x 2, o x y 1 = 0. /

Ejercicio 4. Determina la ecuacin de la circunferencia con centro en el punto C(3, 4) que


pasa por el punto P (0, 0), determina la ecuacin de la recta tangente a la circunferencia en el punto
P.
Ejercicio 5. Determina la recta tangente a la circunferencia x2 + y 2 4x + 4y + 4 = 0 en el
punto P (4, 2).

106
4. Cnicas 4.1. Circunferencia

4.1.3. Interseccin de dos circunferencias


Si tenemos dos circunferencias, los puntos de corte de las circunferencias se corresponden con las
soluciones del sistema formado por las ecuaciones de las circunferencias:


x2 + y 2 + A1 x + B1 y + C1 = 0
x2 + y 2 + A x + B y + C = 0
(4.3)
2 2 2

Las soluciones del sistema son soluciones de la ecuacin que se obtiene cuando se resta una ecuacin
de otra

(A1 A2 )x + (B1 B2 )y + C1 C2 = 0

La anterior ecuacin es la de una recta, que en el caso en que las dos circunferencias tengan algn
punto en comn esta ha de pasar por l. Dicha recta se llama eje radical de las circunferencias.
Las soluciones del sistema (4.3) son soluciones del sistema

x2+ y 2 + A1 x + B1 y + C1 = 0
(A A )x + (B B )y + C C = 0
(4.4)
1 2 1 2 1 2

que se resuelve sin dificultad despejando una de las incgnitas en la ecuacin segunda y sustituyendo
en la primera de las ecuaciones. Al hacer lo anterior con el sistema (4.4), obtenemos una ecuacin de
segundo grado, pudindose dar una de las siguientes situaciones:

1. Si la ecuacin tiene dos soluciones. Las circunferencias son secantes, esto es, con dos punto en
comn. El eje radical de las dos circunferencias pasa por los puntos de corte
2. Una nica solucin. Las circunferencias tienen un nico punto de contacto. Se dice que son
tangentes. El eje radical pasa por el punto de tangencia de las dos circunferencias y es a la vez
tangente a las dos.
3. Sin soluciones. No tienen puntos en comn.
Ejercicio 6. Determina los puntos de corte de las circunferencias de centro C1 (0, 0) y radio
r1 = 1 y la circunferencia de centro C2 (3/2, 0) y radio r2 = 34 . Determina el eje radical de ambas
circunferencias.
Ejercicio 7. Demuestra que el eje radical de dos circunferencias es perpendicular a la recta
que une los centros de estas.

107
4.2. Elipse 4. Cnicas

4.2. Elipse
La elipse es la curva que se obtiene cuando seccionamos un cono por un plano que no sea paralelo a
ninguna de las generatrices.
Podramos utilizar como definicin de elipse la anterior caracterizacin, pero a efectos prcticos nos
interesa dar una definicin donde estn ms patentes aspectos mtricos.

Definicin 4.2.1. Sean F y F 0 dos puntos del plano se llama elipse de focos F y F 0 al lugar
geomtrico de los puntos del plano P cuya suma de distancias de P a F y P a F 0 es constante. Al
valor de esa constante se designa por 2a.

B
P0 P

A0 A
F0 F

P 00
B0

Nota 4.2.1 (Simetra). Con la definicin dada observamos que la elipse tiene una doble simetra.
Teniendo en cuenta la simetra y con la notacin anterior podemos asegurar que d(F, B) = d(F 0 , B) =
a.
Como B es un punto de la elipse se ha de verificar que d(F, B) + d(F 0 , B) = 2a, la simetra de la
elipse fuerza a que d(F, B) = d(F 0 , B) = a.

Nota 4.2.2 (Elementos). En todas las elipse podemos distinguir los siguientes elementos.
Llamaremos distancia focal a d(F, F 0 ) = 2c.
La recta que contiene al segmento F F 0 corta a la elipse en los puntos A y A0 . El segmento
AA0 se le llama eje mayor de la elipse. La longitud de AA0 se corresponde con 2a, pues
d(A, A0 ) = d(A, F ) + d(A0 , F 0 ) = 2a. Al valor a se le denomina semieje mayor de la elipse.
La mediatriz del segmento F F 0 corta a la elipse en B y B 0 . Al segmento BB 0 se le llama eje
menor de la elipse y su longitud se denota por 2b. Al valor b se le denomina semieje menor de
la elipse.
Al corte de los segmentos AA0 y BB 0 se le llama centro (O) de la elipse.
Los segmentos que unen los focos con un punto P de la elipse se denominan radios vectores del
punto P .

108
4. Cnicas 4.2. Elipse

a
b
c
A0 0
A
F O F
a

B0

Proposicin 4.2.1. Con la notacin que venimos utilizando, en una elipse se verifica que

a2 = b 2 + c 2 .

Demostracin: Si P est en la elipse se verifica que d(P, F ) + d(P, F 0 ) = 2a. Como B est en la elipse
entonces d(B, F ) + d(B, F 0 ) = 2a d(B, F ) = a, aplicando el teorema de Pitgoras al tringulo
OBF , tenemos a2 = b2 + c2 .

I Ejemplo 4.2.1. De una elipse conocemos los focos F 0 (2, 1) y F (4, 2) y uno de sus vrtices
A(6, 3). Determinemos los elementos de la elipse, de manera que podamos hacer un boceto.
El centro es el punto medio de los focos C(1, 21 ). Podemos determinar los parmetros a y c,

1 5 5
a = d(C, A) = (1 + ( 3) =
2 6)2

2 2

1 3 5
c = d(C, F ) = (1 4)2 + ( 2)2 =
2 2
Podemos determinar b,

125 45

b= a2 c 2 = = 5
4 4
Conocemos la posicin de los vrtices B 0 y B, podemos hacer un boceto de la elipse.

y
B

A
C F
x

A0 F0

B0

109
4.2. Elipse 4. Cnicas

Determinemos la ecuacin:


d(F 0 , P ) + d(F, P ) = 5 5

(x + 2)2 + (y + 1)2 = 5 5 (x 4)2 + (y 2)2

(x + 2)2 + (y + 1)2 = 125 + (x 4)2 + (y 2)2 10 5 (x 4)2 + (y 2)2

(x + 2)2 + (y + 1)2 125 (x 4)2 (y 2)2 = 10 5 (x 4)2 + (y 2)2

6y + 12x 140 = 10 5 (x 4)2 + (y 2)2
(6y + 12x 140)2 = 500((x 4)2 + (y 2)2 )
464 y 2 + 144 x y + 320 y 356 x2 + 640 x + 9600 = 0

Es fcil darse cuenta que hay infinitas elipses igual a la que hemos dibujado, pero en otra disposicin,
sera conveniente trabajar con aquellas elipses que tuvieran una ecuacin sencilla. /

4.2.1. Ecuacin reducida

Llamaremos ecuacin reducida de la elipse, a la ecuacin de una elipse que est centrada con respecto
a los ejes coordenados y con los focos en el eje OX.
Tiene la ventaja sobre cualquier otra disposicin de la elipse de la sencillez de su ecuacin.
Busquemos la ecuacin de la elipse con focos F (c, 0) y F 0 (c, 0) y constante 2a.
Si P (x, y) es un punto de la elipse se ha de verificar que:

y
P (x, y)

x
F 0 (c, 0) F (c, 0)

110
4. Cnicas 4.2. Elipse

d(P, F ) + d(P, F 0 ) = 2a

(x c)2 + y 2 + (x + c)2 + y 2 = 2a

(x c)2 + y 2 = 2a (x + c)2 + y 2

(x c)2 + y 2 = 4a2 4 (x2 + 2xc + c2 + y 2 )a + x2 + 2xc + c2 + y 2

(x c)2 + y 2 4a2 x2 2xc c2 y 2 = 4 (x2 + 2xc + c2 + y 2 )a

xc a2 = (x2 + 2xc + c2 + y 2 )a

x2 c2 + 2xca2 + a4 = x2 + 2xc + c2 + y 2 a2
x 2 c 2 + a4 a2 x 2 a2 c 2 a2 y 2 =0
x2 (c2 a2 ) a2 y 2 = a2 (c2 a2 )
x2 (c2 + a2 ) + a2 y 2 = a2 (c2 + a2 )
x 2 b 2 + a2 y 2 = a2 b 2
x2 y 2
+ 2 =1 (4.5)
a2 b
De la ecuacin (4.5) podemos deducir propiedades de simetra de la elipse.
Pues si (x0 , y0 ) es un punto de la elipse (4.5), los puntos (x0 , y0 ) y (x0 , y0 ) verifican igualmente la
ecuacin (4.5). Lo que indica que la elipse es simtrica con respecto a los ejes coordenados, eje mayor
y menor de la elipse.
Ejercicio 8. Demuestra que la elipse centrada en el origen de coordenadas, focos F 0 (0, c) y
F (0, c) y longitud del eje mayor 2a,

y
F (0, c)

F 0 (0, c)

viene dada por la ecuacin

x2 y 2
+ 2 =1 (4.6)
b2 a
donde a2 = b2 + c2 .
Ejercicio 9. Tenemos una elipse con el eje mayor situado sobre la bisectriz del primer cuadrante
de los ejes coordenados. Uno de los vrtices sobre el eje mayor tiene coordenadas A0 (1, 1) y los

111
4.2. Elipse 4. Cnicas

focos tienen coordenadas F 0 (0, 0) y F (5, 5). Calcula las coordenadas del centro de la elipse, la distancia
focal y lo que miden el eje mayor y el eje menor. Determina la ecuacin de la elipse. Cul sera la
ecuacin reducida de la anterior elipse?

 Hemos visto hasta este momento que las elipses quedan caracterizadas por dos valores , los
parmetros a y b. No obstante puede haber elipses iguales con ecuaciones diferentes como las
que corresponden al ejercicio anterior. Observamos que la elipse que tiene la ecuacin que llamamos
reducida tiene las mismas propiedades mtricas que otras con ecuaciones ms complejas.

4.2.2. Elipse trasladada


x2 y2
Si una elipse con ecuacin a2
+ b2
= 1 se traslada,

C(, )
Pt (x0 , y0 )

x
P (x0 , y0 )

siendo el nuevo centro de la elipse el punto C(, ), los puntos de la nueva elipse satisfacen la ecuacin

(x )2 (y )2
+ =1 (4.7)
a2 b2
Basta observar que si Pt (x0 , y0 ) es un punto de la elipse trasladada, el punto P (x0 , y0 ), ha de
2 2
verificar la ecuacin de la elipse original con lo que (x0a)
2 + (y0 b)
2 = 1.
x2 y2
Si la elipse que se traslada tiene de ecuacin b2
+ a2
= 1, y el centro de la nueva es C(, ), la
ecuacin de la elipse trasladada es
(x )2 (y )2
+ =1 (4.8)
b2 a2
Si las ecuaciones (4.7) y (4.8) se quitan parntesis y denominadores se obtienen ecuaciones de la forma

Ax2 + By 2 + Cx + Dy + E = 0 (4.9)

con A y B con el mismo signo.


De forma recproca, podemos transformar una ecuacin del tipo (4.9) completando cuadrados en una
ecuacin en la que el primer miembro sea una suma de cuadrados y el segundo miembro cualquiera
de los valores 1, 0 o 1. En el caso que el segundo miembro de la ecuacin transformada sea 1, la
ecuacin se ajustar a uno de los modelos (4.7) o (4.8).

112
4. Cnicas 4.2. Elipse

I Ejemplo 4.2.2. Determinemos las caractersticas de la elipse de ecuacin: x2 +2y 2 2x+8y +5 = 0.


Iremos transformando la ecuacin teniendo como modelo a conseguir cualquiera de las ecuaciones
(4.7, 4.8).

x2 + 2y 2 2x + 8y + 5 = 0
x2 2x + 2y 2 + 8y + 5 = 0
x2 2x + 1 + 2(y 2 + 4y) + 5 1 = 0
x2 2x + 1 + 2(y 2 + 4y + 4) + 5 1 8 = 0
(x 1)2 + 2(y + 2)2 = 3
(y + 2)2
(x 1)2 + =3
1/2
(x 1)2 (y + 2)2
+ =1
3 3/2


Podemos asegurar que la elipse est centrada en C(1, 2), con valores a = 3 y b = 32 . /

Ejercicio 10. Comprueba si las siguientes ecuaciones corresponden a elipses. En caso afirmativo,
indica en cada caso los elementos de la elipse y haz un dibujo de esta.

a) 4x2 + 25y 2 24x + 100y + 36 = 0 c) 4x2 + y 2 2y 15 = 0


b) 4x2 + 9y 2 16x + 18y 11 = 0 d) x2 + 4y 2 + 2x 8y + 21 = 0.

4.2.3. Excentricidad
c
Llamaremos excentricidad de la elipse al nmero e = .
a

113
4.3. Hiprbola 4. Cnicas

c c c c

e = 0,99216 e = 0,96825 e = 0,92702 e = 0,86603

c c c c

e = 0,78061 e = 0,66144 e = 0,48412 e = 0,0

La excentricidad es un nmero del intervalo [0, 1) que nos mide el grado de achatamiento de la elipse.
Si e 0 entonces la elipse se aproxima a la forma de una circunferencia.
Nota 4.2.3 (rbita terrestre). Es sabido que la rbita terrestre es elptica y que el Sol est situado
en uno de los focos de la elipse, las representaciones que por lo general se hace de esa rbita es muy
elptica, pero la realidad es que la excentricidad de la rbita es e = 0,017, casi circular.

4.3. Hiprbola
La hiprbola es la curva que se obtiene cuando cortamos un cono por un plano que es paralelo a dos
de sus generatrices.
Definicin 4.3.1 (Hiprbola). Sean dos puntos fijos F y F 0 , llamaremos hiprbola de focos F y F 0
al lugar geomtrico de los puntos P cuya diferencia de distancias a F y F 0 es constante. Al valor de
la constante se designa por 2a.
Nota 4.3.1. Cuando en la definicin utilizamos la expresin diferencia de distancia, nos referimos
a considerar la diferencia de la mayor distancia entre la menor, a efectos prcticos, usando el valor
absoluto, podemos expresar lo anterior poniendo |d(P, F ) d(P, F 0 )|, de esta forma estamos con
sidarando las dos posibilidades d(P, F ) d(P, F 0 ) y d(P, F 0 ) d(P, F ).

F0 F
A0 A

114
4. Cnicas 4.3. Hiprbola

P hiprbola |d(P, F ) d(P, F 0 )| = 2a

Nota 4.3.2 (Elementos). Podemos distinguir en una hiprbola los siguientes elementos:
1. Llamaremos distancia focal a la distancia entre los focos, representaremos dicho por valor por
2c. d(F, F 0 ) = 2c.
2. Llameremos eje real de la hiprbola a la recta que une los focos.
3. Llameremos eje imaginario a la mediatriz del segmento F F 0 .
4. El punto medio del segmento F F 0 se llama centro.
5. Llamaremos radios vectores de un punto P de la hiprbola a los segmentos F 0 P y F P .
6. Llamaremos vrtices de la hiprbola a los puntos de corte de esta con el eje real. Los denotaremos
con A y A0 . Adems se verifica que d(A, A0 ) = 2a.

4.3.1. Ecuacin reducida


Llamaremos ecuacin reducida, a la ecuacin de una hiprbola centrada con respecto a los ejes
coordenados.
y

F0 A0 A F x

Busquemos la ecuacin de la hiprbola con focos F (c, 0) y F (c, 0) y constante 2a.

P (x, y) hiprbola |d(P, F ) d(P, F 0 )| = 2a


(x c)2 + y 2 (x + c)2 + y 2 = 2a

(x c)2 + y 2 = 2a + (x + c)2 + y 2

(x c)2 + y 2 = 4a2 4a (x2 + 2xc + c2 + y 2 ) + x2 + 2xc + c2 + y 2

xc a2 = a (x2 + 2xc + c2 + y 2 )
x2 c2 + 2xca2 + a4 = a2 (x2 + 2xc + c2 + y 2 )
x2 (c2 a2 ) a2 y 2 = a2 (c2 a2 )

115
4.3. Hiprbola 4. Cnicas

Llamando c2 a2 = b2 , tenemos

x 2 b 2 a2 y 2 = a2 b 2
x2 y 2
2 =1 (4.10)
a2 b
De la ecuacin (4.10) se puede deducir el carcter simtrico de la hiprbola, basta ver que si (x0 , y0 )
est en la hiprbola, (x0 , y0 ) est igualmente en la hiprbola, con lo que podemos asegurar que
es simtrica con respecto al eje real. Podemos asegurar que el punto (x0 , y0 ) tambin est en la
hiprbola (4.10), lo que nos garantiza la simetra con respecto al eje imaginario.
El valor b, se denomina semieje imaginario de la hiprbola.
Grficamente, como c2 = a2 + b2 , el valor b se identifica con el cateto de un tringulo rectngulo de
hipotenusa c y cateto a.

c
b
a A F

Ejercicio 11. Demuestra que si tenemos una hiprbola con focos F (0, c) y F 0 (0, c) y vrtices
A(0, a) y A0 (0, a),

A0

F0

la ecuacin adopta la forma


y 2 x2
2 =1
a2 b
donde b2 = c2 a2 .

116
4. Cnicas 4.3. Hiprbola

4.3.2. Hiprbolas trasladadas


La ecuacin de una hiprbola con los ejes paralelos a los ejes coordenados y el centro en C(, ), se
ajusta a uno de los modelos

(x )2 (y )2
=1 (4.11)
a2 b2
(y )2 (x )2
=1 (4.12)
a2 b2
Basta hacer las mismas consideraciones ya hechas en el caso de las elipses.
Si desarrollamos las anteriores ecuaciones, llegamos a una ecuacin de la forma

Ax2 + By 2 + Cx + Dy + E = 0 (4.13)

pero ahora A y B con signo opuesto.


Ahora, al igual que antes, podemos a partir de una ecuacin del tipo (4.13) obtener una ecuacin del
tipo (4.11) o (4.12), tratando de completar cuadrados.
Ejercicio 12. Comprueba si las siguientes ecuaciones corresponden a hiprbolas. En caso
afirmativo, indica en cada caso los elementos de la hiprbola y haz un dibujo de esta.

a) 16x2 9y 2 96x 36y 36 = 0 c) 4x2 y 2 + 8x + 4y 4 = 0


b) 9x2 4y 2 54x 16y + 29 = 0 d) 9x2 + 4y 2 + 18x 24y 9 = 0.

4.3.3. Asntotas
x2 y2
Dada la hiprbola de ecuacin a2
b2
= 1, llamaremos asntotas de la hiprbola a las rectas:

b b
y= x y= x
a a

b
a

Las asntotas verifican dos propiedades importantes:

117
4.3. Hiprbola 4. Cnicas

1. Las asntotas y la hiprbola no tienen puntos en comn.


2. Si llamamos ya e yh , a las ordenadas de la asntota y de la rama correspondiente de la hiprbola
para una misma abscisa x, se verifica:

lm ya yh = 0 (4.14)
x

b
a x x

Ejercicio 13. Demuestra las anteriores propiedades.


2 2
Nota 4.3.3. Dada la hiprbola xa2 yb2 = 1 y un punto P (, ) de esta, si se considera la recta rP
que pasa por el origen de coordenadas (0, 0) y el punto P y calculamos la pendiente de rP cuando
hacemos que el valor que nos da es el de la pendiente de la asntota y = ab x.

4.3.4. Excentricidad
x2 y2
Tenemos la hiprbola de ecuacin a2
b2
= 1, llamaremos excentricidad de la hiprbola al valor

c
e=
a
Como c > a entonces e > 1.
Nuevamente nos mide lo achatada que est la hiprbola.

c
c
c
a a a

e = 2,23605 e = 1,56204 e = 1,07703

118
4. Cnicas 4.4. Parbola

4.3.4.1. Hiprbola equiltera

Llamaremos hiprbola equiltera a la hiprbola cuya ecuacin reducida verifica que a = b con lo que
la ecuacin tiene la forma: x2 y 2 = a2 .

b
a


La excentricidad de la hiprbola equiltera es 2.

4.4. Parbola
La parbola es la curva que se obtiene al cortar una superficie cnica con un plano paralelo a una sola
generatriz.

Definicin 4.4.1. Fijamos un punto F y una recta d. Llamaremos parbola de foco F y directriz d,
al lugar geomtrico de los puntos del plano que equidistan de F y d.

A F

a a

Ejercicio 14. Cul es la ecuacin de la parbola con foco F (2, 2) y directriz x + y = 0? Haz
un dibujo de la parbola indicando sus elementos.

Nota 4.4.1 (Elementos). Consideramos elementos significativos de una parbola:


Llamaremos eje de la parbola a la recta perpendicular a la directriz que pasa por el foco.
Llamaremos vrtice al punto de corte de la parbola y el eje de simetra de esta.
Llamaremos parmetro de la parbola al valor p = d(F, d). La forma de la parbola depende
nicamente de este valor.

119
4.4. Parbola 4. Cnicas

4.4.1. Ecuacin reducida


Calcularemos la ecuacin de la parbola de foco F ( p2 , 0) y directriz x = p2 .
Observa que corresponde a una parbola que tiene el vrtice en (0, 0) y parmetro p. Si P (x, y) es un
punto de la parbola ha de verificar d(F, P ) = d(F, d).

p 2 p
x + y2 = x +
2 2
2
p p2
x2 px + + y 2 = x2 + px +
4 4
y 2 = 2px (4.15)

Nota 4.4.2. Hay otras situaciones en las que la ecuacin de una parbola adopta una expresin
sencilla.
Eje coincidente con el eje OX, foco F ( p2 , 0) y directriz x = p2 . Ecuacin: y 2 = 2px.
Eje coincidente con el eje OY , foco F (0, p2 ) y directriz y = p2 . Ecuacin: x2 = 2py.
Eje coincidente con el eje OY , foco F (0, p2 ) y directriz y = p2 . Ecuacin: x2 = 2py.

Ejercicio 15. Comprueba que las anteriores ecuaciones se corresponden con las caractersticas
que se indican. Haz en cada caso un dibujo de cada una de las parbolas.

4.4.2. Parbolas trasladadas


Al igual que con las elipses e hiprbolas, es fcil obtener las ecuaciones de parbolas que estn
trasladadas con respecto a las posiciones de ecuacin reducida.
Eje paralelo al eje OX y vrtice en V (, ). Ecuacin:

(y )2 = 2p(x ) (4.16)

Eje paralelo al eje OY y vrtice en V (, ). Ecuacin:

(x )2 = 2p(y ) (4.17)

Las anteriores ecuaciones son de la forma:

y 2 + Ay + Bx + C = 0 x2 + Ax + By + C = 0 (4.18)

Al igual que hacamos en las otras cnicas, si conseguimos transformar una ecuacin del tipo (4.18)
a una de las formas (4.16) o (4.17). Podremos extraer informacin sobre las caractersticas de la
parbola as como de su situacin.
Ejercicio 16. Las ecuaciones corresponden a parbolas, busca sus elementos y dibjalas:
2
a) y + 6y 6x 9 = 0

120
4. Cnicas 4.4. Parbola

b) y 2 2y 8x + 1 = 0
c) x2 2x y + 3 = 0
d) 3x2 + 12x + 4y + 16 = 0
e) 3x2 + 12x 2y + 16 = 0.
Ejercicio 17. Busca los elementos de la parbola con ecuacin y = ax2 + bx + c. Se ha de
b2
conseguir (x )2 = 2p(y ), completando cuadradros. Resulta V ( b 2a
, c 4a 1
) y p = 2a , a partir
del conocimiento del vrtice y el parmetro se calcula la directriz y el foco.

121
Ejercicios
Ejercicio 1. Determina en forma general la ecuacin de la circunferencia:
a) De centro (3, 2) y radio 5 unidades.
b) De centro (1, 4) y radio 8 unidades.
c) De centro (2, 0) y que pasa por el punto P (3, 4).
Ejercicio 2. Determina el centro y el radio de las circunferencias:
a) x + y 2 4x 6y 12 = 0
2

b) x2 + y 2 + 6x 8y = 0
c) 8x2 + 8y 2 8x + 12y 123 = 0.
Ejercicio 3. Halla la circunferencia concntrica a:
a) x2 + y 2 6x + 2y 6 = 0 y pasa por el punto P (3, 4).
b) x2 + y 2 2x + 0,96 = 0 y que tiene radio 4 unidades.
Ejercicio 4. En cada uno de los casos siguientes, C es el centro y r el radio de la circunferencia.
Calcula la ecuacin de la circunferencia que se ajuste a las especificaciones. Determina, cuando no se
sepa el centro y el radio de la circunferencia.
a) C(0, 0), r = 5.
b) C(3, 2), r = 2.
c) C(0, 0), A(2, 1), donde A, es un punto de la circunferencia.
d) Un dimetro con extremos en A(2, 0) y B(0, 3).
e) Pasa por A(1, 4) y es concntrica a x2 + y 2 + 6x 4y 1 = 0.
f) Pasa por los puntos: A(2, 0), B(3, 0) y C(0, 4).
Ejercicio 5. En cada uno de los casos siguientes, C es el centro y r el radio de la circunferencia.
Calcula la ecuacin de la circunferencia que se ajuste a las especificaciones. Determina, cuando no se
sepa el centro y el radio de la circunferencia.
a) Centro C(3, 1) y tangente a la recta 2x 3y + 4 = 0.
b) Pasa por O(0, 0) y tangente a y = 0 y x + y 1 = 0.
c) Centro C(2, 3) y tangente al eje de abscisas.
d) Centro C(1, 3) y es tangente a la recta 3x + 4y + 10 = 0.
e) El centro est en x + 2y = 0 y pasa por A(0, 1) y B(4, 3).
Ejercicio 6. Calcula la ecuacin de la circunferencia que pasa por el punto A(2, 1) y es
tangente a 3x 2y 6 = 0 en el punto (4, 3).
Ejercicio 7. Halla la ecuacin de la circunferencia que pasa por los puntos P (3, 1) y Q(1, 2),
y tiene su centro sobre la recta r : x 2y + 1 = 0.
Ejercicio 8. Halla la ecuacin de la circunferencia que tiene de centro C(3, 4) y es tangente
a la recta r : x + y + 5 = 0.
Ejercicios 4. Cnicas

Ejercicio 9. Calcula la ecuacin de la circunferencia cuyo centro es el punto (2, 1) y es tangente


a la recta de ecuacin 2x + y 3 = 0. Calcula los puntos de corte de dicha circunferencia con la recta
que pasa por (2, 1) y tiene como vector de direccin (1, 1).
Ejercicio 10. Para cada una de las siguientes circunferencias, calcula el centro y el radio.

a) x2 + y 2 4x = 0 c) x2 + y 2 + 3x 7y = 0
b) x2 + y 2 + 2y 3 = 0 d) 4x2 + 4y 2 4x + 16y 3 = 0.

Ejercicio 11. Sean los puntos A(0, 2) y B(0, 2).


a) Determina la ecuacin de las circunferencias que pasan por A y B.
b) De las anteriores circunferencias, determina el centro y el radio de la que es tangente a la recta
y = 3x + 2.
Ejercicio 12. Halla la ecuacin de la circunferencia que pasa por los puntos A(5, 1), B(9, 5) y
cuyo centro se encuentra sobre la recta 3x 2y 5 = 0.
Ejercicio 13. Halla la ecuacin de la circunferencia que es concntrica a x2 + y 2 8x 4y = 0
y tangente a 7x 24y 55 = 0.
Ejercicio 14. Halla la tangente a la circunferencia:
a) x + y 2 = 16 desde el punto P (6, 0).
2

b) x2 + y 2 4y = 0 desde el punto P (5, 3) .


c) x2 + y 2 2x 2y + 1 = 0 desde el punto P (5, 3) .
Ejercicio 15. Sea la circunferencia 16x2 + 16y 2 + 48x 8y = 43 y la recta s : 8x 4y + 73 = 0.
Halla el punto de la circunferencia ms prximo a la recta.
Ejercicio 16. Determina la ecuacin de la circunferencia tangente a los ejes coordenados y
que pasa por el punto P (2, 5).
Ejercicio 17. Halla las ecuaciones de las tangentes a la circunferencia x2 + y 2 2x + 4y = 0,
paralelas a la recta 3x y = 2.
Ejercicio 18. Determina la ecuacin de la circunferencia tangente a los ejes y que pasa por
(2, 3).
Ejercicio 19. Tenemos la circunferencia de ecuacin x2 + y 2 4x 8y + 10 = 0. Sabemos que
el punto medio de una cuerda de esa circunferencia es A(1, 2). Calcula la longitud de la cuerda.
Ejercicio 20. Halla la ecuacin de la circunferencia que tiene centro en el punto (3, 1) y es
tangente a la recta r : 3x 4y + 5 = 0.
Ejercicio 21. Cul es la ecuacin de la circunferencia circunscrita al tringulo de vrtices:
A(0, 0), B(3, 1) y C(5, 7)?
Ejercicio 22. Halla la ecuacin de la circunferencia que pasa por los puntos A(2, 1) y B(2, 3),
y tiene su centro sobre la recta r : x + y + 4 = 0.

124
4. Cnicas Ejercicios

Ejercicio 23. Sea la recta de ecuacin r : 2x y 3 = 0 y el punto A(4, 2). Calcula la ecuacin
de la circunferencia que pasa por el origen, por el punto A, y por el punto A0 , simtrico de A respecto
de la recta dada.
Ejercicio 24. Encuentra la ecuacin de la circunferencia inscrita en el tringulo formado por
las rectas: r : x + y 5 = 0, s : x + 7y 7 = 0 y t : 7x + y + 14 = 0.
Ejercicio 25. Calcula la ecuacin de la circunferencia que pasa por A(2, 1) y B(2, 3) y tiene
su centro en la recta x + y + 4 = 0. Especifica los elementos caractersticos de la misma.

Ejercicio 26. Halla la ecuacin de la circunferencia de radio r = 2 2 , que pasa por el origen
de coordenadas y cuyo centro est situado en la bisectriz del segundo cuadrante.
Ejercicio 27. Qu posicin ocupan las rectas r : x y = 0 y s : 2x y 6 = 0 respecto de la
circunferencia x2 + y 2 8x + 12 = 0?
Ejercicio 28. Sea la circunferencia de ecuacin x2 + y 2 = 25 , calcula las rectas tangentes a la
circunferencia, paralelas a la recta r : x + 2y 2 = 0.
Ejercicio 29. Sea una circunferencia que pasa por los puntos O(0, 0), A(a, 0) y B(0, b). Qu
relacin debe existir entre a y b para que la circunferencia pase por el punto C(3, 4)? Determina el
centro y el radio de la circunferencia que pasa por C.
Ejercicio 30. Calcula los ejes, vrtices, focos y excentricidad de las elipses:

a) 25x2 + 16y 2 = 400 b) 16x2 + 9y 2 = 576 c) 3x2 + 5y 2 = 15.

Ejercicio 31. Determina la ecuacin reducida en cada uno de las siguientes elipses:
a) Eje mayor 12 y distancia focal 8.
b) Distancia focal 12 y excentricidad 34 .
c) Distancia focal 7 y eje menor 4.
d) Distancia focal 10 y pasa por el punto P (4, 3).
Ejercicio 32. Determina las elipses, cuyo centro, foco y vrtice se da:
a) C(0, 0), F (2, 0) y A(3, 0)
b) C(1, 1), F (1, 2) y A(1, 4)
c) C(0, 0), F (h, 0) y A(4h, 0)
d) C(0, 0), F (0, 4) y A(0, 5)
e) C(3, 2), F (1, 2) y A(2, 2).
Ejercicio 33. Calcula la ecuacin reducida de las elipses que verifican las siguientes condiciones:
a) Pasa por el punto (0, 4) y su excentricidad es 35 .
b) El eje menor mide 4 y el punto (2, 1) pertenece a la curva.

3
c) Pasa por (2, 0) y (1, 2
).

3

d) Pasa por los puntos (1, 2
) y ( 2, 22 ).

125
Ejercicios 4. Cnicas

Ejercicio 34. Halla los puntos de interseccin de la recta y = x 1 con la elipse de ecuacin
x + 4y 2 = 8.
2

Ejercicio 35. Ecuacin de la elipse cuyos ejes son los ejes coordenados (elipse reducida) y
pasa por los puntos (3, 1) y (1, 2). Halla los vrtices, los focos y la excentricidad. Dibuja la elipse
(aproximadamente) indicando cada uno de los elementos anteriores.

Ejercicio 36. Determina la ecuacin reducida de la elipse que pasa por los puntos (2, 12 5 21 ) y
(5, 0). Determina las coordenadas de los vrtices. Determina los puntos de corte de la recta que pasa
por el punto (1, 2) y tiene como vector de direccin (1, 0).
Ejercicio 37. Excentricidad. Se llama excentricidad de una elipse al nmero c/a que est
comprendido entre 0 y 1. Halla las excentricidades de las elipses de ecuaciones:
x2 y2 x2 y2
a) 169
+ 25
=1 b) 169
+ 144
=1

Comprueba despus de dibujarlas, que la excentricidad mide el grado de achatamiento de la elipse;


cuanto mayor es la excentricidad ms achatada es la elipse y cuanto menor, ms redonda.
Ejercicio 38. La rbita de la Tierra es una elipse ocupando el Sol uno de los focos, siendo
la longitud del semieje mayor de la elipse 148 millones de km y la excentricidad 0,0168. Determina
en millones de km la mxima y mnima distancia de la Tierra al Sol durante su movimiento de
traslacin.
Ejercicio 39. En las siguientes elipses, calcula las coordenadas de los focos, de los vrtices y la
excentridad.
x2 y2 x2 y2
a) 16
+ 12
=1 b) 4a2
+ 9b2
=1

Ejercicio 40. En la elipse de ecuacin 4x2 + 9y 2 48x 72y + 144 = 0, determina las
coordenadas del centro, los vrtices, los focos y los semiejes.
3
Ejercicio 41. Una elipse tiene excentricidad 5
y pasa por el punto P (3, 4). Calcula la ecuacin
reducida y la recta tangente en P .
Ejercicio 42. Calcula la recta tangente a la elipse 25x2 + 36y 2 = 900 que es:
a) Paralela a la recta r : 2x y + 12 = 0.
b) Perpendicular a la misma recta.
Ejercicio 43. Calcula las tangentes a la elipse 4x2 +9y 2 = 36 que pasan por el punto P (8, 2).
Ejercicio 44. La recta 5x 4y + 14 = 0 es tangente a la elipse 5x2 + 8y 2 = 28. Calcula las
coordenadas del punto de contacto.
Ejercicio 45. Deduce la ecuacin de una elipse de focos F (0, c) y F 0 (0, c) y eje mayor 2a.
Ejercicio 46. Calcula las coordenadas de los puntos de interseccin de las elipses:
x2 y 2 x2 y 2
+ 2 =1 + 2 = 1.
a2 b b2 a

126
4. Cnicas Ejercicios

Haz un dibujo donde se refleje la situacin que has de resolver.


Ejercicio 47. Latus rectus. En una elipse se llama latus rectus al segmento que es perpendi-
cular al eje mayor y que pasa por un foco. Calcula lo que mide el latus rectus en la elipse de ecuacin
2
x2
a2
+ yb2 = 1.
Ejercicio 48. Determina la ecuacin de la elipse en la que el latus rectus es visto bajo un
ngulo de 90 desde el centro de la elipse y cuyo semieje mayor es a = 4.
Ejercicio 49. De una elipse conocemos los focos F 0 (2, 4), F (6, 1) y uno de sus vrtices A(8, 12 ).
Se pide:
a) Coordenadas del vrtice A0 y del centro de la elipse.
b) Ecuacin de la elipse.
c) Dibujo de la elipse.
2 2
Ejercicio 50. Dada la elipse de ecuacin xa2 + yb2 = 1 y la cuerda P Q que pasa por el foco
derecho, perpendicular al eje mayor, estudia la posibilidad de que el ngulo P AQ, donde A es el
vrtice derecho del eje mayor, sea recto. Es posible? Por qu?
2
x2
Ejercicio 51. Determina los vrtices del tringulo equiltero circunscrito a la curva 4
+ y9 = 1,
sabiendo que uno de sus vrtices est en el eje OY y tiene ordenada positiva.
Ejercicio 52. Determina los vrtices de un tringulo equiltero circunscrito a la curva (elipse)
2
x2
4
+ y9 = 1, sabiendo que uno de los vrtices est en el eje OY y tiene ordenada positiva.
2 2
Ejercicio 53. Dada la hiprbola de ecuacin x16 y4 = 1. Determina los vrtices, focos,
excentricidad y asntotas.
Haz un dibujo aproximado de la hiprbola, sealando cada uno de los elementos anteriormente
calculados.
Ejercicio 54. Halla la ecuacin de la hiprbola conociendo:
a) Eje focal 8 y distancia focal 10.
b) Excentricidad 43 y eje focal 12.
c) Eje focal 12 y pasa por P (8, 14).
Ejercicio 55. Encuentra los vrtices, focos, asntotas y excentricidad de la hiprbola 9x2 16y 2 =
144.
Ejercicio 56. Calcula los ejes, focos, vrtices, asntotas y excentricidad de las hiprbolas:

a) 2x2 y 2 = 9. b) 9x2 16y 2 = 144 c) x2 y 2 = 32.

Ejercicio 57. Una hiprbola tiene por asntotas las rectas y = 34 x y pasa por el punto P (8, 5).
Halla su ecuacin y las rectas tangentes por el punto Q(2, 0).
Ejercicio 58. Cules son las asntotas de la hiprbola 2x2 y 2 = 9? Dibuja la hiprbola
indicando todos sus elementos.
Ejercicio 59. Cuntos puntos en comn pueden tener una recta y una hiprbola?

127
Ejercicios 4. Cnicas

Ejercicio 60. Calcula el rea del tringulo, que la tangente a la hiprbola x y = 50 en el punto
de abscisa x = 5, forma con los ejes de coordenadas.
Ejercicio 61. Una elipse y una hiprbola estn colocadas de modo que los focos de la primera
son los vrtices de la segunda y recprocamente. Encuentra la ecuacin de la hiprbola sabiendo que la
ecuacin de la elipse es
x2 y 2
+ =1
16 9
Ejercicio 62. Halla la ecuacin de la circunferencia que pasa por los focos de la hiprbola
x2
4
y 2 = 4 y por el punto de la hiprbola de ordenada positiva y abscisa x = 8. Calcula el rea del
crculo. Dibuja la circunferencia y la hiprbola.
Ejercicio 63. Halla la ecuacin de la hiprbola, cuya diferencia de distancias a F (4, 0) y a
F (6, 0) vale 6.
Ejercicio 64. Hiprbola equiltera. Toda hiprbola de ecuacin x2 y 2 = r2 se llama
hiprbola equiltera. Demuestra que las asntotas de una hiprbola equiltera son perpendiculares
entre s.
Ejercicio 65. Demuestra que
x2 y 2
= 1 es una hiprbola equiltera c = 2
a2 b2

Ejercicio 66. De una hiprbola conocemos los vrtices A0 (2, 4), A(6, 1) y uno de sus focos
F (8, 12 ). se pide:
a) Coordenadas del otro foco F 0 y del centro de la hiprbola.
b) Ecuacin de la hiprbola.
c) Ecuacin reducida de la hiprbola.
d) Dibujo de la hiprbola.
Ejercicio 67. Con centro el origen de coordenadas y ngulo 90 grados, giramos la hiprbola de
2 2
ecuacin xa2 yb2 = 1, Cul es la ecuacin de la nueva hiprbola?
Cules son las ecuaciones de las asntotas?
2
x2
Ejercicio 68. Hiprbolas conjugadas. Se llama hiprbola conjugada de la hiprbola a2
yb2 =
1, a la que tiene focos F 0 (0, c) y F (0, c) y vrtices A0 (0, b) y A(0, b).
x2 y2
Demuestra que la hiprbola conjugada de a2
b2
= 1 es la que tiene ecuacin
x2 y 2
+ 2 =1
a2 b
Demuestra que tienen las mismas asntotas.
Ejercicio 69. Prueba que el rea del tringulo limitado por una tangente cualquiera a la hiprbola
x y = 8 con los ejes coordenados es constante: Razona la respuesta.
Ejercicio 70. Calcula el vrtice, foco y directriz de las parbolas:

128
4. Cnicas Ejercicios

a) y = x2 d) y 2 = x
b) 3y 2 + x = 0 e) (x 2)2 = 2(y + 1)
c) (x 2)2 = 2(y + 1) f) (y 2)2 = 4(x 1).

Ejercicio 71. Halla la parbola de directriz x + y = 6 y foco F (0, 0) .


Ejercicio 72. Calcula la ecuacin de la parbola con directriz x + y = 0 y vrtice el punto
V (2, 1).
Ejercicio 73. Calcula la ecuacin de las parbolas:
a) De foco F (0, 1) y directriz y = 1.
b) De foco F (2, 1) y directriz 3x 2y = 0.
c) De vrtice V (4, 5) y directriz x = 0.
d) De directriz y = 2 y V (4, 1).
e) De foco F (4, 0) y directriz 5x 12y + 6 = 0.
Ejercicio 74. Halla las rectas tangentes a las parbolas en los puntos que se indican:

a) 2y 2 + x = 0 en x = 8 c) y 2 = x + 7 en x = 2.
b) 3y 2 = 4x en y = 2

Ejercicio 75. Tenemos una parbola con foco en F (2, 3) y vrtice en V (1, 2). Calcula
a) Ecuaciones del eje y de la directriz.
b) Ecuacin de la parbola.
c) Puntos de corte de la parbola con los ejes coordenados.
d) Ecuacin reducida de la parbola.
Ejercicio 76. Sabemos de cursos anteriores que la representacin de la funcin y = ax2 + bx + c
con a 6= 0 corresponde a una parbola. Determina la directriz, el foco y el vrtice.
Ejercicio 77. Tenemos una parbola de con eje x y + 1 = 0, foco en F (2, 3) y vrtice en
V (1, 2). Se pide:
a) Ecuacin de la directriz.
b) Ecuacin de la parbola.
c) Puntos de corte de la parbola con los ejes coordenados.
d) Ecuacin reducida de la parbola.
Ejercicio 78. Halla los puntos de la parbola y 2 5y + 6 = x que equidistan de A(7, 4) y
B(3, 2) .
Ejercicio 79. Halla la ecuacin de la circunferencia que pasa por el vrtice y foco de la parbola
2
y = 8x y cuyo centro est en al recta y = x + 2.
Ejercicio 80. La tangente a la curva y = x2 + 7x 10 en el punto de abscisa x = 3 forma con
los ejes de coordenadas un tringulo. Calcula su rea.

129
Ejercicios 4. Cnicas

Ejercicio 81. Halla el parmetro y las coordenadas del foco de las parbolas y 2 = 12x e
y 2 = 43 x.
Ejercicio 82. En la parbola y = 4x2 24x + 35 se considera la cuerda AB tal que A( 52 , 0) y
B(4, 3). Se pide:
a) La paralela a AB que pasa por el vrtice de la parbola.
b) La circunferencia que pasa por los puntos A y B y tiene su centro en la directriz de la parbola.
Razona las respuestas.
Ejercicio 83. Latus rectus. Prueba que la longitud de la cuerda que pasa por el foco y es
perpendicular al eje de la parbola de ecuacin y 2 = 2px es igual a 2p.
Ejercicio 84. Halla la ecuacin del lugar geomtrico de los puntos del plano P que equidistan
del punto F (0, 1) y la recta r : 3x 4y 2 = 0. De qu cnica se trata?
Qu transformacin habra que hacer a la cnica, para que resulte la de ecuacin y 2 = 2px? Cunto
vale p?
Ejercicio 85. Dados los puntos A(0, 1) y B(4, 5), se pide:
a) Dibujar un punto X tal que el tringulo ABX sea rectngulo con hipotenusa AB.
b) Cuntos puntos hay que cumplan esa condicin?
c) Qu figura forman esos puntos X? Halla su ecuacin.
Ejercicio 86. Halla el lugar geomtrico de los puntos del plano tales que la suma de los cuadrados
de sus distancias a A(1, 0) y a B(4, 4) es igual a 13.
Ejercicio 87. Halla el lugar geomtrico de los puntos del plano tales que la diferencia de los
cuadrados de las distancias a los puntos A(6, 2) y B(11, 10) vale 26.
Ejercicio 88. Halla el lugar geomtrico de los puntos del plano tales que su distancia a la
circunferencia C de centro A(1, 4) y radio 6, es igual a 3.
Ejercicio 89. Halla el lugar geomtrico de los puntos del plano tales que su distancia al punto
A(8, 0) es el doble de su distancia al punto B(0, 6).
Ejercicio 90. Halla el lugar geomtrico de los puntos del plano tales que su distancia al origen
de coordenadas es la mitad de su distancia al punto A(3, 0).
Ejercicio 91. Halla el lugar geomtrico de los puntos P del plano tales que el ngulo AP B es
recto, siendo los puntos A(1, 2) y B(5, 18).
Ejercicio 92. Halla el lugar geomtrico que forman los puntos del primer cuadrante desde los
que se ve el segmento AB bajo un ngulo de 45 , siendo, A(1, 0) y B(0, 2).
Ejercicio 93. Si sobre un segmento como base, dibujas varios tringulos del mismo permetro,
en qu cnica estarn los terceros vrtices? por qu? Deduce la ecuacin reducida de esta curva.
Razona la respuesta.
Ejercicio 94. Halla el lugar geomtrico de los puntos cuya diferencia de distancias a F1 (2, 0) y
F2 (1, 0) es igual a 1. De qu cnica se trata? Halla las ecuaciones de las rectas tangentes en sus
vrtices. Razona las respuestas.

130
4. Cnicas Ejercicios

Ejercicio 95. Sea f (x) = x2 . Se considera el lugar geomtrico de los puntos del plano que son
punto medio del segmento que une dos puntos cualesquiera de la grfica con abscisas diferenciadas en
dos unidades. Halla la ecuacin que define dicho lugar geomtrico. Identifica la cnica obtenida en el
apartado anterior.
Ejercicio 96. Sean los puntos P (2, 0) y Q(0, 1)
a) Determina la ecuacin que define el lugar geomtrico de los puntos del plano que son centro de las
circunferencias que pasan por los puntos P y Q.
b) Una circunferencia de longitud 2 que contiene al origen de coordenadas, est centrada en uno de
los puntos del lugar geomtrico anterior. Calcula las coordenadas del centro de la circunferencia.
Ejercicio 97. Calcula el lugar geomtrico de los puntos del plano cuya distancia a la recta
r : y +1 = 0 es igual a la distancia al punto (0, 1). Clasifica dicha cnica y encuentra sus caractersticas.
Ejercicio 98. Determina la ecuacin del lugar geomtrico de los puntos P del plano para los
# #
que el producto escalar de los vectores P Q y P R es 5, siendo Q(1, 0) y R(3, 0). Identifica la cnica
resultante y sus elementos caractersticos.
Ejercicio 99. Determina el centro y el radio de la circunferencia de ecuacin C : x2 +y 2 2y = 0.
Sabiendo que la distancia de un punto a una circunferencia (cuando es exterior a ella) es la diferencia
entre la distancia al centro y el radio de la circunferencia, determina la ecuacin que define el lugar
geomtrico de los puntos del plano que equidistan de la recta y = 0 y de la circunferencia C. Identifica
la cnica resultante.
Ejercicio 100. Una circunferencia tiene el mismo centro que una elipse, pasa por los focos F y
F 0 y se cortan en cuatro puntos. Sea P uno de esos puntos. Si el eje mayor de la elipse es 15 y el
rea del tringulo F P F 0 es 26, cul es la distancia focal?
2 2
Ejercicio 101. Tenemos la elipse de ecuacin xa2 + yb2 = 1, con excentricidad e y focos F 0 (c, 0)
y F (c, 0). Si tenemos un punto P (x0 , y0 ) de la elipse, demuestra que los radios vectores del punto P
verifican
P F 0 = a + ex0 P F = a ex0
2 y2
Ejercicio 102. Demuestra que la recta tangente a un punto P de la elipse xa2 + b2
= 1, es
perpendicular a la bisectriz de las rectas que pasan por P y por cada uno de los focos.
2 2
Ejercicio 103. Dada la elipse xa2 + yb2 = 1, se sabe que est situada en en el cuadrado de
vrtices (1, 0), (0, 1), (1, 0) y (0, 1), y adems es tangente a cada uno de los lados del cuadrado.
Demuestra que a2 + b2 = 1.
Ejercicio 104. Demuestra que la condicin para que la recta r : Ax + By = AB y la elipse
x2 y2
a2
b2
= 1, sean tangentes es A2 a2 + B 2 b2 = A2 B 2 .
2 2
Ejercicio 105. Determina un punto sobre xa2 + yb2 = 1, de manera que la tangente a la elipse
en ese punto sea paralela a la recta que une los vrtices A y B.
Ejercicio 106. Se considera una parbola cualquiera P y se la traslada una cierta distancia en
la direccin de su eje. Analiza si la parbola P y su trasladada P 0 se pueden cortar.

131
Ejercicios 4. Cnicas

Ejercicio 107. Se pueden definir las cnicas como el lugar geomtrico de los puntos P , cuya
distancia a un punto fijo F es e veces la distancia de P a r, donde r es una recta fija.
r)
d(P, F ) = ed(P,

Al punto F se le llama foco de la cnica, a la recta r se le llama directriz. El nmero e es la


excentricidad.
El tipo de cnica depende del valor de e, as si
e < 1, la cnica es una elipse.
e = 1, la cnica es una parbola.
e > 1, la cnica es una hiprbola.
Trata de comprobar lo anterior para un caso particular, siendo F (4, 2) y r : x 3 = 0.
a) Para e = 2. Busca la ecuacin de la cnica. De qu tipo es? Cules son los elementos significativos
de esta? Calcula la excentricidad.
b) Para e = 1/2. Busca la ecuacin de la cnica. De qu tipo es? Cules son los elementos
significativos de esta? Calcula la excentricidad.
2 2
Ejercicio 108. Tangente. Tenemos la hiprbola de ecuacin xa2 yb2 = 1, sea P (x0 , y0 ) un
punto en la hiprbola. Demuestra que la recta tangente a la hiprbola en el punto P es igual a:
xx0 yy0
2 =1
a2 b

Ejercicio 109. Demuestra que una elipse y una hiprbola con los mismos focos, las rectas
tangentes en los puntos comunes son perpendiculares.
2 2
Ejercicio 110. Si tenemos la elipse de ecuacin xa2 + yb2 = 1, demuestra que la proyeccin de
un foco sobre una tangente cualquiera a la elipse es un punto de la circunferencia x2 + y 2 = a2 .
2 2
Ejercicio 111. Demuestra que la relacin entre la ordenada de un punto de la elipse xa2 + yb2 = 1
y la del punto de igual abscisa de la circunferencia circunscrita x2 + y 2 = a2 , es constante e igual a ab .
2 2
Ejercicio 112. Tenemos la hiprbola xa2 yb2 = 1 con centro O y foco F . Trazamos la circunfe-
rencia con centro en M , punto medio de OF y radio OF . Dicha circunferencia corta a la asntota de
la hiprbola en el punto P . Demuestra que OP = a y que P F = b.

132

You might also like